0% ont trouvé ce document utile (0 vote)
123 vues102 pages

Cas cliniques pour professionnels de santé

Le document contient des informations sur la prise en charge d'un patient présentant une douleur thoracique et une hypotension. Il présente également des questions sur différentes pathologies comme l'œdème pulmonaire aigu, les brûlures ou l'insuffisance rénale chronique.

Transféré par

qualitywood.ak
Copyright
© © All Rights Reserved
Nous prenons très au sérieux les droits relatifs au contenu. Si vous pensez qu’il s’agit de votre contenu, signalez une atteinte au droit d’auteur ici.
Formats disponibles
Téléchargez aux formats PDF, TXT ou lisez en ligne sur Scribd
0% ont trouvé ce document utile (0 vote)
123 vues102 pages

Cas cliniques pour professionnels de santé

Le document contient des informations sur la prise en charge d'un patient présentant une douleur thoracique et une hypotension. Il présente également des questions sur différentes pathologies comme l'œdème pulmonaire aigu, les brûlures ou l'insuffisance rénale chronique.

Transféré par

qualitywood.ak
Copyright
© © All Rights Reserved
Nous prenons très au sérieux les droits relatifs au contenu. Si vous pensez qu’il s’agit de votre contenu, signalez une atteinte au droit d’auteur ici.
Formats disponibles
Téléchargez aux formats PDF, TXT ou lisez en ligne sur Scribd

1. A 70 years old patient present to the ER for chest pain.

n. ECG done ٠ُ‫َ ا‬٠‫ ػخٓخ ك‬70 َٔ‫زِؾ ٖٓ حُؼ‬٣ ٞ٣َٓ


showed an ST elevation on the DI, DII and aVF with a heart rate of َٜ‫ أظ‬.ٍ‫ي‬ُٜ‫ ح‬٢‫حٍة رٔزذ أُْ ك‬ٞ‫ؿَكش حُط‬
68/min. Patient present a hypotension of 70/40mmHg without ST ‫ حٍطلخع‬ٙ‫ طْ اؿَحإ‬١ٌُ‫ حُوِذ ح‬٢٤‫طوط‬
evidence of crackles on pulmonary auscultation, what the best taken ‫َرخص‬ٟ ٍ‫ رٔؼي‬،aVFٝ DIIٝ DI ٠ِ‫ػ‬
in charge? ٢‫خ ك‬ٟ‫ حٗولخ‬ٞ٣َُٔ‫َ ح‬ٜ‫ظ‬٣ .‫وش‬٤‫ ىه‬/ 68 ‫هِذ‬
٠ِ‫َ ػ‬٤ُ‫ٕ ى‬ٝ‫ ري‬mmHg40/70 ّ‫ حُي‬٢‫ـ‬ٟ
a. Give fluid َ٠‫ أك‬ٞٛ ‫ ٓخ‬،١ٞ‫حُو٘و٘ش ػ٘ي حُظٔٔغ حَُث‬
b. Start dobutamine ‫؟‬ًٙ‫ٌٖٔ حطوخ‬٣ ٍ‫هَح‬
c. Start noradrenaline َ‫حث‬ُٞٔ‫ اػطخء ح‬.a
d. Start dopamine ٖ٤ٓ‫طخ‬ٞ‫ر‬ٝ‫ حُزيء رخُي‬.b
ٖ٤ُ‫٘خ‬٣ٍ‫ٍحى‬ُٞ٘‫ حُزيء رخ‬.c
ٖ٤ٓ‫رخ‬ٝ‫ حُزيء رخُي‬.d
Un patient de 70 ans se présente à l‟urgence pour des douleurs
thoraciques. L‟ECG effectué a montré une élévation du ST sur l‟ID, le
DII et l‟aVF avec une fréquence cardiaque de 68 / min. Le patient
présente une hypotension de 70/40mmHg sans preuve de crépitements
sur auscultation pulmonaire, quelle est la meilleure prise en charge ?
a. Donner du liquide
b. Commencer dobutamine
c. Commencer noradrénaline
d. Commencer dopamine

2. The acute pulmonary edema of hemodynamics origin except ٍ‫ي‬ُٜٔ‫ش حُلخىس ًحص ح‬٣ٞ‫ًٓش حَُث‬ُٞ‫ح‬
:‫ ٓخ ػيح‬٢ِ٣ ‫ ًَ ٓخ‬٢ٛ ،١ٞٓ‫ حُي‬٢ٌ٤ٓ‫٘خ‬٣‫حُي‬
a. Reflect a severe left heart failure ‫َٔ كخى‬٣‫ٍ هِذ أ‬ٜٞ‫ طؼٌْ ه‬.a
b. Can complicate a severe mitral stenosis ‫ي‬٣‫ ٗي‬٢‫ن طخؿ‬٤٠‫خ ط‬ٜ‫خػلظ‬٠ٓ ٖٓ .b
c. It‟s the result of high pulmonary capillary pressure ٢‫ ك‬٢‫ـ‬ٟ ‫ ٗخطـش ػٖ حٍطلخع‬٢ٛ .c
d. All answers are false ‫ش‬٣ٞ‫ش حَُث‬٣ٞٓ‫َحص حُي‬٤‫حُ٘ؼ‬
e. All answers are true ‫جش‬١‫غ حإلؿخرخص هخ‬٤ٔ‫ ؿ‬.d
‫لش‬٤‫ل‬ٛ ‫غ حإلؿخرخص‬٤ٔ‫ ؿ‬.e
L‟œdème pulmonaire aigu d‟origine hémodynamique sauf

a. Refléter une insuffisance cardiaque gauche sévère


b. Peut compliquer une sténose mitrale sévère
c. C‟est le résultat d‟une pression capillaire pulmonaire élevée
d. Toutes les réponses sont fausses
e. Toutes les réponses sont vraies

1
3. In the patient with major burns, choose the right statement: َ‫ حهظ‬،‫َس‬٤‫م ًز‬َٝ‫خد رل‬ٜٓ ٞ٣َُٔ ‫رخُ٘ٔزش‬
:‫لش‬٤‫ل‬ُٜ‫حُؼزخٍس ح‬
a. Selective decontamination of the digestive tract has ُ‫خ‬ٜ‫ ُِـ‬٢‫َ حالٗظوخث‬٤ٜ‫ ػزض إٔ حُظط‬.a
been demonstrated to reduce the incidence of burn ٢‫ع اٗظخٕ ك‬ٝ‫وَِ ٖٓ كي‬٣ ٢ٔ٠ُٜ‫ح‬
wound infection. .‫م‬َٝ‫حُـَف حُ٘خطؾ ػٖ حُل‬
b. The fluid requirement as calculated by the Parkland ‫حثَ ًٔخ‬ُٞٔ‫ـذ اػطخء ٓظطِزخص ح‬٣ .b
formula should be administered over 12 hours. ٠ِ‫ـش رخًٍالٗي ػ‬٤ٜ‫رش ر‬ٞٔ‫ ٓل‬٢ٛ
c. Pre-hospital antibiotic should be started. .‫ ٓخػش‬12 ٍ‫ٓيح‬
d. Erythematous areas should be included in the total َ‫ش هز‬٣ٞ٤‫خىحص حُل‬٠ُٔ‫ـذ حُزيء رخ‬٣ .c
body surface area (TBSA) calculation. .٠‫ٍ حُٔٔظ٘ل‬ٞ‫ىه‬
٢‫ش ك‬٤ٓ‫ن حُلٔخ‬١‫ٖ حُٔ٘خ‬٤ٔ٠‫ـذ ط‬٣ .d
ْٔ‫ ٓٔخكش ٓطق حُـ‬٢ُ‫كٔخد اؿٔخ‬
Chez le patient souffrant de brûlures majeures, choisissez la bonne .)TBSA(
déclaration:
a. Il a été démontré que la décontamination sélective du tube
digestif réduit l‟incidence de l‟infection par brûlure.
b. Les besoins en liquides calculés selon la formule Parkland
doivent être administrés pendant 12 heures.
c. L‟antibiotique préhospitalier doit être commencé.
d. Les zones érythémateuses doivent être incluses dans le calcul
de la surface corporelle totale (TBSA)

4. All of the followings induce high AG metabolic acidosis except: ٖ٠ٔ‫ حٍطلخع حُظل‬٠ِ‫ ػ‬َٝ‫ل‬٣ ٢ِ٣ ‫ًَ ٓخ‬
:‫ ٓخ ػيح‬AG ٢‫حالٓظوالر‬
a. Lactic acidosis due to tissue hypoxia. ٚ‫ رٔزذ ٗو‬٢٘‫ٖ حُِز‬٠ٔ‫ حُظل‬.a
b. Ketoacidosis. .‫ حألٗٔـش‬٢‫حألًٔـش ك‬
c. Diarrhea. .٢ِٗٞ‫ٖ حُو‬٠ٔ‫ حُظل‬.b
d. Renal failure. .ٍ‫خ‬ٜٓ‫ حإل‬.c
١ٌُِٞ‫ حُلَ٘ ح‬.d

Tous les éléments suivants induisent une acidose métabolique AG


élevée sauf:
a. Acidose lactique due à l‟hypoxie tissulaire.
b. Acidocétose.
c. Diarrhée.
d. Insuffisance rénale

2
5. The most common cause of Chronic Kidney Disease is: ُِٖٓٔ‫ ح‬١ٌُِٞ‫ػخ ُِيحء ح‬ٞ٤ٗ َ‫حُٔزذ حألًؼ‬
:ٞٛ
a. Diabetes ١ٌَُٔ‫ ىحء ح‬.a
b. Hypertension ّ‫ حُي‬٢‫ـ‬ٟ ‫ حٍطلخع‬.b
c. History of AKI AKI ‫ن‬٣ٍ‫ طخ‬.c
d. Frequent NSAID use ‫خىحص‬٠ُٔ ٌٍَ‫ حالٓظويحّ حُٔظ‬.d
‫ش‬٣‫ي‬٤‫ث‬َٝ٤‫َ حُٔظ‬٤‫خد ؿ‬ٜ‫حالُظ‬

La cause la plus fréquente de maladie rénale chronique est:


a. Diabète
b. Hypertension
c. Histoire d‟AKI
d. Utilisation fréquente d‟AINS

6. Which is the most common form of acute glomerulonephritis? ‫زخص‬٤‫خد ًز‬ٜ‫ػخ الُظ‬ٞ٤ٗ َ‫ حٌَُ٘ حألًؼ‬ٞٛ ‫ٓخ‬
‫ حُلخى؟‬٠ٌُِ‫ح‬
a. Post streptococcal ‫خص‬٣‫خرش رخُؼوي‬ٛ‫ليع رؼي حإل‬٣ .a
‫خص‬٣‫ رؼي حُؼوي‬IgA ‫ش‬٤ٌُِ‫ حػظالٍ ح‬.b
b. IgA nephropathy
ٍٞ‫غ حُظط‬٣َٓ ٠ًِ ‫زخص‬٤‫خد ًز‬ٜ‫ حُظ‬.c
c. Rapidly progressive glomerulonephritis ٍٞ‫ى رخٓظ‬ٞ‫ ٓظالُٓش ؿ‬.d
d. Goodpasture‟s syndrome ٢‫ حُـ٘خث‬٠ٌُِ‫زخص ح‬٤‫خد ًز‬ٜ‫ حُظ‬.e
e. Membranous glomerulonephritis

Quelle est la forme la plus courante de glomérulonéphrite aiguë?


a. post-streptococcique
b. Néphropathie à IgA
c. Glomérulonéphrite rapidement progressive
d. Syndrome de Goodpasture
e. Glomérulonéphrite membraneuse

7. The cut off of the ejection fraction (EF) that decide to put the ‫غ‬ٟٝ ٍَ‫و‬٣ ١ٌُ‫) ح‬EF( ٢‫هطغ حٌَُٔ حُوٌك‬
diagnosis between heart failure with mid-range EF and heart failure ٢ٓٞ‫ ٓظ‬EF ‫ٍ حُوِذ ٓغ‬ٜٞ‫ٖ ه‬٤‫ ر‬ٚ٤‫حُظ٘و‬
with reduced EF is: :ٞٛ ٞ‫ ٓ٘ول‬EF ‫كَ٘ حُوِذ ٓغ‬ٝ ٟ‫حُٔي‬
%20 .a
a. 20% %30 .b
b. 30% %40 .c
c. 40% %65 .d
d. 65%

La coupure de la fraction d‟éjection (FE) qui décide de placer le


diagnostic entre l‟insuffisance cardiaque avec EF moyenne et
l‟insuffisance cardiaque avec EF réduite est:
a. 20%
b. 30%
c. 40%
d. 65%

3
8. Which of this medication is not a class I recommendation for ‫ش ٖٓ حُيٍؿش‬٤ٛٞ‫ٔض ط‬٤ُ ‫ش‬٣ٝ‫ حألى‬ٌٙٛ ٖٓ ١‫أ‬
treatment of heart failure (decrease mortality and hospitalization) ٍ‫َ ٓؼي‬٤ِ‫ٍ حُوِذ (طو‬ٜٞ‫ ُؼالؽ ه‬٠ُٝ‫حأل‬
:)‫حالٓظ٘لخء‬ٝ ‫خص‬٤‫ك‬ُٞ‫ح‬
a. Diuretics ٍٞ‫ ٓيٍحص حُز‬.a
b. Beta-blockers ‫ظخ‬٤‫ كخؿزخص ر‬.b
c. Valsartan-Sacubutril َ٣َ‫ط‬ٞ‫ر‬ًٞ‫ كخُٔخٍطخٕ ٓخ‬.c
d. SGLT2 inhibitors ( dapa- or empa-gliflozine) -‫ آزخ‬ٝ‫أ‬-‫ (ىحرخ‬SGLT2 ‫ ٓؼزطخص‬.d
e. Mineralocoticoides antagonists )ٖ٣ُِٞ‫ل‬٤ِ‫ؿ‬
‫ش‬٤ٗ‫يحص حُٔؼي‬٤‫ث‬ٌٞ٤‫ط‬ٌُٞ‫خىحص ح‬٠ٓ .e
Lequel de ces médicaments n‟est pas une recommandation de classe I
pour le traitement de l‟insuffisance cardiaque (diminution de la
mortalité et de l‟hospitalisation)

a. Diurétiques
b. Bêta-bloquants
c. Valsartan-Sacubutril
d. Inhibiteurs du SGLT2 ( dapa- ou empa-gliflozine)
e. Antagonistes des minéralocoticoïdes

9. All of the following are direct complications of short bowel syndrome ‫خػلخص حُٔزخَٗس‬٠ُٔ‫ ح‬٢ٛ ٢ِ٣ ‫ًَ ٓخ‬
EXCEPT: :‫َس ٓخ ػيح‬٤ٜ‫ُٔظالُٓش حألٓؼخء حُو‬
‫ حَُٔحٍس‬٢‫ٍ ك‬َٝ‫ٔظ‬٤ٌُُٞ‫خص ح‬٤ٜ‫ ك‬.a
a. Cholesterol gallstones ٢‫خٕ حُظخؿ‬٣َُ٘‫ ح‬َٝٓ .b
b. Coronary artery disease ١‫ حُٔؼي‬ٞٔ‫ اكَحُ حُل‬١َ‫ ك‬.c
c. Gastric acid hypersecretion ّٞ٤ُٔ‫خص أًٔخالص حٌُخ‬٤ٜ‫ ك‬.d
d. Renal calcium oxalate calculi ‫ش‬٣ٌُِٞ‫ح‬
e. Steatorrhea ٢٘ٛ‫خٍ حُي‬ٜٓ‫ حإل‬.e

Tous les éléments suivants sont des complications directes du


syndrome de l‟intestin court SAUF:
a. Calculs biliaires de cholestérol
b. Maladie coronarienne
c. Hypersécrétion d‟acide gastrique
d. Calculs rénaux d‟oxalate de calcium
e. Stéatorrhée

4
10. A 62-year-old female presented to the ER with two days‟ history of ٠ُ‫َص ا‬٠‫ ك‬،‫ ػخٓخ‬62 َٔ‫حَٓأس طزِؾ ٖٓ حُؼ‬
nausea and vomiting. CT scan of the abdomen with IV contrast did ٕ‫خ‬٤‫ٖ ٖٓ حُـؼ‬٤ٓٞ٣ ‫ن‬٣ٍ‫حٍة ٓغ طخ‬ٞ‫ؿَكش حُط‬
not reveal any abnormalities. Her blood test showed mild ٢‫َ حُٔوطؼ‬٣ٜٞ‫ٌ٘ق حُظ‬٣ ُْ .‫ء‬٢‫حُو‬ٝ
leukocytosis, slightly low bicarbonate levels. ABG was done and ١‫ ػٖ أ‬١‫ي‬٣ٍُٞ‫ٖ ح‬٣‫ٓذ ُِزطٖ ٓغ حُظزخ‬ٞ‫حُٔل‬
revealed that she is mildly acidotic. She is known to have diabetes ١ٞٓ‫ حُي‬ٚ‫َ حُلل‬ٜ‫ أظ‬.٢‫ؼ‬٤‫ز‬١ َ٤‫ء ؿ‬٢ٗ
mellitus type two. She is taking metformin 1000 mg twice per day, ،‫خء‬٠٤‫خص حُز‬٣ٌَُ‫ ػيى ح‬٢‫لش ك‬٤‫خىس هل‬٣ُ
sitagliptin 100 mg per day and Lantus 15 units daily at bedtime. ْ‫ ط‬.‫ال‬٤ِ‫ش ه‬٠‫ٗخص ٓ٘ول‬ٞ‫ٌَر‬٤‫خص ر‬٣ٞ‫ٓٔظ‬ٝ
Labs: Electrolytes: within normal range, Creatinine: 1.3 .‫ق‬٤‫ٖ هل‬٠ٔ‫خ طل‬ٜ٣‫ٖ إٔ ُي‬٤‫طز‬ٝ ABG َٔ‫ػ‬
mg/dl, Urine ketones test is negative. Lactic acid: 2.5 mmol/l ٖٓ ١ٌَُٔ‫خرش ريحء ح‬ٜٓ ‫خ‬ٜٗ‫ف أ‬َٝ‫ٖٓ حُٔؼ‬
(normal: 0.5-1 mmol/L), Anion gap: 16. 1000 ٖ٤ٍٓٞ‫ظل‬٤ُٔ‫ٍ ح‬ٝ‫خ طظ٘خ‬ٜٗ‫ ا‬.٢ٗ‫ حُؼخ‬٢ُٔ٘‫ح‬
ْ‫ ٓـ‬100 ٖ٤‫زظ‬٤ِ‫ظخؿ‬٤ٓٝ ،‫خ‬٤ٓٞ٣ ٖ٤‫ٓـْ َٓط‬
What is the most likely cause of her lactic acidosis? ‫هض‬ٝ ٢‫خ ك‬٤ٓٞ٣ ‫كيس‬ٝ 15 ّٞ‫الٗظ‬ٝ ‫خ‬٤ٓٞ٣
.ُّٞ٘‫ح‬
a. Intestinal ischemia not seen on CT scan
ٍ‫ٖٔ حُٔؼي‬ٟ :‫حٍى‬ُٞ٘‫ ح‬:١َ‫ حُٔوز‬ٚ‫حُلل‬
b. Metformin
،َ‫ِظ‬٤ٔ٣‫ ى‬/ ْ‫ ٓـ‬1.3 :ٖ٤٘٤‫خط‬٣ٌَُ‫ ح‬،٢‫ؼ‬٤‫حُطز‬
c. Sitagliptin
:ٖ‫ حُِز‬ٞٔ‫ ك‬.٢‫ٍ ِٓز‬ٞ‫ٗخص حُز‬ٞ‫ظ‬٤ً ٍ‫حهظزخ‬
d. Underlying sepsis
/ ٍٞٔ٤ِٓ 1-0.5 :٢‫ؼ‬٤‫ز‬١( َ‫ ُظ‬/ ٍٞٔ٤ِٓ 2.5
e. Insulin
.16 :ٕٞ٤ٗ‫س حأل‬ٞ‫ كـ‬،)َ‫ُظ‬
٢٘‫ٖ حُِز‬٠ٔ‫لخ ُِظل‬٤‫ حُٔزذ حألًؼَ طَؿ‬ٞٛ ‫ٓخ‬
Une femme de 62 ans s‟est présentée à l‟urgence avec des antécédents
‫خ؟‬ٛ‫ػ٘ي‬
de nausées et de vomissements depuis deux jours. La
‫ش‬٤‫َ حَُٔث‬٤‫ش ؿ‬٣ٞ‫ش حُٔؼ‬٣َٝ‫ حُظ‬ٚ‫ ٗو‬.a
tomodensitométrie de l‟abdomen avec contraste IV n‟a révélé aucune
‫ش‬٤‫ حألٗؼش حُٔوطؼ‬٢‫ك‬
anomalie. Son test sanguin a montré une leucocytose légère, des
ٖ٤ٍٓٞ‫ظل‬٤ٓ .b
niveaux de bicarbonate légèrement bas. ABG a été fait et a révélé
ٖ٤‫زظ‬٤ِ‫ظخؿ‬٤ٓ .c
qu‟elle est légèrement acidotique. Elle est connue pour avoir le
ٖٓ‫ اٗظخٕ ًخ‬.d
diabète sucré de type deux. Elle prend 1000 mg de metformine deux
ٖ٤ُٞٔٗ‫ حأل‬.e
fois par jour, de la sitagliptine 100 mg par jour et 15 unités de Lantus
par jour au coucher. Laboratoires: Électrolytes: dans la plage normale,
Créatinine: 1,3 mg / dl, Le test des cétones urinaires est négatif. Acide
lactique : 2,5 mmol/l (normal : 0,5-1 mmol/L), Anion gap : 16.
Quelle est la cause la plus probable de son acidose lactique?
a. Ischémie intestinale non observée sur la tomodensitométrie
b. Metformine
c. Sitagliptine
d. Septicémie sous-jacente
e. Insuline

5
11. A 56-year-old woman with diabetes type 2 is having elevated ‫خرش ريحء‬ٜٓ ‫ ػخٓخ‬56 َٔ‫حَٓأس طزِؾ ٖٓ حُؼ‬
morning glucose of 180-200 mg/dl. She is on long-acting insulin 20 ‫ ٖٓ حٍطلخع ٗٔزش‬٢ٗ‫ طؼخ‬،2 ٢ُٔ٘‫ ٖٓ ح‬١ٌَُٔ‫ح‬
units at bedtime only. During the day glucose is 90-140. Her Hb A1C / ْ‫ ٓـ‬200-180 ٖٓ ‫زخف‬ُٜ‫ ح‬٢‫ُ ك‬ًِٞٞ‫حُـ‬
is 6.6 %. What is your next step? ٖ٤ُٞٔٗ‫كيس ٖٓ أ‬ٝ 20 ٍٝ‫خ طظ٘خ‬ٜٗ‫ ا‬.َ‫ِظ‬٤ٔ٣‫ى‬
a. Increase her insulin dose ٍ‫خ‬ُٜ٘‫ هالٍ ح‬.٢‫ّ كو‬ُٞ٘‫َ حألٓي ػ٘ي ح‬٣ٞ١
b. Inquire about her eating habits and time of her ٞٛ HbA1C .140-90 ٞٛ ًُِٞٞ‫حُـ‬
meals ‫ش؟‬٤ُ‫طي حُظخ‬ٞ‫ هط‬٢ٛ ‫ ٓخ‬.6.6%
c. Add metformin 1000 mg before she goes to bed ٖ٤ُٞٔٗ‫خىس ؿَػش حأل‬٣ُ .a
d. Add short acting insulin in the morning ‫ش‬٤‫خ حُـٌحث‬ٜ‫ حالٓظلٔخٍ ػٖ ػخىحط‬.b
‫خ‬ٜ‫ؿزخط‬ٝ ‫ي‬٤‫حػ‬ٞٓٝ
َ‫ ٓـْ هز‬1000 ٖ٤ٍٓٞ‫ظل‬٤ٓ ‫خكش‬ٟ‫ ا‬.c
Une femme de 56 ans atteinte de diabète de type 2 a une glycémie ٕ‫ حُلَح‬٠ُ‫خد ا‬ٌُٛ‫ح‬
matinale élevée de 180 à 200 mg / dl. Elle prend de l‟insuline à action ٢‫ ك‬ٟ‫َ حُٔي‬٤ٜ‫ٖ ه‬٤ُٞٔٗ‫خكش أ‬ٟ‫ ا‬.d
prolongée 20 unités au coucher seulement. Pendant la journée, le ‫زخف‬ُٜ‫ح‬
glucose est de 90 à 140. Son taux d‟Hb A1C est de 6,6 %. Quelle est
votre prochaine étape?

a. Augmenter sa dose d‟insuline


b. Renseignez-vous sur ses habitudes alimentaires et l‟heure de
ses repas
c. Ajouter de la metformine 1000 mg avant d‟aller au lit
d. Ajouter de l‟insuline à courte durée d‟action le matin

6
12. A 36-year-old nurse is brought to ER by her husband due to an ‫ ػخٓخ‬36 َٔ‫ش طزِؾ ٖٓ حُؼ‬َٟٔٓ ٍ‫خ‬٠‫ظْ اك‬٣
episode of hypoglycemia. This happened 3 times before, during which ‫خ رٔزذ‬ٜ‫ؿ‬ُٝ َ‫حٍة ٖٓ هز‬ٞ‫ ؿَكش حُط‬٠ُ‫ا‬
she feels confused. Her husband is diabetic, he checked her glucose 3 ‫ٌح‬ٛ ‫ كيع‬.ّ‫ حُي‬٢‫ حٌَُٔ ك‬ٚ‫رش ٖٓ ٗو‬ٞٗ
and it was in the 50‟s during these episodes. She is not known to have .‫خ رخالٍطزخى‬ُٜ‫ ٗؼَص هال‬،َ‫َٓحص ٖٓ هز‬
any medical problem before. Her husband today gave her juice before ٚ‫ هخّ رلل‬،١ٌَُٔ‫خد ريحء ح‬ٜٓ ‫خ‬ٜ‫ؿ‬ُٝ
bringing her to ER. In the ER her glucose is 105 mg/dl. What would ٍ‫ هال‬50 ٍ‫ ٓـخ‬٢‫ًخٕ ك‬ٝ ‫خ‬ٜ٣‫ُ ُي‬ًِٞٞ‫حُـ‬
you recommend to this patient? ٢ٗ‫خ طؼخ‬ٜٗ‫ف أ‬َٝ‫َ حُٔؼ‬٤‫ ٖٓ ؿ‬.‫رخص‬ُٞ٘‫ ح‬ٌٙٛ
‫خ‬ٜ‫ؿ‬ُٝ ‫خ‬ٛ‫ أػطخ‬.َ‫ش ٖٓ هز‬٤‫ز‬١ ‫ ٌِٓ٘ش‬١‫ٖٓ أ‬
a. Admit to hospital for further work up
‫ ؿَكش‬٠ُ‫خ ا‬ٍٛ‫خ‬٠‫َ هزَ اك‬٤ٜ‫ّ حُؼ‬ٞ٤ُ‫ح‬
b. Check her insulin, proinsulin, c- peptide
ٕ‫ ًخ‬،‫حٍة‬ٞ‫ ؿَكش حُط‬٢‫ ك‬.‫حٍة‬ٞ‫حُط‬
c. Check TSH and cortisol level
‫ق‬ٜ٘‫ رٔخًح ط‬.َ‫ِظ‬٤ٔ٣‫ ى‬/ ْ‫ ٓـ‬105 ًُِٞٞ‫حُـ‬
d. Ask her to keep sugary drink with her all the
‫ش؟‬٠٣َُٔ‫ ح‬ٌٙٛ
time and use when needed.
‫ي ٖٓ حُزلغ‬٣ُِٔ ٠‫خ حُٔٔظ٘ل‬ُٜ‫ اىهخ‬.a
‫ؼش‬٤ِ١ ،ٖ٤ُٞٔٗ‫ حُظلون ٖٓ حأل‬.b
Une infirmière de 36 ans est amenée aux urgences par son mari en
‫ؽ‬- ‫ي‬٤‫ حُززظ‬،ٖ٤ُٞٔٗ‫حأل‬
raison d‟un épisode d‟hypoglycémie. Cela s‟est produit 3 fois
TSH ٟٞ‫ حُظلون ٖٓ ٓٔظ‬.c
auparavant, au cours desquelles elle se sent confuse. Son mari est
ٍِٝ٤‫ٍط‬ٌُٞ‫ح‬ٝ
diabétique, il a vérifié sa glycémie et c‟était dans les années 50 lors de
‫د‬َٝ٘ٔ‫خ حالكظلخظ ر‬ٜ٘ٓ ‫ حُطِذ‬.d
ces épisodes. Elle n‟est pas connue pour avoir eu un problème
‫هض‬ُٞ‫حٍ ح‬ٞ١ ‫خ‬ٜ‫ ٓؼ‬١ٌَٓ
médical auparavant. Son mari lui a donné aujourd‟hui du jus avant de
.‫ ػ٘ي حُلخؿش‬ٚٓ‫حٓظويح‬ٝ
l‟amener aux urgences. Aux urgences, sa glycémie est de 105 mg/dl.
Que recommanderiez-vous à ce patient?

a. Admettre à l‟hôpital pour un examen plus approfondi


b. Vérifiez son insuline, sa proinsuline, son peptide c
c. Vérifiez le taux de TSH et de cortisol
d. Demandez-lui de garder la boisson sucrée avec elle tout le
temps et de l‟utiliser au besoin

7
13. Which of the following statements is FALSE about atrial fibrillation ٖ‫لش ػ‬٤‫ل‬ٛ َ٤‫ش ؿ‬٤ُ‫ ٖٓ حُؼزخٍحص حُظخ‬١‫أ‬
(Afib)? ‫)؟‬Afib( ٢٘٣ً‫حَُؿلخٕ حأل‬
a. It is characterized by irregular rhythm with no P wave on ّ‫َ ٓ٘ظظْ ٓغ ػي‬٤‫وخع ؿ‬٣‫ِ رب‬٤ٔ‫ظ‬٣ .a
the surface ECG. .ECG ‫ ٓطق‬٠ِ‫ ػ‬P ‫ؿش‬ٞٓ ‫ى‬ٞ‫ؿ‬ٝ
b. It is associated with increased risk of ischemic stroke. ‫خرش‬ٛ‫خىس هطَ حإل‬٣ُ ‫ظَحكن ٓغ‬٣ .b
c. Only patients with heart failure and atrial fibrillation need .‫ش‬٤‫رخٌُٔظش حُيٓخؿ‬
anticoagulation, otherwise no indication for ٖٓ ٕٞٗ‫ؼخ‬٣ ٖ٣ٌُ‫ ح‬٠َُٟٔ‫ ح‬٢‫ كو‬.c
anticoagulation. ٢٘٣ً‫حَُؿلخٕ حأل‬ٝ ‫ٍ حُوِذ‬ٜٞ‫ه‬
d. Treatment modalities may include pharmacological rate ،َ‫خىحص حُظوؼ‬٠ٓ ٠ُ‫ٕ ا‬ٞ‫لظخؿ‬٣
control, cardioversion and ablation depends on the patient ‫ ٓ٘غ‬٠ِ‫ؿي ٓئَٗ ػ‬ٞ٣ ‫اال ال‬ٝ
profile. .َ‫حُظوؼ‬
٢‫َم حُؼالؽ حُظلٌْ ك‬١ َٔ٘‫ هي ط‬.d
‫ْ ٗظْ حُوِذ‬٣ٞ‫ طو‬، ٢‫حث‬ٝ‫حُٔؼيٍ حُي‬
Laquelle des affirmations suivantes est FAUX au sujet de la ‫ ِٓق‬٠ِ‫ؼظٔي ػ‬٣ ٍ‫خ‬ٜ‫حالٓظج‬ٝ
fibrillation auriculaire (fibrillation auriculaire)? .ٞ٣َُٔ‫ق ح‬٣َ‫طؼ‬

a. Elle se caractérise par un rythme irrégulier sans onde P sur


l‟ECG de surface.
b. Elle est associée à un risque accru d‟AVC ischémique.
c. Seuls les patients atteints d‟insuffisance cardiaque et de
fibrillation auriculaire ont besoin d‟anticoagulation, sinon
aucune indication d‟anticoagulation.
d. Les modalités de traitement peuvent inclure le contrôle
pharmacologique, la cardioversion et l‟ablation en fonction du
profil du patient.

8
14. A 58-year-old man presents with pain in his legs and a difficulty in ُْ‫ ٖٓ أ‬٢ٗ‫ؼخ‬٣ ‫ ػخٓخ‬58 َٔ‫زِؾ ٖٓ حُؼ‬٣ َ‫ٍؿ‬
walking. He has no co-morbidities and denies smoking, alcohol and ٚ٣‫ْ ُي‬٤ُ .٢ُ٘ٔ‫ ح‬٢‫رش ك‬ٞ‫ؼ‬ٛٝ ٚ٤‫ ٓخه‬٢‫ك‬
drug use. He has had a history of same sex partners and has been a ٍٞ‫حٌُل‬ٝ ٖ٤‫ٌَ٘ حُظيه‬٣ٝ ‫ ٓ٘ظًَش‬ٝ‫أَٓح‬
frequent attender to GUM clinics. On examination you find that he ٖٓ ‫ن‬٣ٍ‫ طخ‬ٚ٣‫ ًخٕ ُي‬.‫ حُٔويٍحص‬٢١‫طؼخ‬ٝ
has a loss of proprioception and reflexes in the lower limbs along with ‫َح‬ٟ‫ًخٕ كخ‬ٝ ْ٘‫حًَُ٘خء ٖٓ ٗلْ حُـ‬
a pupil that only responds to accommodation. His initial blood tests ٢ُٞ‫خىحص حُطذ حُز‬٤‫ ػ‬٢‫رٌَ٘ ٓظٌٍَ ك‬
are unremarkable with normal B12 levels. What is the correct ٖٓ ٢ٗ‫ؼخ‬٣ ٚٗ‫ طـي أ‬ٚ‫ ػ٘ي حُلل‬.٢ِٓ‫حُظ٘خ‬
diagnosis? ٢‫ى حُلؼَ ك‬ٝ‫ٍى‬ٝ ‫ن‬٤ٔ‫كويحٕ حُلْ حُؼ‬
a. Diabetic neuropathy ‫ ؿ٘ذ ٓغ كيهش‬٠ُ‫ش ؿ٘زخ ا‬٤ِ‫َحف حُٔل‬١‫حأل‬
b. Guillian-Barre syndrome ‫ش‬٤ُٝ‫ حأل‬ٚٓ‫ حهظزخٍحص ى‬.‫ق‬٤ٌ‫ ُِظ‬٢‫ذ كو‬٤‫طٔظـ‬
c. Lead poisoning .‫ش‬٤‫ؼ‬٤‫ حُطز‬B12 ‫خص‬٣ٞ‫ظش ٓغ ٓٔظ‬ٞ‫َ ِٓل‬٤‫ؿ‬
d. Sub-acute degeneration of the cord ‫ق؟‬٤‫ل‬ُٜ‫ ح‬ٚ٤‫ حُظ٘و‬ٞٛ ‫ٓخ‬
e. Tertiary syphilis ١ٌَٓ ٢‫ز‬ٜ‫ حػظالٍ ػ‬.a
ٚ٣ٍ‫خٕ رخ‬٤ِ٤‫ ٓظالُٓش ؿ‬.b
Un homme de 58 ans présente des douleurs aux jambes et des ٙ‫خ‬َُٛ‫ حُظْٔٔ رخ‬.c
difficultés à marcher. Il n‟a pas d‟autres pathologies et nie avoir ‫ طٌْ٘ حُلزَ طلض حُلخى‬.d
consommé du tabac, de l‟alcool et des drogues. Il a des antécédents de ٢‫ ػخُؼ‬١َُٛ .e
partenaires de même sexe et a fréquenté fréquemment les cliniques
GUM. À l‟examen, vous constatez qu‟il a une perte de proprioception
et de réflexes dans les membres inférieurs ainsi qu‟une pupille qui ne
répond qu‟à l‟accommodation. Ses premiers tests sanguins sont banals
avec des niveaux normaux de B12. Quel est le bon diagnostic?
a. Neuropathie diabétique
b. Syndrome de Guillian-Barré
c. Saturnisme
d. Dégénérescence subaiguë du cordon ombilical
e. Syphilis tertiaire

9
15. A 27-year-old girl presents to her GP with severe cramping ٠ُ‫َ ا‬٠‫ ػخٓخ طل‬27 َٔ‫كظخس طزِؾ ٖٓ حُؼ‬
abdominal pain and diarrhoea for the past 2 weeks. She had a similar ٖ‫ حُزط‬٢‫يس ك‬٣‫ش ٗي‬٤‫خ حُؼخّ ٓغ آالّ ط٘٘ـ‬ٜ‫ز‬٤‫ز‬١
episode 8 weeks earlier, which resolved without medical attention. At .ٖ٤٤ٟ‫ٖ حُٔخ‬٤‫ػ‬ٞ‫ حألٓز‬ٟ‫ ٓي‬٠ِ‫خٍ ػ‬ٜٓ‫حإل‬ٝ
present the diarrhoea is bloody and bowel motions are associated with ٢‫حُظ‬ٝ ،‫غ‬٤‫ أٓخر‬8 َ‫رش ٓٔخػِش هز‬ٞٗ ‫خ‬ٜ٣‫ًخٗض ُي‬
crampy abdominal pains. In addition the GP notes oral ulcers, which ‫هض‬ُٞ‫ ح‬٢‫ ك‬.‫ش‬٤‫ز‬١ ‫ٕ ٓيحهِش‬ٝ‫خ ى‬ِٜ‫طْ ك‬
are painful and according to the patient have been present for the past ‫ كًَخص‬٢‫طَطز‬ٝ ١ٞٓ‫خٍ ى‬ٜٓ‫َ حإل‬ٟ‫حُلخ‬
3 days. Her family history includes a grandfather who had a ٠ُ‫خكش ا‬ٟ‫ رخإل‬.‫ش‬٤‫حألٓؼخء رآالّ حُزطٖ حُظ٘٘ـ‬
colostomy for a gastro intestinal (GI) problem. What is the most ‫ذ طوَكخص حُلْ حُٔئُٔش‬٤‫الكع حُطز‬٣ ،‫ًُي‬
likely cause for her symptoms and signs? .ّ‫خ‬٣‫ أ‬3 ٌ٘ٓ ‫ىس‬ٞ‫ؿ‬ٞٓ ‫ ًخٗض‬ٞ٣َُِٔ ‫كوخ‬ٝٝ
a. Crohn‟s disease. َ‫ كـ‬ٚ٣‫خ ًخٕ ُي‬ٛ‫خ إٔ ؿي‬ٜ‫ن ػخثِظ‬٣ٍ‫ٖٔ طخ‬٠‫ظ‬٣
b. HenochŔSchönlein purpura. ‫ ٓخ‬.)GI( ٢ٔ٠ُٜ‫خُ ح‬ٜ‫ حُـ‬٢‫ٕ ٌُِٔ٘ش ك‬ُٞٞ‫ه‬
c. Shigella infection. ‫خ؟‬ٜ‫ػالٓخط‬ٝ ‫خ‬ٜٟ‫لخ ألػَح‬٤‫حُٔزذ حألًؼَ طَؿ‬
d. Giardia infection. .ًَٕٝ ‫ ىحء‬.a
.ٖ٤ِٗٞٗ ‫م‬ٞ٘٤ٛ ‫ش‬٣َ‫ كَك‬.b
Une jeune fille de 27 ans se présente à son médecin généraliste avec .‫ال‬٤‫ـ‬٤ُ٘‫خرش رخ‬ٛ‫ حإل‬.c
de fortes crampes abdominales et de la diarrhée depuis 2 semaines. .‫خ‬٣‫خٍى‬٤‫خرش رخُـ‬ٛ‫ حإل‬.d
Elle a eu un épisode similaire 8 semaines plus tôt, qui s‟est résolu
sans soins médicaux. À l‟heure actuelle, la diarrhée est sanglante et
les selles sont associées à des crampes abdominales. En outre, le
médecin généraliste note des ulcères buccaux, qui sont douloureux et
selon le patient sont présents depuis 3 jours. Ses antécédents
familiaux comprennent un grand-père qui a subi une colostomie pour
un problème gastro-intestinal (GI). Quelle est la cause la plus
probable de ses symptômes et signes?
a. Maladie de Crohn.
b. Purpura de Henoch-Schönlein.
c. Infection à Shigella.
d. Infection à Giardia.

16. Hematuria, hypertension, protenuria and red cell casts in the urine are ‫ش‬٤٘٤‫ط‬َٝ‫ِش ر‬٤‫ ر‬،ّ‫ حُي‬٢‫ـ‬ٟ ‫ حٍطلخع‬،‫ش‬٣ٞٓ‫ِش ى‬٤‫ر‬
indicative of? ٍ‫ٍ طي‬ٞ‫ حُز‬٢‫خص كَٔحء ك‬٣ًَ ‫حٗخص‬ٞ‫حٓط‬ٝ
a. hepatorenal syndrome ‫؟‬٠ِ‫ػ‬
‫ش‬٣ٌُِٞ‫ش ح‬٣‫ حُٔظالُٓش حٌُزي‬.a
b. nephritic conditions
‫ش‬٣ٌُِٞ‫ حُلخالص ح‬.b
c. rhabdomyolysis ٢ِ٠‫ طلَِ ػ‬.c
d. amanita phylloides poisoning ‫ي‬٣ِٞ٤‫ظخ ك‬٤ٗ‫ حُظْٔٔ رلطَ أٓخ‬.d

L‟hématurie, l‟hypertension, la protenurie et les globules rouges dans


l‟urine sont indicatives?
a. syndrome hépato-rénal
b. états néphritiques
c. rhabdomyolyse
d. amanita phylloïdes empoisonnement

10
17. Which of the following patients with pyelonephritis can be safely ٖ٤‫خر‬ُٜٔ‫ْ ح‬ًًَٛ ٢ُ‫ حُظخ‬٠َُٟٔ‫ ٖٓ ح‬١‫أ‬
treated as an outpatient? ٕ‫ رؤٓخ‬ٚ‫ٌٖٔ ػالؿ‬٣ ‫ش‬٤ٌُِ‫ح‬ٝ ‫ش‬٠٣ٞ‫خد حُل‬ٜ‫رخُظ‬
a. A 75-year-old diabetic female ‫؟‬٢‫ هخٍؿ‬ٞ٣ًَٔ
‫ ػخٓخ‬75 َٔ‫ حَٓأس طزِؾ ٖٓ حُؼ‬.a
b. A 20-year-old 20-week pregnant female with mild abdominal
١ٌَُٔ‫خرش ريحء ح‬ٜٓ
cramping ‫ ػخٓخ‬20 َٔ‫ حَٓأس طزِؾ ٖٓ حُؼ‬.b
c. A 30-year-old female with persistent vomiting and fever ٖٓ ٢ٗ‫ طؼخ‬20 ‫ع‬ٞ‫ حألٓز‬٢‫كخَٓ ك‬
d. A 33-year-old male with renal calculi ٖ‫ حُزط‬٢‫لش ك‬٤‫خص هل‬ِٜ‫طو‬
e. None of the above can be safely treated as outpatients ٢ٗ‫ ػخٓخ طؼخ‬30 َٔ‫ طزِؾ ٖٓ حُؼ‬٠‫ أٗؼ‬.c
٠ٔ‫حُل‬ٝ َٔ‫ء حُٔٔظ‬٢‫ٖٓ حُو‬
Lequel des patients suivants atteints de pyélonéphrite peut être traité ‫ ػخٓخ‬33 َٔ‫زِؾ ٖٓ حُؼ‬٣ َ‫ ٍؿ‬.d
‫ش‬٣ٌُِٞ‫خص ح‬٤ٜ‫خد رخُل‬ٜٓ
en toute sécurité en ambulatoire?
ٕ‫ ٓٔخ ٓزن رؤٓخ‬١‫ٌٖٔ ػالؽ أ‬٣ ‫ ال‬.e
a. Une femme diabétique de 75 ans ٖ٤٤‫ هخٍؿ‬٠ًَٟٔ
b. Une femme enceinte de 20 semaines de 20 semaines
souffrant de légères crampes abdominales
c. Une femme de 30 ans souffrant de vomissements persistants
et de fièvre
d. Un homme de 33 ans souffrant de calculs rénaux
e. Aucun des éléments ci-dessus ne peut être traité en toute
sécurité en ambulatoire

18. Osteoporosis, choose the correct answer. :‫لش‬٤‫ل‬ُٜ‫ حهظَ حإلؿخرش ح‬،ّ‫٘خٗش حُؼظخ‬ٛ
a. Osteoporosis is the decrease in bone turnover due to increased ٢‫ ك‬ٝ‫ حٗولخ‬٢ٛ ّ‫٘خٗش حُؼظخ‬ٛ .a
osteoblastic activity ‫خىس‬٣ُ ‫ٍ حُؼظخّ رٔزذ‬ٞ‫ٓؼيٍ حُظل‬
b. Is a painless disease, unless a fracture occurs ّ‫خص حُؼظخ‬٤ٗ‫ رخ‬٢‫ ك‬١‫حُ٘٘خ‬
c. Male hypogonadism is a risk factor for osteomalacia, not ‫ليع‬٣ ُْ ‫ ٓخ‬،ُْ‫َ ٓئ‬٤‫ ؿ‬َٝٓ ٞٛ .b
osteoporosis ًَٔ
d. Calcium and vitamin D supplementation should not be given ًٌٍُٞ‫ش ػ٘ي ح‬٤ِٓ‫ٍ حُـيى حُظ٘خ‬ٜٞ‫ ه‬.c
before menopause ،ّ‫ٖ حُؼظخ‬٤ِ‫ ػخَٓ هطَ ُظ‬ٞٛ
ّ‫٘خٗش حُؼظخ‬ٛ ْ٤ُٝ
ّٞ٤ُٔ‫ اػطخء ٓظٔٔخص حٌُخ‬٢‫٘زـ‬٣ ‫ ال‬.d
Ostéoporose, choisissez la bonne réponse. ‫ٖ ى هزَ حٗوطخع حُطٔغ‬٤ٓ‫ظخ‬٤‫ك‬ٝ
a. L‟ostéoporose est la diminution du renouvellement osseux due
à une activité ostéoblastique accrue
b. Est une maladie indolore, sauf si une fracture se produit
c. L‟hypogonadisme masculin est un facteur de risque
d‟ostéomalacie et non d‟ostéoporose
d. La supplémentation en calcium et en vitamine D ne doit pas
être administrée avant la menopause

11
19. All are causes of fulminant hepatic failure except one: ١‫ ٖٓ أٓزخد حُلَ٘ حٌُزي‬ٞٛ ٢ِ٣ ‫ًَ ٓخ‬
a. Acute alcoholic hepatitis :‫ق ٓخػيح‬١‫حُوخ‬
b. Wilson‟s disease ‫ حُلخى‬٢ُٞ‫خد حٌُزي حٌُل‬ٜ‫ حُظ‬.a
c. Acute fatty liver of pregnancy ِٕٞٔ٣ٝ ‫ ىحء‬.b
َٔ‫ حُلخى رٔزذ حُل‬٢٘ٛ‫ حٌُزي حُي‬.c
d. Viral hepatis B or C or D
ٝ‫ أ‬C ٝ‫ أ‬B ٢َٓٝ٤‫خد حٌُزي حُل‬ٜ‫ حُظ‬.d
D
Tous sont des causes d‟insuffisance hépatique fulminante sauf une:
a. Hépatite alcoolique aiguë
b. Maladie de Wilson
c. Stéatose hépatique aiguë de la grossesse
d. Hépatis virale B ou C ou D

20. Which one of the following does not affects the choice of ‫خىحص‬٠ُٔ‫خٍ ح‬٤‫ حهظ‬٠ِ‫ئػَ ػ‬٣ ‫ ال‬٢ِ٣ ‫ ٓٔخ‬١‫أ‬
prophylactic antibiotic? ‫ش؟‬٤‫هخث‬ُٞ‫ش ح‬٣ٞ٤‫حُل‬
a- The expected spectrum of organisms likely to be ‫خص‬٣ٞ٠‫هغ ُِؼ‬ٞ‫ق حُٔظ‬٤‫ حُط‬.a
encountered ‫خ‬ٜ‫ظ‬ٜ‫حؿ‬ٞٓ ‫حَُٔؿق‬
b- The cost ‫ حُظٌِلش‬.b
c- The personal preference ٢ٜ‫َ حُ٘و‬٤٠‫ حُظل‬.c
d- The hospital policies ٠‫خٓش حُٔٔظ٘ل‬٤ٓ .d
e- The local resistance strains ‫ٓش‬ٝ‫ حُٔالالص حُٔوخ‬.e

Lequel des éléments suivants n‟affecte pas le choix de l‟antibiotique


prophylactique?
a. Le spectre attendu des organismes susceptibles d‟être
rencontrés
b. Le coût
c. La préférence personnelle
d. Les politiques hospitalières
e. Les souches de résistance locales

12
21. Which one of the following may NOT require more than one dose of ‫ظطِذ أًؼَ ٖٓ ؿَػش‬٣ ‫ هي ال‬٢ِ٣ ‫ ٓٔخ‬١‫أ‬
prophylactic antibiotic? ‫ش؟‬٤‫هخث‬ُٞ‫ش ح‬٣ٞ٤‫خىحص حُل‬٠ُٔ‫حكيس ٖٓ ح‬ٝ
a- Prolonged operations ‫ِش حألٓي‬٣ٞ١ ‫ش‬٤‫خص حُـَحك‬٤ِٔ‫ حُؼ‬.a
b- Excessive blood loss ١َ‫ كويحٕ حُيّ حُٔل‬.b
c- Gastrointestinal surgery ٢ٔ٠ُٜ‫خُ ح‬ٜ‫ ؿَحكش حُـ‬.c
d- Insertion of prosthesis ٢‫ط٘خػ‬ٛ‫َف ح‬١ ٍ‫ اىهخ‬.d
e- Unexpected contamination ‫هغ‬ٞ‫َ ٓظ‬٤‫ع ؿ‬ِٞ‫ ط‬.e

Lequel des éléments suivants peut ne pas nécessiter plus d‟une dose
d‟antibiotique prophylactique?
a. Opérations prolongées
b. Perte de sang excessive
c. Chirurgie gastro-intestinale
d. Insertion de prothèse
e. Contamination inattendue

22. Which of the following is NOT a natural barrier to infection? ‫؟‬ٟٝ‫خ ُِؼي‬٤‫ؼ‬٤‫ز‬١ ‫ْ كخؿِح‬٤ُ ٢ِ٣ ‫ ٓٔخ‬١‫أ‬
a- Intact epithelial surface ْ٤ِٓ ١ٍ‫خ‬ٜ‫ ٓطق ظ‬.a
b- Antibodies ‫خىس‬٠ُٔ‫ حألؿٔخّ ح‬.b
c- Antibiotics ‫ش‬٣ٞ٤‫خىحص حُل‬٠ُٔ‫ ح‬.c
d- Macrophages ‫َس‬٤‫ حُزخُؼخص حٌُز‬.d

Lequel des éléments suivants n‟est PAS une barrière naturelle à


l‟infection?
a. Surface épithéliale intacte
b. Anticorps
c. Antibiotiques
d. Macrophages
23. Which of the following is NOT a cause of reduced host resistance to ‫ٓش‬ٝ‫ ٓوخ‬ٝ‫ْ ٓززخ الٗولخ‬٤ُ ٢ُ‫ ٖٓ حُظخ‬١‫أ‬
infection? ‫؟‬ٟٝ‫ق ُِؼي‬٤٠ُٔ‫ح‬
a- Malnutrition ‫ش‬٣ٌ‫ء حُظـ‬ٞٓ .a
b- Heart failure ‫ٍ حُوِذ‬ٜٞ‫ ه‬.b
c- Cancer ٕ‫خ‬١َُٔ‫ ح‬.c
d- AIDS ُ‫ي‬٣‫ حإل‬.d

Lequel des éléments suivants n‟est PAS une cause de résistance


réduite de l‟hôte à l‟infection?
a. Malnutrition
b. Insuffisance cardiaque
c. Cancer
d. SIDA

13
24. Which of the following is NOT a risk factor for wound infection? ٕ‫خرش رخٗظخ‬ٛ‫ْ ػخَٓ هطَ ُإل‬٤ُ ٢ِ٣ ‫ ٓٔخ‬١‫أ‬
a- Poor perfusion ‫ حُـَف؟‬٢‫ك‬
b- Use of skin clips for wound closure ‫ش‬٣َٝ‫ؼق حُظ‬ٟ .a
c- Poor surgical technique ‫ حٓظويحّ ٓ٘خري حُـِي إلؿالم‬.b
d- Uremia ‫حُـَف‬
‫جش‬٤ٓ ‫ش‬٤‫ش ؿَحك‬٤٘‫ طو‬.c
Lequel des éléments suivants n‟est PAS un facteur de risque ‫ش‬٣ٞٓ‫ِش ى‬٤‫ ر‬.d
d‟infection de la plaie?
a. Mauvaise perfusion
b. Utilisation de clips cutanés pour la fermeture de la plaie
c. Mauvaise technique chirurgicale
d. Urémie

25. In primary hyperparathyroidism, localization of the abnormal gland is ٖٓ ،٢ُٝ‫ ؿخٍحص حُيٍم حأل‬١‫ ٗ٘خ‬١َ‫ ك‬٢‫ك‬
best done by: ‫ش‬٤‫ؼ‬٤‫َ حُطز‬٤‫غ حُـيس ؿ‬ٟٞٔ‫َ ٓؼَكش ط‬٠‫حألك‬
a- Ultrasound :‫ن‬٣َ١ ٖ‫ػ‬
b- Tl-Tc scintigraphy ‫ش‬٤‫ط‬ُٜٞ‫م ح‬ٞ‫ؿخص ك‬ُٞٔ‫ ح‬.a
c- MIBI scan Tl-Tc ٢ٗ‫خ‬٠ٓٝ َ٣ٜٞ‫ ط‬.b
d- Ultrasound and scintigraphy at MIBI MIBI ‫ ٓٔق‬.c
e- MRI َ٣ٜٞ‫حُظ‬ٝ ‫ش‬٤‫ط‬ُٜٞ‫م ح‬ٞ‫ؿخص ك‬ُٞٔ‫ ح‬.d
MIBI ٢‫ ك‬٢ٗ‫خ‬٠ُٓٞ‫ح‬
Dans l‟hyperparathyroïdie primaire, la localisation de la glande ٢ٔ٤١‫ٖ حُٔـ٘خ‬٤َُٗ‫َ رخ‬٣ٜٞ‫ حُظ‬.e
anormale est mieux effectuée par:
a. Échographie
b. Tl-Tc scintigraphie
c. MIBI
d. Échographie et scintigraphie à MIBI
e. l‟IRM

14
26. The standard treatment for differentiated thyroid cancer 5 cm in ‫ش‬٤‫خٕ حُـيس حُيٍه‬١َُٔ ٢ٓ‫خ‬٤‫َ٘ٔ حُؼالؽ حُو‬٣
diameter without adenopathy includes: ٍ‫ٕ حػظال‬ٝ‫ ْٓ ى‬5 َٙ‫زِؾ هط‬٣ ١ٌُ‫ِ ح‬٣‫حُٔظٔخ‬
a- An ipsilateral lobo-isthmectomy :٢ِ٣ ‫ ٓخ‬١‫ؿي‬
b- An ipsilateral lobo-isthmectomy with ْ‫ حُزَُم ر٘ل‬ٚ‫خٍ ك‬ٜ‫ حٓظج‬.a
ipsilateral lymph node dissection ‫حُـخٗذ‬
c- An ipsilateral lobo-isthmectomy with ْ‫ حُزَُم ر٘ل‬ٚ‫خٍ ك‬ٜ‫ حٓظج‬.b
bilateral lymph node dissection ‫ق حُؼويس‬٣َ٘‫حُـخٗذ ٓغ ط‬
d- A subtotal thyroidectomy with bilateral ‫ش ر٘لْ حُـخٗذ‬٣ٝ‫ٔلخ‬٤ُِ‫ح‬
ْ‫ حُزَُم ر٘ل‬ٚ‫خٍ ك‬ٜ‫ حٓظج‬.c
dissection.
‫ حُـخٗذ‬٢‫ق ػ٘خث‬٣َ٘‫حُـخٗذ ٓغ ط‬
e- A total thyroidectomy
‫ش‬٣ٝ‫ٔلخ‬٤ُِ‫ُِؼويس ح‬
‫ ٓغ‬٢‫ش حُـِث‬٤‫خٍ حُـيس حُيٍه‬ٜ‫ حٓظج‬.d
.‫ حُـخٗذ‬٢‫ق ػ٘خث‬٣َ٘‫ط‬
Le traitement standard du cancer différencié de la thyroïde de 5 cm de ٢ٌُِ‫ش ح‬٤‫خٍ حُـيس حُيٍه‬ٜ‫ حٓظج‬.e
diamètre sans adénopathie comprend :
a. Une lobo-isthmectomie ipsilatérale
b. Une lobo-isthmectomie ipsilatérale avec curage ganglionnaire
ipsilatéral
c. Une lobo-isthmectomie ipsilatérale avec curage ganglionnaire
bilatéral
d. Une thyroïdectomie subtotale avec curage bilatéral.
e. Une thyroïdectomie totale
27. An intervention for nodular thyroid pathology can be proposed, ‫ش‬٤‫ حُـيس حُيٍه‬ٝ‫ٌٖٔ حهظَحف طيهَ ألَٓح‬٣
without cytopuncture, in all these situations EXCEPT one: ٌٙٛ ًَ ٢‫ ك‬،١ِٞ‫ٕ حُزٍِ حُو‬ٝ‫ ى‬،‫ش‬٣‫ي‬٤‫حُؼو‬
a- Single nodule 4 cm in diameter :‫حُلخالص ٓخ ػيح‬
b- Multinodular goiter ْٓ 4 ‫خ‬َٛ‫يس ٓلَىس هط‬٤‫ ػو‬.a
c- Nodule in Graves' disease ‫ش ٓظؼيى‬٤‫وْ حُـيس حُيٍه‬٠‫ ط‬.b
d- Nodule with history of old irradiation of the ‫يحص‬٤‫حُؼو‬
neck ِ‫ل‬٣َ‫ ؿ‬َٝٓ ٢‫يحص ك‬٤‫ حُؼو‬.c
ْ٣‫غ حُوي‬٤‫ن ٖٓ حُظ٘ؼ‬٣ٍ‫خ طخ‬ُٜ ‫يس‬٤‫ ػو‬.d
‫َُِهزش‬
Une intervention pour pathologie nodulaire thyroïdienne peut-elle être
proposée, sans cytopuncture, dans toutes ces situations SAUF une:
a. Nodule unique de 4 cm de diamètre
b. Goitre multinodulaire
c. Nodule dans la maladie de Graves
d. Nodule avec antécédents d‟irradiation ancienne du cou

15
28. Regarding Graves' disease, choose the wrong answer: ‫ حهظَ حإلؿخرش‬،ِ‫ل‬٣َ‫ظؼِن ريحء ؿ‬٣ ‫ٔخ‬٤‫ك‬
a- It is the most common form of :‫جش‬١‫حُوخ‬
thyrotoxicosis ‫ػخ‬ٞ٤ٗ َ‫ حٌَُ٘ حألًؼ‬ٞٛ ‫ٌح‬ٛ .a
b- It is more common in men ٢‫ُِظْٔٔ حُيٍه‬
c- It can be associated with extra-thyroid ٍ‫ػخ ػ٘ي حَُؿخ‬ٞ٤ٗ َ‫ أًؼ‬ٞٛ .b
manifestations ‫َ هخٍؽ‬ٛ‫ظَحكن ٓغ ٓظخ‬٣ ٕ‫ٌٖٔ أ‬٣ .c
d- It's due to an autoimmune disorder ‫ش‬٤‫حُـيس حُيٍه‬
٢‫طَحد ٓ٘خػ‬ٟ‫ ٗخطؾ ػٖ ح‬ٞٛ .d
e- It is characterized by the triad: goiter,
٢‫ًحط‬
thyrotoxicosis, exophthalmos
‫وْ حُـيس‬٠‫ ط‬:‫ع‬ُٞ‫ِ رخُؼخ‬٤ٔ‫ظ‬٣ .e
En ce qui concerne la maladie de Graves, choisissez la mauvaise ‫ظ‬ٞ‫ ؿل‬،٢‫ حُظْٔٔ حُيٍه‬،‫ش‬٤‫حُيٍه‬
réponse: ٖ٤‫حُؼ‬

a. C‟est la forme la plus courante de thyrotoxicose


b. Il est plus fréquent chez les hommes
c. Il peut être associé à des manifestations extra-thyroïdiennes
d. C‟est dû à une maladie auto-immune
e. Il se caractérise par la triade: goitre, thyrotoxicose,
exophtalmie
29. The superior thyroid artery is a branch of: :ٖٓ ‫ كَع‬ٞٛ ١ِٞ‫ حُؼ‬٢‫خٕ حُيٍه‬٣َُ٘‫ح‬
a- The internal carotid artery ٢ِ‫ حُيحه‬٢‫خٕ حُٔزخط‬٣َُ٘‫ ح‬.a
b- The external carotid artery ٢‫ حُوخٍؿ‬٢‫خٕ حُٔزخط‬٣َُ٘‫ ح‬.b
c- The primitive carotid artery ٢‫ حُزيحث‬٢‫خٕ حُٔزخط‬٣َُ٘‫ ح‬.c
d- The subclavian artery ‫س‬ٞ‫خٕ طلض حُظَه‬٣َُ٘‫ ح‬.d
e- The facial artery ٢ٜ‫ؿ‬ُٞ‫خٕ ح‬٣َُ٘‫ ح‬.e

L‟artère thyroïdienne supérieure est une branche de:


a. L‟artère carotide interne
b. L‟artère carotide externe
c. L‟artère carotide primitive
d. L‟artère sous-clavière
e. L‟artère facial

16
30. Regarding the thyroid gland, all of the following are true EXCEPT: ‫ق‬٤‫ل‬ٛ ٢ِ٣ ‫ ًَ ٓخ‬،‫ش‬٤‫ظؼِن رخُـيس حُيٍه‬٣ ‫ٔخ‬٤‫ك‬
a- It is related to the external branch of the :‫ٓخ ػيح‬
superior laryngeal nerve ‫ذ‬ٜ‫ ُِؼ‬٢‫ رخُلَع حُوخٍؿ‬٢‫ طَطز‬.a
b- It is vascularized by the superior thyroid ١ِٞ‫ حُؼ‬١َ‫حُل٘ـ‬
artery and the inferior thyroid artery ٖٓ ‫ش‬٣ٞٓ‫ش ى‬٤‫ػ‬ٝ‫خ رؤ‬ٜ‫ظ‬٣ٌ‫ظْ طـ‬٣ .b
c- It is related to the parathyroid glands ٕ‫خ‬٣َُ٘‫ح‬ٝ ١ِٞ‫ حُؼ‬٢‫خٕ حُيٍه‬٣َُ٘‫ح‬
d- It is located in front of the thyroid ٢ِ‫ حُٔل‬٢‫حُيٍه‬
‫ رـخٍحص حُيٍم‬٢‫ طَطز‬.c
cartilage
‫ش‬٤‫ف حُـيس حُيٍه‬َٝ٠‫ طوغ أٓخّ ؿ‬.d
e- It is related to the recurrent nerve
‫ذ حَُحؿغ‬ٜ‫ رخُؼ‬٢‫ طَطز‬.e

En ce qui concerne la glande thyroïde, tous les éléments suivants sont


vrais SAUF:
a. Il est lié à la branche externe du nerf laryngé supérieur
b. Il est vascularisé par l‟artère thyroïdienne supérieure et l‟artère
thyroïdienne inférieure
c. Il est lié aux glandes parathyroïdes
d. Il est situé devant le cartilage thyroïdien
e. Il est lié au nerf récurrent
31. Regarding the innervation of the levator ani muscles and the external ‫الص حَُحكؼش‬٠‫ذ حُؼ‬٤ٜ‫ظؼِن رظؼ‬٣ ‫ٔخ‬٤‫ك‬
sphincter, choose the correct answer: ‫ حهظَ حإلؿخرش‬،‫ش‬٤‫َس حُوخٍؿ‬ُٜٔ‫ح‬ٝ ‫َُِ٘ؽ‬
a- These muscles are innervated by the :‫لش‬٤‫ل‬ُٜ‫ح‬
branches of the lumbosciatic plexus ‫ع‬َٝ‫زش رل‬ٜ‫الص ٓؼ‬٠‫ حُؼ‬ٌٙٛ .a
b- These muscles are innervated by the ‫ش‬٤ًٍُٞ‫ش ح‬٤٘‫َس حُوط‬٤‫ل‬٠ُ‫ح‬
pudendal nerve ‫ذ‬ٜ‫زش رخُؼ‬ٜ‫الص ٓؼ‬٠‫ حُؼ‬ٌٙٛ .b
c- The levator muscles are innervated by ٢‫حُلَؿ‬
the lumbosciatic plexus, the external ‫الص حَُحكؼش‬٠‫ذ حُؼ‬٤ٜ‫ظْ طؼ‬٣ .c
،‫ش‬٤ًٍُٞ‫ش ح‬٤٘‫َس حُوط‬٤‫ل‬٠ُ‫حٓطش ح‬ٞ‫ر‬
sphincter by the pudendal nerve
‫حٓطش‬ٞ‫ش ر‬٤‫َس حُوخٍؿ‬ُٜٔ‫ح‬ٝ
d- All of these answers are correct
٢‫ذ حُلَؿ‬ٜ‫حُؼ‬
‫لش‬٤‫ل‬ٛ ٙ‫رش أػال‬ٞ‫ ًَ حألؿ‬.d
En ce qui concerne l‟innervation des muscles releveurs de l‟anus et du
sphincter externe, choisissez la bonne réponse:
a. Ces muscles sont innervés par les branches du plexus
lombosciatique
b. Ces muscles sont innervés par le nerf pudendal
c. Les muscles releveurs sont innervés par le plexus
lombosciatique, le sphincter externe par le nerf pudendal
d. Toutes ces réponses sont correctes

17
32. All these elements are parameters influencing anal continence, ٠ِ‫َ طئػَ ػ‬٤٣‫ ٓؼخ‬٢ٛ َٛ‫ حُؼ٘خ‬ٌٙٛ ًَ
EXCEPT one: :‫ ٓخ ػيح‬،٢‫َ حَُ٘ؿ‬ٜ‫حُل‬
a- Rectal compliance ْ٤‫ػش حُٔٔظو‬ٝ‫ ٓطخ‬.a
b- The good integrity of the sphincter ‫َس‬ُٜٔ‫خُ ح‬ٜ‫يس ُـ‬٤‫ ٓالٓش ؿ‬.b
apparatus َ‫ ٓخث‬ٝ‫ِذ أ‬ٛ :ُ‫ش حُزَح‬٤‫ػ‬ٞٗ .c
c- The quality of the stools: solid or liquid ‫يس‬٤‫ٕ ؿ‬ُٞٞ‫ش ه‬٤ًَ‫ ك‬.d
d- Good colonic motility ٢‫ذ حُلَؿ‬ٜ‫ ٓالٓش حُؼ‬.e
e- The integrity of the pudendal nerve
Tous ces éléments sont des paramètres influençant la continence
anale, SAUF un:
a. Observance rectale
b. La bonne intégrité de l‟appareil du sphincter
c. La qualité des selles : solides ou liquides
d. Bonne motilité colique
e. L‟intégrité du nerf pudendal

33. On physical examination of a prolapse of the rectal mucosa, choose ٢١‫ حُـ٘خء حُٔوخ‬١ٞ‫ز‬ُٜ ٢ٗ‫ حُزي‬ٚ‫ػ٘ي حُلل‬
the correct answer: :‫لش‬٤‫ل‬ُٜ‫ حهظَ حإلؿخرش ح‬،ْ٤‫ُِٔٔظو‬
a- Pushing out through the anus a sleeve ‫ش‬٤١‫خص ٓوخ‬٤١ ‫ حُيكغ رـالف ٓغ‬.a
with circumferential mucous folds ‫ش ٖٓ هالٍ كظلش حَُ٘ؽ‬٤‫ط‬٤‫ٓل‬
b- A bulge in the posterior wall of the َ‫ز‬ُِٜٔ ٢‫ حُـيحٍ حُوِل‬٢‫ حٗظلخم ك‬.b
vagina in thrust ‫ػ٘ي حُيكغ‬
c- Pushing out through the anus a small ‫ حُيكغ ٖٓ هالٍ كظلش حَُ٘ؽ‬.c
prolapse with centrifugal folds ‫خص‬٤١ ‫َ ٓغ‬٤‫ـ‬ٛ ١ٞ‫ز‬ٛ ‫ليع‬٣
١ًَُِٔ‫حُطَى ح‬
d- In pushing nothing comes out of the
‫ء ٖٓ كظلش‬٢ٗ ‫وَؽ‬٣ ‫ ػ٘ي حُيكغ ال‬.d
anus, but it causes a bulge in the
٢‫ٔزذ حٗظلخم ك‬٣ ٌُٚ٘ ،‫حَُ٘ؽ‬
perineum ٕ‫حُؼـخ‬
e- In pushing it provokes the exteriorization ٍ‫ؽ حُيّ ٖٓ هال‬َٝ‫َ حُيكغ ه‬٤‫ؼ‬٣ .e
of blood through the anus ‫كظلش حَُ٘ؽ‬

Lors de l‟examen physique d‟un prolapsus de la muqueuse rectale,


choisissez la bonne réponse:
a. Pousser à travers l‟anus une manche avec des plis muqueux
circonférentiels
b. Un renflement dans la paroi postérieure du vagin en poussée
c. Pousser à travers l‟anus un petit prolapsus avec des plis
centrifuges
d. En poussant rien ne sort de l‟anus, mais cela provoque un
renflement dans le périnée
e. En poussant, il provoque l‟extériorisation du sang par l‟anus

18
34. In the immediate postoperative period which of these incidents is a ٖٓ ١‫ أ‬،‫ حُلظَس حُٔزخَٗس ٓخ رؼي حُـَحكش‬٢‫ك‬
life-threatening emergency to treat: ‫خس‬٤‫يىس ُِل‬ٜٓ ‫خٍثش‬١ ‫ كخُش‬٢ٛ ‫حىع‬ٞ‫ حُل‬ٌٙٛ
a- The patient complains of pain at the :‫ حُؼالؽ‬٢‫طٔظيػ‬
operative site ‫هغ‬ٞٓ ٢‫ ٖٓ أُْ ك‬ٞ٣َُٔ‫ ح‬ٌٞ٘٣ .a
b- Arrived in the recovery room the patient ‫ش‬٤ِٔ‫حُؼ‬
with an impossibility of breathing ٕ‫ ؿَكش حإلٗؼخ‬٠ُ‫ ا‬ٞ٣َُٔ‫َ ح‬ٜ٣ .b
c- The patient complains of nausea ْ‫ حُظ٘ل‬٢‫ٓغ حٓظلخُش ك‬
d- Oliguria during diuresis monitoring ٕ‫خ‬٤‫ ٖٓ حُـؼ‬ٞ٣َُٔ‫ ح‬ٌٞ٘٣ .c
ٍٞ‫ٍ أػ٘خء َٓحهزش اىٍحٍ حُز‬ٞ‫ هِش حُز‬.d
e- Arrival in the recovery room the patient
،ٕ‫ ؿَكش حإلٗؼخ‬٠ُ‫ٍ ا‬ُٞٛٞ‫ ػ٘ي ح‬.e
shivers and is hypothermic
ٞ‫ط٘ول‬ٝ ٞ٣َُٔ‫َطـق ح‬٣
Dans la période postopératoire immédiate, lequel de ces incidents est ٚ‫كَحٍط‬
une urgence potentiellement mortelle à traiter:
a. Le patient se plaint de douleurs au site opératoire
b. Arrivé dans la salle de réveil le patient avec une impossibilité
de respirer
c. Le patient se plaint de nausées
d. Oligurie pendant la surveillance de la diurèse
e. Arrivée dans la salle de réveil le patient frissonne et est en
hypothermie

35. Regarding deep vein thrombosis, choose the correct answer: َ‫ حهظ‬،‫ن‬٤ٔ‫ حُؼ‬١‫ي‬٣ٍُٞ‫ظؼِن رخُوؼخٍ ح‬٣ ‫ٔخ‬٤‫ك‬
a- The presence of varicose veins of the :‫لش‬٤‫ل‬ُٜ‫حإلؿخرش ح‬
lower limbs is a risk factor for DVT ‫َحف‬١‫ حأل‬٢‫ ك‬٢ُ‫ح‬ٝ‫ى حُي‬ٞ‫ؿ‬ٝ .a
b- Cancerological pelvic surgery is a risk DVT ٍ َ‫ ػخَٓ هط‬ٞٛ ‫ش‬٤ِ‫حُٔل‬
factor for DVT ٢ٛ ‫ش‬٤ٗ‫خ‬١َُٔ‫ ح‬ٝٞ‫ ؿَحكش حُل‬.b
c- Hernia repair in a 20-year-old person with DVT ٍ َ‫ػخَٓ هط‬
a hypercoagulant syndrome is a risk ‫زِؾ‬٣ ٚ‫ ٗو‬ٟ‫الف حُلظن ُي‬ٛ‫ؼي ا‬٣ .c
‫خرخ‬ٜٓ ‫ ػخٓخ‬20 َٔ‫ٖٓ حُؼ‬
factor for DVT
َ‫ حُظوؼَ ػخَٓ هط‬١َ‫رٔظالُٓش ك‬
d- All of these answers are correct DVT ‫خرش د‬ٛ‫ُإل‬
e- All these answers are wrong ‫لش‬٤‫ل‬ٛ ٙ‫رش أػال‬ٞ‫ ًَ حألؿ‬.d
‫لش‬٤‫ل‬ٛ َ٤‫ ؿ‬ٙ‫رش أػال‬ٞ‫ ًَ حألؿ‬.e
En ce qui concerne la thrombose veineuse profonde, choisissez la
bonne réponse:
a. La présence de varices des membres inférieurs est un facteur
de risque de TVP
b. La chirurgie pelvienne cancérologique est un facteur de risque
de TVP
c. La réparation d‟une hernie chez une personne de 20 ans
atteinte d‟un syndrome hypercoagulant est un facteur de risque
de TVP
d. Toutes ces réponses sont correctes
e. Toutes ces réponses sont fausses

19
36. All of the following are factors predisposing to postoperative ٙ‫زش الٗؤخ‬ٛ‫حَٓ ٓئ‬ٞ‫ ػ‬٢ٛ ٢ِ٣ ‫ًَ ٓخ‬
atelectasis EXCEPT: :‫ ٓخ ػيح‬٢‫حَُثش رؼي حُؼَٔ حُـَحك‬
a- Long-term abdominal surgery ‫ال‬٣ٞ١ ‫هظخ‬ٝ ‫ ؿَحكش رطٖ حٓظـَهض‬.a
b- A smoking patient ٖ‫ ٓيه‬ٞ٣َٓ .b
c- Abdominal surgery with poor pain control ٢‫ؼق ك‬ٟ ‫ ؿَحكش رطٖ ٓغ‬.c
d- A hernia repair under local anesthesia ُْ‫ حأل‬٠ِ‫طَس ػ‬٤ُٔ‫ح‬
e- Thoracic surgery َ٣‫الف كظن طلض حُظوي‬ٛ‫ ا‬.d
٢‫ؼ‬ُٟٞٔ‫ح‬
‫ش‬٣ٍ‫ي‬ٛ ‫ ؿَحكش‬.e
Tous les facteurs suivants sont des facteurs prédisposant à
l‟atélectasie postopératoire SAUF:
a. Chirurgie abdominale à long terme
b. Un patient fumeur
c. Chirurgie abdominale avec un mauvais contrôle de la douleur
d. Une réparation de hernie sous anesthésie locale
e. Chirurgie thoracique

37. All of the following can reduce postoperative morbidity or detect it ‫ رؼي‬َُٝٔ‫وَِ ٖٓ ح‬٣ ٕ‫ٌٖٔ أ‬٣ ٢ِ٣ ‫ًَ ٓخ‬
earlier, EXCEPT for one: ٢‫ ك‬ٚ‫ حًظ٘خك‬٠ِ‫ٔخػي ػ‬٣ٝ‫ش أ‬٤‫ش حُـَحك‬٤ِٔ‫حُؼ‬
a- Change the infusion site every 3 days :‫ ٓخ ػيح‬،ٌَ‫هض ٓز‬ٝ
b- It is not necessary to check the surgical ّ‫خ‬٣‫ أ‬3 ًَ ‫ذ‬٣َٔ‫هغ حُظ‬ٞٓ َ٤٤‫ طـ‬.a
site before the third day, especially if the ‫هغ‬ٞٓ ٚ‫ كل‬١ٍَٝ٠ُ‫ْ ٖٓ ح‬٤ُ .b
surgery is clean ‫ش‬ٛ‫ هخ‬،‫ّ حُؼخُغ‬ٞ٤ُ‫حُـَحكش هزَ ح‬
c- Ambulate the patient as soon as possible to ‫لش‬٤‫اًح ًخٗض حُـَحكش ٗظ‬
avoid DVT ‫هض‬ٝ ‫ أَٓع‬٢‫ ك‬ٞ٣َُٔ‫ آؼخف ح‬.c
DVT ‫ٌٖٓٔ ُظـ٘ذ‬
d- Make a daily urinary culture in a patient
‫خ ػ٘ي‬٤ٓٞ٣ ٍٞ‫ اؿَحء ٍُع ر‬.d
without risk factor
‫يىس‬ٜٓ َٓ‫ح‬ٞ‫ ػ‬ٚ٣‫ْ ُي‬٤ُ ٞ٣َٓ
e- Teach the patient to perform spirometry as ْ‫خّ حُظ٘ل‬٤‫ اؿَحء ه‬ٞ٣َُٔ‫ْ ح‬٤ِ‫ طؼ‬.e
early as possible ٌٖٔٓ ‫هض‬ٝ ‫ أهَد‬٢‫ك‬

Tous les éléments suivants peuvent réduire la morbidité


postopératoire ou la détecter plus tôt, SAUF un:
a. Changez le site de perfusion tous les 3 jours
b. Il n‟est pas nécessaire de vérifier le site chirurgical avant le
troisième jour, surtout si la chirurgie est propre
c. Déambuler le patient dès que possible pour éviter la TVP
d. Faire une culture urinaire quotidienne chez un patient sans
facteur de risque
e. Apprenez au patient à effectuer la spirométrie le plus tôt
possible

20
38. Postoperative fever on the first day is due to all of these causes ّٞ٤ُ‫ ح‬٢‫ ٓخ رؼي حُـَحكش ك‬٠ٔ‫ى ٓزذ حُل‬ٞ‫ؼ‬٣
EXCEPT one: :‫ ٓخ ػيح‬٢ِ٣ ‫ ًَ ٓخ‬٠ُ‫ٍ ا‬ٝ‫حأل‬
a- Pulmonary atelectasis ‫ حَُثش‬ٙ‫ حٗؤخ‬.a
b- An abscess of walls at the level of the ‫هغ‬ٞٓ ٟٞ‫ ٓٔظ‬٠ِ‫ هَحؽ حُـيٍحٕ ػ‬.b
operating site after a clean surgery ‫لش‬٤‫ش رؼي حُـَحكش حُ٘ظ‬٤ِٔ‫حُؼ‬
c- Phlebitis at the infusion site ‫ذ‬٣َٔ‫هغ حُظ‬ٞٓ ٢‫ي ك‬٣ٍُٞ‫خد ح‬ٜ‫ حُظ‬.c
d- Reaction to blood transfusion ّ‫ ٗوَ حُي‬٠ِ‫ ٍى كؼَ ػ‬.d
e- After a very septic surgery, the fever of the ٌٕٞ‫ ط‬،‫ش‬٣‫ش ُِـخ‬٤ٗ‫ رؼي ؿَحكش اٗظخ‬.e
‫ش‬٤ٌ٤ٓ‫ٍ ًال‬ٝ‫ّ حأل‬ٞ٤ُ‫ ح‬٠ٔ‫ك‬
first day is classic

La fièvre postopératoire du premier jour est due à toutes ces causes


SAUF une:
a. Atélectasie pulmonaire
b. Un abcès des parois au niveau du site opératoire après une
chirurgie propre
c. Phlébite au site de perfusion
d. Réaction à la transfusion sanguine
e. Après une chirurgie très septique, la fièvre du premier jour est
classique
39. Concerning the anal incontinence symptoms, choose the wrong َ‫ حهظ‬،٢‫ حُِْٔ حَُ٘ؿ‬ٝ‫ظؼِن رؤػَح‬٣ ‫ٔخ‬٤‫ك‬
answer: :‫جش‬١‫حإلؿخرش حُوخ‬
a- Urge to defecate َُ‫ حُظز‬٠ِ‫ كغ ػ‬.a
b- Inability to contain gas ُ‫حء حُـخ‬ٞ‫ حكظ‬٠ِ‫ ػيّ حُويٍس ػ‬.b
c- Defiling at night, morning awakening ٢‫زخك‬ٛ ‫وخظ‬٤‫ حٓظ‬، َ٤ُِ‫ ح‬٢‫ع ك‬ِٞ‫ ط‬.c
soiled with feces ُ‫ٓظٔن رخُزَح‬
d- Inability to contain intense watery ٍ‫خ‬ٜٓ‫حء حإل‬ٞ‫ حكظ‬٠ِ‫ ػيّ حُويٍس ػ‬.d
diarrhea, even in young patients ٠َُٟٔ‫ ػ٘ي ح‬٠‫ كظ‬،‫ي‬٣‫ حُ٘ي‬٢‫حُٔخث‬
ُٖٔ‫ ح‬٢‫ـخٍ ك‬ُٜ‫ح‬
e- Inability to contain solid stools
ُ‫حء حُزَح‬ٞ‫ حكظ‬٠ِ‫ ػيّ حُويٍس ػ‬.e
‫ِذ‬ُٜ‫ح‬

En ce qui concerne les symptômes de l‟incontinence anale, choisissez


la mauvaise réponse:
a. Envie de déféquer
b. Incapacité à contenir les gaz
c. Souillure la nuit, réveil matinal souillé par les matières fécales
d. Incapacité à contenir la diarrhée aqueuse intense, même chez
les jeunes patients
e. Incapacité à contenir les selles solides

21
40. Chronic constipation could be the cause of all of the following ‫ ٓزذ‬ٞٛ ُِٖٓٔ‫ٕ حإلٓٔخى ح‬ٌٞ٣ ٕ‫ٌٖٔ أ‬٣
conditions EXCEPT: :‫ش ٓخ ػيح‬٤ُ‫غ حُلخالص حُظخ‬٤ٔ‫ؿ‬
a- Anal fissure ٢‫ حُ٘ن حَُ٘ؿ‬.a
b- Descending perineum with polyorgan ‫ ٓظؼيى‬١ٞ‫ز‬ٛ ‫ حُؼـخٕ ٓغ‬١ٞ‫ز‬ٛ .b
prolapse ‫خء‬٠‫حألػ‬
c- Excess weight ‫ُٕ حُِحثي‬ُٞ‫ ح‬.c
d- Grade II hemorrhoids ‫ش‬٤ٗ‫َ ٖٓ حُيٍؿش حُؼخ‬٤ٓ‫ح‬ٞ‫ حُز‬.d
e- Inguinal hernia ٢‫ حُلظن حإلٍر‬.e

La constipation chronique pourrait être la cause de toutes les


conditions suivantes SAUF:
a. Fissure anale
b. Périnée descendant avec prolapsus polyorganique
c. Excès de poids
d. Hémorroïdes de grade II
e. Hernie inguinale

41. Choose the correct sequence concerning the physical exam in acute ٚ‫ظؼِن رخُلل‬٣ ‫ٔخ‬٤‫ق ك‬٤‫ل‬ُٜ‫حهظَ حُظَِٔٔ ح‬
abdomen: :‫ حُزطٖ حُلخى‬٢‫ ك‬٢ٗ‫حُزي‬
a- Auscultation percussion inspection ْ‫ حُـ‬١َ‫ حُظٔٔغ حُوَع حُظل‬.a
palpation ْ‫ حُظٔٔغ حُوَع حُـ‬١َ‫ حُظل‬.b
b- Inspection auscultation percussion ‫ حُظٔٔغ حُـْ حُوَع‬١َ‫ حُظل‬.c
palpation ْ‫ حُوَع حُـ‬١َ‫ حُظٔٔغ حُظل‬.d
c- Inspection auscultation palpation ‫ حُـْ حُوَع حُظٔٔغ‬١َ‫ حُظل‬.e
percussion
d- Auscultation inspection percussion
palpation
e- Inspection palpation percussion
auscultation

Choisissez la bonne séquence concernant l‟examen physique dans


l‟abdomen aigu:
a. Auscultation percussion inspection palpation
b. Inspection auscultation percussion palpation
c. Inspection auscultation palpation percussion
d. Auscultation inspection percussion palpation
e. Inspection palpation percussion auscultation

22
42. Choose the wrong answer about hypercalcemia treatment: ١َ‫ٍ ػالؽ ك‬ٞ‫جش ك‬١‫حهظَ حإلؿخرش حُوخ‬
a- Administration of large volumes of hypertonic :ّ‫ّ حُي‬ٞ٤ُٔ‫ًخ‬
saline for patient with healthy kidneys ٍِٞ‫َس ٖٓ ٓل‬٤‫خص ًز‬٤ًٔ ‫ اػطخء‬.a
b- In severe hypercalcemia secondary to renal ًٝ ٞ٣َُٔ َ‫ط‬ٞ‫ حُظ‬٢ُ‫ ػخ‬٢‫ِٓل‬
failure, hemodialysis may be required ‫ٔش‬٤ِٓ ٠ًِ
c- Give bisphosphonates for patients whose ٢ُ‫ي حُظخ‬٣‫ّ حُيّ حُ٘ي‬ٞ٤ُٔ‫ ًخ‬١َ‫ ك‬٢‫ ك‬.b
hypercalcemia is caused by malignancy ‫٘خى‬ٛ ٌٕٞ‫ هي ط‬،١ٌُِٞ‫ُِلَ٘ ح‬
٠ٌُِ‫َ ح‬٤ٔ‫كخؿش ُـ‬
d- Parathyroidectomy for hypercalcemia caused by
٠َُِٟٔ ‫ٗخص‬ٞ‫ٓل‬ٞ‫ِك‬٤‫ اػطخء ر‬.c
hyperparathyroidism
ّٞ٤ُٔ‫ ًخ‬١َ‫ٕ ٖٓ ك‬ٞٗ‫ؼخ‬٣ ٖ٣ٌُ‫ح‬
Choisissez la mauvaise réponse au traitement de l‟hypercalcémie: ‫غ‬٤‫ٍّ هز‬ٝ ‫حُيّ رٔزذ‬
a. Administration de grands volumes de solution saline ‫خٍ ؿخٍحص حُيٍم ػ٘ي‬ٜ‫ حٓظج‬.d
hypertonique pour les patients ayant des reins sains ْ‫ّ حُيّ ٗخؿ‬ٞ٤ُٔ‫ ًخ‬١َ‫خرش رل‬ٛ‫حإل‬
b. En cas d‟hypercalcémie sévère secondaire à une insuffisance ‫ ؿخٍحص حُيٍم‬١‫ ٗ٘خ‬١َ‫ػٖ ك‬
rénale, une hémodialyse peut être nécessaire
c. Administrer des bisphosphonates aux patients dont
l‟hypercalcémie est causée par une malignité
d. Parathyroïdectomie pour l‟hypercalcémie causée par
l‟hyperparathyroïdie

43. One wrong answer about hyperkalemia etiologies: ١َ‫ٍ ٓٔززخص ك‬ٞ‫جش ك‬١‫حكيس هخ‬ٝ ‫اؿخرش‬
a- Crush injuries, burns and other catabolism- :ّ‫ّ حُي‬ٞ٤ٓ‫طخ‬ٞ‫ر‬
inducing events ‫م‬َٝ‫حُل‬ٝ ‫خرخص حُٔلن‬ٛ‫ ا‬.a
b- Renal insufficiency َٝ‫ طل‬٢‫خ ٖٓ حألكيحع حُظ‬َٛ٤‫ؿ‬ٝ
c- Adrenal insufficiency, and excessive K+ ٞ٣ٞ‫ حُظو‬٠ِ‫ػ‬
administration ١ٌُِٞ‫ٍ ح‬ٜٞ‫ حُو‬.b
d- Alkalosis ٢‫ ك‬١‫حإلكَح‬ٝ ،َ‫ٍ ؿيس حٌُظ‬ٜٞ‫ ه‬.c
+ K ‫اػطخء‬
Une mauvaise réponse sur les étiologies de l‟hyperkaliémie: ‫ حُوالء‬.d
a. Blessures par écrasement, brûlures et autres événements
induisant le catabolisme
b. Insuffisance rénale
c. Insuffisance surrénale et administration excessive de K+
d. Alcalose

44. Which of the following signs is not suggestive of acute abdomen: ٖ‫ رخُزط‬٢‫ك‬ٞ‫ش ال ط‬٤ُ‫ ٖٓ حُؼالٓخص حُظخ‬١‫أ‬
a- Guarding :‫حُلخى‬
b- Rebound tenderness ٢ِ٠‫ ٓ٘ؼٌْ حُيكخع حُؼ‬.a
c- Active bowel sounds ‫الّ حالٍطيحى‬٣‫ ا‬.b
d- Rigidity ‫ حألٓؼخء‬٢‫حص ٗ٘طش ك‬ٞٛ‫ أ‬.c
e- Fever ‫الرش‬ُٜ‫ ح‬.d
٠ٔ‫ حُل‬.e
Lequel des signes suivants ne suggère pas un abdomen aigu :
a. Protéger
b. la sensibilité du rebond
c. Sons intestinaux actifs
d. Rigidité
e. Fièvre

23
45. Which of the following are important in the evaluation of abdominal :ٖ‫ْ آالّ حُزط‬٤٤‫ طو‬٢‫ْ ك‬ٜٓ ٢ِ٣ ‫ ٓٔخ‬١‫أ‬
pain: ‫حُ٘يس‬ٝ ‫ش‬٤ٜ‫ حُ٘و‬.a
a- Character and severity ‫ض‬٤‫ه‬ٞ‫حُظ‬ٝ ‫هغ‬ُٞٔ‫ ح‬.b
b- Location and timing ‫حُٔوللش‬ٝ ‫حَٓ حُٔ٘يىس‬ٞ‫ حُؼ‬.c
c- Aggravating and alleviating factors ‫لش‬٤‫ل‬ٛ ٙ‫رش أػال‬ٞ‫ ًَ حألؿ‬.d
d- All of the above ‫لش‬٤‫ل‬ٛ َ٤‫ ؿ‬ٙ‫رش أػال‬ٞ‫ ًَ حألؿ‬.e
e- None of the above
Lesquels des éléments suivants sont importants dans l‟évaluation de la
douleur abdominale:
a. Caractère et gravité
b. Lieu et moment
c. Facteurs aggravants et atténuants
d. Tout ce qui précède
e. Aucune des réponses ci-dessus

46. All are indications in acute abdomen for urgent operation EXCEPT: ٢‫ اؿَحء ػَٔ ؿَحك‬٢‫ٔظيػ‬٣ ٢ِ٣ ‫ًَ ٓخ‬
a- Equivocal abdominal finding along with :‫ كخُش حُزطٖ حُلخى ٓخ ػيح‬٢‫ػخؿَ ك‬
septicemia ‫ش ِٓظزْ ٓظَحكوش ٓغ‬٤٘‫َ رط‬ٛ‫ح‬ٞ‫ ظ‬.a
b- Pneumoperitoneum ١ٞٓ‫حٗظخٕ ى‬
c- Incarcerated femoral hernia more than 7 ‫لخم‬ُٜ‫حف ح‬َٝ‫ حٓظ‬.b
hours with disorientation, defense and fever 7 ٖٓ َ‫ّ أًؼ‬ٞ‫ كظن كوٌ ٓلز‬.c
d- Mesenteric occlusion on angiography ‫ ىكخع‬،ٕ‫خ‬ٛٞ‫ٓخػخص ٓظَحكن ٓغ ط‬
٠ٔ‫ك‬ٝ
e- Absence of peristaltism on auscultation
َ٣ٜٞ‫ ط‬٢‫َ ك‬ٜ‫ظ‬٣ ٢‫و‬٣ٍ‫ حٗٔيحى ٓٔخ‬.d
Tous sont des indications dans l‟abdomen aigu pour une opération ‫ش‬٤‫ػ‬ٝ‫حأل‬
urgente SAUF: ‫حُظٔٔغ‬ ‫ػ٘ي‬ ‫حُظٔؼؾ‬ ‫خد‬٤‫ ؿ‬.e
a. Découverte abdominale équivoque avec septicémie
b. Pneumopéritoine
c. Hernie fémorale incarcérée plus de 7 heures avec
désorientation, défense et fièvre
d. Occlusion mésentérique à l‟angiographie
e. Absence de péristaltisme à l‟auscultation

47. All of the above are signs and symptoms of peritonitis EXCEPT: ‫خد‬ٜ‫ حُظ‬ٝ‫أػَح‬ٝ ‫ ػالٓخص‬ٞٛ ٢ِ٣ ‫ًَ ٓخ‬
a- Free air on plain films :‫لخم ٓخ ػيح‬ُٜ‫ح‬
b- Bradycardia ‫ش‬٣‫ حألكالّ حُؼخى‬٠ِ‫حء حُلَ ػ‬ُٜٞ‫ ح‬.a
c- Fever and chills ‫ء حُوِذ‬٢‫ ر‬.b
d- Acute abdomen ‫َس‬٣َ‫حُو٘ؼ‬ٝ ٠ٔ‫ حُل‬.c
‫ حُزطٖ حُلخى‬.d

Tout ce qui précède sont des signes et des symptômes de péritonite


SAUF:
a. Air libre sur les films ordinaires
b. Bradycardie
c. Fièvre et frissons
d. Abdomen aigu

24
48. All of the above are intraperitoneal organs EXCEPT: :‫لخم ٓخ ػيح‬ُٜ‫خء ىحهَ ح‬٠‫ أػ‬ٞٛ ٢ِ٣ ‫ًَ ٓخ‬
a- Liver ‫ حٌُزي‬.a
b- Spleen ٍ‫ حُطلخ‬.b
c- Stomach ‫ حُٔؼيس‬.c
d- Ascending colon ‫خػي‬ُٜ‫ٕ ح‬ُٞٞ‫ حُو‬.d

Tout ce qui précède sont des organes intrapéritonéaux SAUF:


a. Foie
b. Rate
c. Estomac
d. Ascendant Côlon

49. All of the following are indications for surgery in Pseudocyst of ٢‫ش ك‬٤‫ش ؿَحك‬٤ِٔ‫ ٓئَٗحص ُؼ‬٢ٛ ٢ِ٣ ‫ًَ ٓخ‬
Pancreas Except: :‫خّ ٓخ ػيح‬٣ٌَ٘‫ْ حٌُخًد ُِز‬٤ٌُ‫ح‬
a- Pseudocyst size >6 cm and symptomatic ْٓ 6> ‫ْ حٌُخًد‬٤ٌُ‫ كـْ ح‬.a
b- Bleeding ٝ‫ٓظَحكن ٓغ أػَح‬ٝ
c- Gastric outlet obstruction ‫ق‬٣ِٗ .b
d- Adhesion of stomach to abdominal wall ‫ حٗٔيحى ٓوَؽ حُٔؼيس‬.c
e- Jaundice ٖ‫خم حُٔؼيس رـيحٍ حُزط‬ٜ‫ حُظ‬.d
ٕ‫َهخ‬٤ُ‫ ح‬.e
Tous les éléments suivants sont des indications pour la chirurgie dans
Pseudokyste du pancréas Sauf:
a. Taille du pseudokyste >6 cm et symptomatique
b. Saignement
c. Obstruction de la sortie gastrique
d. Adhérence de l‟estomac à la paroi abdominale
e. Jaunisse

50. Cullen‟ s sign is an indication of: :٠ِ‫ ٓئَٗ ػ‬٢ٛ ٖ٤ًُٞ ‫ػالٓش‬
a- Bleeding esophageal varices ‫ء‬١َُٔ‫ ح‬٢ُ‫ح‬ٝ‫ق ى‬٣ِٗ .a
b- Chronic pancreatitis ُِٖٓٔ‫خّ ح‬٣ٌَ٘‫خد حُز‬ٜ‫ حُظ‬.b
c- Severe gastroenteritis ‫حألٓؼخء حُلخى‬ٝ ‫خد حُٔؼيس‬ٜ‫ حُظ‬.c
d- Necrotizing pancreatitis َ‫خّ حُ٘خه‬٣ٌَ٘‫خد حُز‬ٜ‫ حُظ‬.d

Le signe de Cullen est une indication de:


a. Saignement des varices œsophagiennes
b. Pancréatite chronique
c. Gastro-entérite sévère
d. Pancréatite nécrosante

25
51. Which artery provides the main blood supply to the vulva? ّ‫كَ آيحىحص حُي‬ٞ٣ ١ٌُ‫خٕ ح‬٣َُ٘‫ ح‬ٞٛ ‫ٓخ‬
‫ش ُِلَؽ؟‬٤ٔ٤‫حَُث‬
a. Pudendal ٢‫ حُلَؿ‬.a
b. Inferior hemorroidal ٢ِ‫ حُٔل‬١ٍٞٓ‫ حُزخ‬.b
c. Ilio-inguinal ٢‫ حألٍر‬٢‫ حُلَهل‬.c
d. Femoral ١ٌ‫ حُلو‬.d
‫ طلض حُٔؼيس‬٢ِ‫ حُٔل‬.e
e. Inferior hypogastric

Quelle artère fournit l‟apport sanguin principal à la vulve?


a. Pudendale
b. Hémorroïde inférieure
c. Ilio-inguinale
d. fémorale
e. inférieure hypogastrique

52. What is the recommended daily dose of Iron that women who may ٖٓ ‫خ‬ٜ‫ ر‬٠ُٛٞٔ‫ش ح‬٤ٓٞ٤ُ‫ حُـَػش ح‬٢ٛ ‫ٓخ‬
become pregnant should take? ٢‫حط‬ُِٞ‫خ حُ٘ٔخء ح‬ُٜٝ‫ـذ إٔ طظ٘خ‬٣ ٢‫ي حُظ‬٣‫حُلي‬
a. 1 mg ‫حَٓ؟‬ٞ‫زلٖ ك‬ٜ٣ ‫هي‬
b. 10mg ‫ ِٓؾ‬1 .a
c. 27mg ‫ ِٓؾ‬10 .b
d. 60mg ‫ ِٓؾ‬27 .c
‫ ِٓؾ‬60 .d

Quelle est la dose quotidienne recommandée de fer que les femmes


susceptibles de devenir enceintes devraient prendre?
a. 1 mg
b. 10mg
c. 27mg
d. 60mg

53. Which two high risk HPV types together account for approximately ٍُّٞ‫ّ ح‬َٝ٤‫ػخٕ ٖٓ ك‬ٞٗ ‫ٔخ‬ٛ ٢ُ‫ ٖٓ حُظخ‬١‫أ‬
70% of cervical cancers worldwide? ‫ٔؼالٕ ٓؼخ‬٣ ‫ٍس‬ٞ‫ حُوط‬٢ُ‫ ػخ‬١َ٘‫ حُز‬٢ٔ٤ِ‫حُل‬
a. 6 and 11 ٢‫خٗخص ػ٘ن حَُكْ ك‬١َٓ ٖٓ ٪70 ٢ُ‫ح‬ٞ‫ك‬
b. 11 and 45 ‫غ أٗلخء حُؼخُْ؟‬٤ٔ‫ؿ‬
c. 16 and 18 11ٝ 6 .a
d. 18 and 31 45 ٝ 11 .b
e. 31 and 45. 18 ٝ 16 .c
31 ٝ 18 .d
.45ٝ 31 .e
Quels sont les deux types de VPH à haut risque qui représentent
ensemble environ 70% des cancers du col de l‟utérus dans le monde?
a. 6 et 11
b. 11 et 45
c. 16 et 18
d. 18 et 31
e. 31 et 45.

26
54. In the management of molar pregnancy, the first step would be ‫س‬ٞ‫ٕ حُوط‬ٌٞ‫ ٓظ‬،١ٞ‫ اىحٍس حُلَٔ حَُك‬٢‫ك‬
a. procrastination ٢ٛ ٠ُٝ‫حأل‬
b. proceed to evacuation ‫ِش‬١‫ حُٔٔخ‬.a
c. prophylactic chemotherapy ‫ ٓظخرؼش حإلهالء‬.b
d. monitor with HCG ٢‫هخث‬ُٞ‫ ح‬٢‫خث‬٤ٔ٤ٌُ‫ حُؼالؽ ح‬.c
HCG ّ‫ َٓحهزش رخٓظويح‬.d

Dans la prise en charge de la grossesse molaire, la première étape


serait
a. la procrastination
b. procéder à l‟évacuation
c. de chimiothérapie prophylactique
d. moniteur avec HCG
55. What fetal complication is associated with indomethacin use? ‫ش حَُٔطزطش‬٤٘٤٘‫خػلخص حُـ‬٠ُٔ‫ ح‬٢ٛ ‫ٓخ‬
‫ٖ؟‬٤ٓ‫ظخ‬٤ٓٝ‫رخٓظويحّ حإلٗي‬
a. Hydramnios ٠ُِٔ‫ ح‬ٙٞٓ .a
b. Pulmonary valve atresia ١ٞ‫ٔخّ حَُث‬ُٜ‫ ٍطن ح‬.b
c. Bronchopulmonary dysplasia ١ٞ‫ حَُث‬٢‫ز‬ٜ‫ هَِ حُظ٘ٔؾ حُو‬.c
d. Premature closure of the ductus arteriosus ‫ش‬٤ٗ‫خ‬٣َُ٘‫ حإلؿالم حُٔزٌَ ُِو٘خس ح‬.d

Quelle complication fœtale est associée à l‟utilisation de


l‟indométacine?
a. Hydramnios
b. Atrésie valvulaire pulmonaire
c. Dysplasie bronchopulmonaire
d. Fermeture prématurée du canal artériel
56. The most common neoplastic ovarian masses in post-menopausal ٟ‫ػخ ُي‬ٞ٤ٗ َ‫ش حألًؼ‬٤ٍُٓٞ‫ ح‬ٞ٤‫ط٘٘ؤ ًظَ حُٔز‬
women originates from which of the following? ‫؟‬٢ِ٣ ‫ ٓٔخ‬١‫حُ٘ٔخء رؼي حٗوطخع حُطٔغ ٖٓ أ‬
ٞ٤‫خٍس حُٔز‬ٜ‫ ظ‬.a
a. Ovarian epithelium ٞ٤‫ حُٔز‬ٟ‫ ٓي‬.b
b. Ovarian stroma ‫ش‬٤٠٤‫حُٔز‬ ‫ش‬٤ٓٞ‫حُـَػ‬ ‫خ‬٣‫ حُوال‬.c
c. Ovarian germ cells ٢٠٤‫ كزخٍ حُـْ٘ حُٔز‬.d
d. Ovarian sex cords .‫لش‬٤‫ل‬ٛ َ٤‫ ؿ‬ٙ‫رش أػال‬ٞ‫ ًَ حألؿ‬.e
e. None of the above.

Les masses ovariennes néoplasiques les plus courantes chez les


femmes ménopausées proviennent de laquelle des suivantes?
a. Épithélium ovarien
b. Stroma ovarien
c. Cellules germinales ovariennes
d. Cordons sexuels ovariens
e. Aucune de ces réponses.

27
57. Use of which of the following drugs most clearly decreases combined ٌَ٘‫وَِ ر‬٣ ‫ش‬٤ُ‫ش حُظخ‬٣ٝ‫ ٖٓ حألى‬١‫حٓظويحّ أ‬
hormonal contraceptive efficacy? َٔ‫ٓخثَ ٓ٘غ حُل‬ٝ ‫ش‬٤ُ‫ق ٖٓ كؼخ‬ٟ‫ح‬ٝ
a. Rifampin ‫ش ٓـظٔؼش؟‬٤َُٜٗٞٓ‫ح‬
b. Penicillin ٖ٤‫لخٓز‬٣ٍ .a
c. Doxycycline ٖ٤ِٔ٘‫ حُز‬.b
d. Ciprofloxacin ٖ٤ٌِ٤ٔ٤ًٔٝ‫ ى‬.c
ٖ٤ٓ‫ًٔخ‬ِٞ‫ك‬َٝ‫ز‬٤ٓ .d

L‟utilisation de laquelle des drogues suivantes diminue le plus


clairement l‟efficacité contraceptive hormonale combinée?
a. Rifampicine
b. Pénicilline
c. Doxycycline
d. Ciprofloxacine
58. A 15 years old female patient presented with oligomenorrhea, she had ‫خرش‬ٜٓ ‫ ػخٓخ‬15 َٔ‫ش طزِؾ ٖٓ حُؼ‬٠٣َٓ
3 periods in the last year. She also had acne and hirsutism. Her body ّ‫ حُؼخ‬٢‫ كظَحص ك‬3 ‫خ‬ٜ٣‫ ًخٕ ُي‬،‫ر٘يٍس حُطٔغ‬
weight has increased to 80kg. Her TSH level normal. The most likely ‫خ كذ ٗزخد‬٠٣‫خ أ‬ٜ٣‫ ًخٕ ُي‬.٢ٟ‫حُٔخ‬
diagnosis is: .ْ‫ ًـ‬80 ٠ُ‫خ ا‬ٜٔٔ‫ُٕ ؿ‬ٝ ‫ حٍطلغ‬.‫ش‬٤ٗ‫ٗؼَح‬ٝ
a- Hyperprolactinemia َ‫ حألًؼ‬ٚ٤‫ حُظ٘و‬.٢‫ؼ‬٤‫ز‬١ TSH ٟٞ‫خ ٓٔظ‬ُٜ
b- Adrenal tumor :ٞٛ ‫لخ‬٤‫طَؿ‬
c- Premature ovarian failure ّ‫ٖ حُي‬٤‫الًظ‬َٝ‫ ر‬١َ‫ ك‬.a
d- Polycystic ovary disease ‫ش‬٣َ‫ٍّ حُـيس حٌُظ‬ٝ .b
ٌَ‫ ٓز‬ٞ٤‫ٍ ٓز‬ٜٞ‫ ه‬.c
‫ٔخص‬٤ٌُ‫ ٓظؼيى ح‬ٞ٤‫ ىحء حُٔز‬.d
Une patiente de 15 ans présentant une oligoménorrhée, elle a eu 3
règles au cours de la dernière année. Elle souffrait également d‟acné
et d‟hirsutisme. Son poids corporel est passé à 80 kg. Son taux de
TSH est normal. Le diagnostic le plus probable est :
a. Hyperprolactinémie
b. Tumeur surrénale
c. Insuffisance ovarienne prématurée
d. Maladie des ovaires polykystiques

28
59. A-38-year-old lady, G2P2, her routine Pap smear showed: Atypical ، G2P2 ،‫ ػخٓخ‬38 َٔ‫يس طزِؾ ٖٓ حُؼ‬٤ٓ
squamous cells cannot exclude High grade squamous ‫خ‬٣‫ هال‬:‫ش‬٤٘٤‫ط‬َُٝ‫َص ٓٔلش ػ٘ن حَُكْ ح‬ٜ‫أظ‬
intraepithelial lesion (ASC-H). What is the appropriate ‫ش‬٤‫ش ال طٔظزؼي آكش كَٗل‬٤‫ش ال ٗٔط‬٤‫كَٗل‬
management? ‫ ٓخ‬.)ASC-H( ‫خٍس‬ٜ‫ش حُيٍؿش ىحهَ حُظ‬٤ُ‫ػخ‬
‫َ حُٔ٘خٓذ؟‬٤‫ حُظير‬ٞٛ
a- HPV type testing ٢ٔ٤ِ‫ٍّ حُل‬ُٞ‫ّ ح‬َٝ٤‫ع ك‬ٞٗ ٍ‫ حهظزخ‬.a
b- Endometrial biopsy. ١َ٘‫حُز‬
c- Ultrasound of pelvis. .ْ‫ هِػش رطخٗش حَُك‬.b
d- Colposcopy. .ٝٞ‫ش ُِل‬٤‫ط‬ُٜٞ‫م ح‬ٞ‫ؿخص ك‬ُٞٔ‫ ح‬.c
e- CO-testing after 1 year. .٢ِ‫ز‬ُٜٔ‫َ ح‬٤‫ حُظ٘ظ‬.d
.‫ حهظزخٍ ٓ٘ظَى رؼي ٓ٘ش‬.e
Une femme de 38 ans, G2P2, son test Pap de routine a montré: Les
cellules malpighiennes atypiques ne peuvent pas exclure une lésion
malpighienne intraépithéliale de haut grade (ASC-H). Quelle est la
gestion appropriée?
a. Test de type VPH
b. Biopsie de l‟endomètre.
c. Échographie du bassin.
d. Colposcopie.
e. CO-test après 1 an.
60. Which of the following is true regarding the ovarian vessels? ‫؟‬ٞ٤‫ش حُٔز‬٤‫ػ‬ٝ‫ أ‬٠ِ‫٘طزن ػ‬٣ ٢ُ‫ ٖٓ حُظخ‬١‫أ‬
a. Both ovarian arteries arise from the aorta. ٖٓ ٞ٤‫ٖ حُٔز‬٤٣‫٘٘ؤ ًَ ٖٓ َٗح‬٣ .a
b. Both ovarian veins drain into the inferior vena cava. .١َٜ‫خٕ حألر‬٣َُ٘‫ح‬
c. Right and left ovarian arteries arise from the right renal ٢‫ذ ك‬ٜ‫ ط‬ٞ٤‫ٍىس حُٔز‬ٝ‫ ًَ ٖٓ أ‬.b
artery and aorta, respectively. .٢ِ‫ف حُٔل‬ٞ‫ي حألؿ‬٣ٍُٞ‫ح‬
d. Right and left ovarian veins drain into the right renal vein ٠٘ٔ٤ُ‫ ح‬ٞ٤‫ٖ حُٔز‬٤٣‫ ط٘٘ؤ َٗح‬.c
and inferior vena cava, respectively ١ٌُِٞ‫خٕ ح‬٣َُ٘‫ ٖٓ ح‬َٟٔ٤ُ‫ح‬ٝ
٠ِ‫ ػ‬،١َٜ‫خٕ حألر‬٣َُ٘‫ح‬ٝ ٖٔ٣‫حأل‬
Laquelle des affirmations suivantes est vraie en ce qui concerne les .٢ُ‫ح‬ٞ‫حُظ‬
vaisseaux ovariens? ٠٘ٔ٤ُ‫ ح‬ٞ٤‫ٍىس حُٔز‬ٝ‫ذ أ‬ٜ‫ ط‬.d
١ٌُِٞ‫ي ح‬٣ٍُٞ‫ ح‬٢‫ ك‬َٟٔ٤ُ‫ح‬ٝ
a. Les deux artères ovariennes proviennent de l‟aorte. ،٢ِ‫ف حُٔل‬ٞ‫ي حألؿ‬٣ٍُٞ‫ح‬ٝ ٖٔ٣‫حأل‬
b. Les deux veines ovariennes s‟écoulent dans la veine cave ٢ُ‫ح‬ٞ‫ حُظ‬٠ِ‫ػ‬
inférieure.
c. Les artères ovariennes droite et gauche proviennent
respectivement de l‟artère rénale droite et de l‟aorte.
d. Les veines ovariennes droite et gauche s‟écoulent
respectivement dans la veine rénale droite et la veine cave
inférieure

29
61. Your 25-year-old patient is pregnant at 36 weeks‟ gestation. She َٓ‫ ػخٓخ كخ‬25 َٔ‫ظي حُزخُـش ٖٓ حُؼ‬٠٣َٓ
has an acute urinary tract infection (UTI). Which of the following ٟٝ‫خ ػي‬ٜ٣‫ ُي‬.َٔ‫ ٖٓ حُل‬36 ‫ع‬ٞ‫ حألٓز‬٢‫ك‬
medications is contraindicated in the treatment of the UTI in this ٖٓ ١‫ أ‬.)UTI( ‫ش‬٤ُٞ‫ حُٔٔخُي حُز‬٢‫كخىس ك‬
patient? ‫ ػالؽ‬٢‫خى حٓظطزخد ك‬٠ٓ ٞٛ ‫ش‬٤ُ‫ش حُظخ‬٣ٝ‫حألى‬
a. Ampicillin ‫؟‬ٞ٣َُٔ‫ٌح ح‬ٛ ٢‫ش ك‬٤ُٞ‫خد حُٔٔخُي حُز‬ٜ‫حُظ‬
b. Nitrofurantoin ٖ٤ِ٤ٔ٤‫ أٓز‬.a
c. Trimethoprim/sulfamethoxazole ٖ٣ٞ‫ٍحٗظ‬ٞ‫ك‬َٝ‫ظ‬٤ٗ .b
d. Cephalexin ٍُٝ‫ًٔخ‬ٞ‫ؼ‬٤ٓ‫ ِٓلخ‬/ ْ٣َ‫ر‬ٞ‫ؼ‬٤ٔ٣َ‫ ط‬.c
e. Amoxicillin/clavulanate ٖ٤ٌٔ٤ُ‫لخ‬٤ٓ .d
‫الٗخص‬ٞ‫ ًالك‬/ ٖ٤ِ٤ٔ٤ًٔٞٓ‫ أ‬.e
Votre patiente de 25 ans est enceinte à 36 semaines de gestation. Elle
souffre d‟une infection aiguë des voies urinaires (IVU). Lequel des
médicaments suivants est contre-indiqué dans le traitement de
l‟infection urinaire chez ce patient?
a. Ampicilline
b. Nitrofurantoïne
c. Triméthoprime/sulfaméthoxazole
d. Céphalexine
e. Amoxicilline/clavulanate
62. A 52-year-old lady, menopause for the last 2 years, comes to your ‫ حٗوطخع‬، ‫ ػخٓخ‬52 َٔ‫يس طزِؾ ٖٓ حُؼ‬٤ٓ
clinic, complains of irregular vaginal bleeding for last month. Her ٢‫ طؤط‬، ٖ٤‫ظ‬٤ٟ‫ٖ حُٔخ‬٤‫ حُٔ٘ظ‬ٟ‫ ٓي‬٠ِ‫حُطٔغ ػ‬
pelvic sonography reveled 9mm endometrial thickness. What is the َ٤‫ ؿ‬٢ِ‫ز‬ٜٓ ‫ق‬٣ِٗ ٖٓ ٌٞ٘‫ط‬ٝ ، ‫خىطي‬٤‫ ػ‬٠ُ‫ا‬
most appropriate step? َ٣ٜٞ‫ ً٘ق حُظ‬.٢ٟ‫َ حُٔخ‬ُٜ٘‫ٓ٘ظظْ ح‬
ٖ‫ ػ‬ٝٞ‫ حُل‬٢‫ش ك‬٤‫ط‬ُٜٞ‫م ح‬ٞ‫ؿخص ك‬ُٞٔ‫رخ‬
a. MRI pelvis ‫س‬ٞ‫ حُوط‬٢ٛ ‫ ٓخ‬.ْٓ 9 ْ‫ٓٔخًش رطخٗش ٍك‬
b. Endometrial biopsy, ‫حألٗٔذ؟‬
c. Pap smear ٢ٔ٤١‫ٖ حُٔـ٘خ‬٤َُٗ‫َ رخ‬٣ٜٞ‫ حُظ‬.a
d. Repeat sonography after one month ٝٞ‫ُِل‬
e. Hysterectomy. ْ‫ هِػش رطخٗش ٍك‬.b
ْ‫ ٓٔلش ػ٘ن حَُك‬.c
‫م‬ٞ‫ؿخص ك‬ُٞٔ‫َ رخ‬٣ٜٞ‫ طٌَحٍ حُظ‬.d
Une dame de 52 ans, ménopausée depuis 2 ans, vient à votre clinique, ‫حكي‬ٝ َٜٗ ‫ش رؼي‬٤‫ط‬ُٜٞ‫ح‬
se plaint de saignements vaginaux irréguliers depuis un mois. Son ْ‫خٍ حَُك‬ٜ‫ حٓظج‬.e
échographie pelvienne a révélé une épaisseur endométriale de 9 mm.
Quelle est l‟étape la plus appropriée?
a. IRM du bassin
b. Biopsie de l‟endomètre,
c. Pap test
d. Répéter l‟échographie après un mois
e. Hystérectomie.

30
63. Which of the following is not a parameter of the Bishop score? ‫د؟‬ٞ٘٤‫خّ ُلَُ ر‬٤‫ْ ٓو‬٤ُ ٢ِ٣ ‫ ٓٔخ‬١‫أ‬
a. Cervical dilation ْ‫ طٔيى ػ٘ن حَُك‬.a
b. Cervical effacement ْ‫ ػ٘ن حَُك‬ٞ‫ ٓل‬.b
c. Fetal presentation ٖ٤٘‫ى حُـ‬ٞ‫ؿ‬ٝ .c
ٖ٤٘‫هغ حُـ‬ٞٓ .d
d. Fetal station
ْ‫غ ػ٘ن حَُك‬ٟٞٓ .e
e. Cervical position

Lequel des éléments suivants n‟est pas un paramètre du score Bishop?


a. Dilatation cervicale
b. Effacement cervical
c. Présentation fœtale
d. Station fœtale
e. Position cervicale
64. A 27-year-old multiparous woman had spontaneous rupture of ‫ ػخٓخ‬27 َٔ‫الىس طزِؾ ٖٓ حُؼ‬ُٞ‫حَٓأس ٓظؼيىس ح‬
membranes at 37 weeks of gestation. She is a carrier of group B ٢‫ش ك‬٤٘‫ ُألؿ‬٢‫ ٖٓ طِٔم طِوخث‬٢ٗ‫ًخٗض طؼخ‬
Streptococcus (GBS) as detected on a vaginal swab in the second ‫ كخِٓش‬٢ٛ .َٔ‫ ٖٓ حُل‬37 ‫ع‬ٞ‫حألٓز‬
trimester. What is the most appropriate management? )GBS( ‫ػش د‬ٞٔ‫ش ٖٓ حُٔـ‬٣‫ٍحص حُؼوي‬ٌُِٞٔ
a. Await spontaneous labour and use intrapartum ‫ حُؼِغ‬٢‫ش ك‬٤ِ‫ز‬ٜٓ ‫ ٓٔلش‬٢‫خ ك‬ٜ‫هي طْ حًظ٘خك‬
antibiotic prophylaxis (IAP) ‫َحألٗٔذ؟‬٤‫ حُظير‬ٞٛ ‫ ٓخ‬.َٔ‫ ٖٓ حُل‬٢ٗ‫حُؼخ‬
b. Offer one dose of prophylactic antibiotic before ّ‫حٓظويح‬ٝ ٢‫ حُظِوخث‬ٝ‫ حٗظظخٍ حُٔوخ‬.a
elective caesarean delivery ‫خىحص‬٠ُٔ‫ رخ‬٢‫هخث‬ُٞ‫حُؼالؽ ح‬
c. Offer induction of labour after 24 hours and IAP )IAP( ‫الىس‬ُٞ‫ش أػ٘خء ح‬٣ٞ٤‫حُل‬
d. Offer induction of labour at 41 weeks and IAP ٖٓ ‫حكيس‬ٝ ‫ـذ اػطخء ؿَػش‬٣ .b
e. Offer induction of labour immediately and IAP ‫الىس‬ُٞ‫ هزَ ح‬٢‫هخث‬ٝ ١ٞ٤‫خى ك‬٠ٓ
‫ش‬٣ٍ‫خ‬٤‫ش حالهظ‬٣َٜ٤‫حُو‬
24 ‫ رؼي‬ٝ‫ حُٔوخ‬ٞ٣َ‫ اؿَحء طل‬.c
‫ أػ٘خء‬٢‫هخث‬ٝ ١ٞ٤‫خى ك‬٠ٓٝ ‫ٓخػش‬
Une femme multipare de 27 ans a eu une rupture spontanée des IAP ‫الىس‬ُٞ‫ح‬
membranes à 37 semaines de gestation. Elle est porteuse du ٢‫ك‬ ٝ‫حُٔوخ‬ ٞ٣َ‫طل‬ ‫ اؿَحء‬.d
٢‫هخث‬ٝ ١ٞ٤‫خى ك‬٠ٓٝ 41 ‫ع‬ٞ‫حألٓز‬
streptocoque du groupe B (SGB) détecté sur un écouvillon vaginal au
IAP ‫الىس‬ُٞ‫أػ٘خء ح‬
cours du deuxième trimestre. Quelle est la gestion la plus appropriée?
٠ِ‫ ػ‬ٝ‫ حُٔوخ‬ٞ٣َ‫ اؿَحء طل‬.e
a. Attendre le travail spontané et utiliser l‟antibioprophylaxie ‫أػ٘خء‬ ٢‫هخث‬ٝ ١ٞ٤‫خى ك‬٠ٓٝ ٍٞ‫حُل‬
intrapartum (PAI) IAP ‫الىس‬ُٞ‫ح‬
b. Offrir une dose d‟antibiotique prophylactique avant
l‟accouchement par césarienne élective
c. Offrir le déclenchement du travail après 24 heures et le PEI
d. Offrir le déclenchement du travail à 41 semaines et le PEI
e. Offrir le déclenchement du travail immédiatement et le PEI

31
A 65.
periA pericardial friction rub is commonly associated with: :‫ ٓغ‬١ٍٞٓ‫ظَحكن حإلكظٌخى حُظخ‬٣ ‫ػخىس ٓخ‬
a- Uremia ‫ش‬٣ٞٓ‫ِش ى‬٤‫ ر‬.a
b- Hypertension ّ‫ حُي‬٢‫ـ‬ٟ ‫ حٍطلخع‬.b
c- Cardiac hypertrophy ‫وْ حُوِذ‬٠‫ ط‬.c
d- Mitral stenosis ٢‫ٔخّ حُظخؿ‬ُٜ‫ن ح‬٤٠‫ ط‬.d

Le frottement péricardique péricardique est généralement associé à:


a. Urémie
b. Hypertension
c. Hypertrophie cardiaque
d. Sténose mitrale
3. 66.
The Earliest sign of pathological gastroesophageal reflux in infants is: ٢‫ج‬٣َُٔ‫ ح‬١‫ حالٍطـخع حُٔؼي‬٠ِ‫ٍ ػالٓش ػ‬ٝ‫أ‬
a- Upper bleeding :٢ٛ ‫غ‬َُٟ‫ ػ٘ي ح‬٢َُٟٔ‫ح‬
b- Respiratory symptoms ١ِٞ‫ق ػ‬٣ِٗ .a
c- Oesophageal stricture ‫ش‬٤ٔ‫ ط٘ل‬ٝ‫ أػَح‬.b
d- Postprandial regurgitation ‫ء‬١َُٔ‫ن ح‬٤٠‫ ط‬.c
ًَ‫ هِْ ٓخ رؼي حأل‬.d

Le premier signe de reflux gastro-œsophagien pathologique chez les


nourrissons est:
a. Saignement supérieur
b. Symptômes respiratoires
c. Rétrécissement œsophagien
d. Régurgitation postprandiale

67. A child with recurrent urinary tract infections is most likely to show: ‫خد‬ُٜٔ‫َ حُطلَ ح‬ٜ‫ظ‬٣ ٕ‫ٖٓ حَُٔؿق أ‬
:‫ش حُٔظٌٍَس‬٤ُٞ‫خرخص حُٔٔخُي حُز‬ٜ‫رخُظ‬
a. Posterior urethral valves ‫ش‬٤‫ش هِل‬٤ِ٤ِ‫ٔخٓخص اك‬ٛ .a
b. Vesico-ureteric reflux ٢‫ كخُز‬٢ٗ‫ ؿٍِ ٓؼخ‬.b
c. Neurogenic bladder ‫ش‬٤‫ز‬ٜ‫ ٓؼخٗش ػ‬.c
d. Renal and ureteric calculi ‫ش‬٤‫كخُز‬ٝ ‫ش‬٣ًِٞ ‫خحص‬٤ٜ‫ ك‬.d

Un enfant atteint d‟infections urinaires récurrentes est le plus


susceptible de présenter:
a. Valves urétrales postérieures
b. Reflux vésico-urétéral
c. Vessie neurogène
d. Calculs rénaux et urétéraux

32
68. The coagulation profile in a 13-year old girl with Menorrhagia having 13 َٔ‫ِٓق طوؼَحُيّ ػ٘ي كظخس طزِؾ ٖٓ حُؼ‬
von Willebrand‟s disease is: ‫ ٖٓ ىحء‬٢ٗ‫طؼخ‬ٝ ‫خرش رـِحٍس حُطٔغ‬ٜٓ ‫ػخٓخ‬
:ٞٛ ‫ِزَحٗي‬٣ٝ ٕٞ‫ك‬
a. Isolated prolonged PTT with a normal PT ‫ٍ ٓغ‬ِٝ‫ حُٔؼ‬PTT٢‫خُش ك‬١‫ ا‬.a
b. Isolated prolonged PT with a normal PTT ٢‫ؼ‬٤‫ز‬١ PT
c. Prolongation of both PT and PTT ‫ٍ ٓغ‬ِٝ‫ حُٔؼ‬PT ٢‫خُش ك‬١‫ ا‬.b
d. Prolongation of thrombin time ٢‫ؼ‬٤‫ز‬١ PTT
PT ٖٓ ًَ ٢‫خُش ك‬١‫ ا‬.c
PTTٝ
Le profil de coagulation chez une fille de 13 ans atteinte de ٖ٤‫ٓز‬َٝ‫ ُٖٓ حُؼ‬٢‫خُش ك‬١‫ ا‬.d
ménorragie ayant la maladie de von Willebrand est:
a. PTT prolongé isolé avec un PT normal
b. PT prolongé isolé avec un PTT normal
c. Allongement du PT et du PTT
d. Prolongation du temps de thrombine
69. A 6-year-old is scheduled to have an adenoidectomy. What is the ٖٓ ‫غ حُطلَ حُزخُؾ‬٠‫و‬٣ ٕ‫ٖٓ حُٔوٍَ أ‬
most common reasons for this procedure? ٢ٛ ‫ ٓخ‬.‫خص‬٤ٗ‫خٍ حُـيح‬ٜ‫حص الٓظج‬ٞ٘ٓ 6َٔ‫حُؼ‬
‫ٌح حإلؿَحء؟‬ُٜ ‫ػخ‬ٞ٤ٗ َ‫حألٓزخد حألًؼ‬
a. Enlargement of the adenoids ‫ش‬٤‫خص حألٗل‬٤ٗ‫وْ حُـيح‬٠‫ ط‬.a
b. The child has recurrent Streptococcal tonsillitis and ٖ٤‫ُط‬ُِٞ‫خد ح‬ٜ‫ حُطلَ ٖٓ حُظ‬٢ٗ‫ؼخ‬٣ .b
is having tonsillectomy. The adenoidectomy is ‫غ‬٠‫و‬٣ٝ ٌٍَ‫ش حُٔظ‬٣‫حُؼوي‬
included preventively ٖ٤ٔ٠‫ظْ ط‬٣ .ٖ٤‫ُط‬ُِٞ‫خٍ ح‬ٜ‫الٓظج‬
c. The child has had recurrent middle ear infections. ٢‫هخث‬ٝ ٌَ٘‫خص ر‬٤ٗ‫خٍ حُـيح‬ٜ‫حٓظج‬
Surgery is to remove blockage of the Eustachian ‫خرخص‬ٜ‫ ٖٓ حُظ‬٢ٗ‫ؼخ‬٣ َ‫ ًخٕ حُطل‬.c
tubes. .‫ حُٔظٌٍَس‬٠‫ٓط‬ُٞ‫حألًٕ ح‬
d. The parents request the procedure َ٤‫ إلُحُش حٗٔيحى حُ٘ل‬٢ٛ ‫حُـَحكش‬
‫ٌح حإلؿَحء‬ٛ ٕ‫حُيح‬ُٞ‫ِذ ح‬١ ‫ ُوي‬.d
Un enfant de 6 ans doit subir une adénoïdectomie. Quelles sont les
raisons les plus courantes de cette procédure?
a. Élargissement des végétations adénoïdes
b. L‟enfant a une amygdalite streptococcique récurrente et subit
une amygdalectomie. L‟adénoïdectomie est incluse
préventivement
c. L‟enfant a eu des infections récurrentes de l‟oreille moyenne.
La chirurgie consiste à enlever le blocage des trompes
d‟Eustache.
d. Les parents demandent la procédure

33
70. A 3-year-old boy with normal developmental milestones has delayed ٞٔٗ ُْ‫ ٓؼخ‬ًٝ ‫حص‬ٞ٘ٓ 3 َٔ‫زِؾ ٖٓ حُؼ‬٣ ٢‫ز‬ٛ
speech and difficulty in communication and concentration. He is not ٢‫رش ك‬ٞ‫ؼ‬ٛٝ ّ‫ حٌُال‬٢‫ طؤهَ ك‬ٚ٣‫ش ُي‬٤‫ؼ‬٤‫ز‬١
making friends. Most probable diagnosis is: .‫يحهخص‬ٛ ‫٘غ‬ٜ٣ ‫ ال‬ٚٗ‫ ا‬.ِ٤ًَ‫حُظ‬ٝ َٛ‫ح‬ٞ‫حُظ‬
:ٞٛ ‫لخ‬٤‫ حألًؼَ طَؿ‬ٚ٤‫حُظ٘و‬
a. Autism ‫كي‬ٞ‫ حُظ‬.a
b. Mental retardation ٢ِ‫ حُظوِق حُؼو‬.b
c. Specific learning disability ‫ش ٓليىس‬٤ٔ٤ِ‫رخص طؼ‬ٞ‫ؼ‬ٛ .c
d. None of the above ‫لش‬٤‫ل‬ٛ َ٤‫ ؿ‬ٙ‫رش أػال‬ٞ‫ ًَ حألؿ‬.d
Un garçon de 3 ans ayant des étapes normales de développement a un
retard d‟élocution et des difficultés de communication et de
concentration. Il ne se fait pas d‟amis. Le diagnostic le plus probable
est :
a. Autisme
b. Retard mental
c. Trouble d‟apprentissage spécifique
d. Aucune des réponses ci-dessus
71. Which autoimmune disease is characterized by the absence of T-cells, ِ٤ٔ‫ظ‬٣ ١ٌُ‫ ح‬٢٤‫ حٌُحط‬٢‫ حُٔ٘خػ‬َُٝٔ‫ ح‬ٞٛ ‫ٓخ‬
tetany, and hypocalcemia, with lowered cell-mediated immunity? ٚ‫ٗو‬ٝ ٌُِ‫حُظ‬ٝ ‫ش‬٤‫خ حُظخث‬٣‫خد حُوال‬٤‫رـ‬
‫ حُٔ٘خػش‬٢‫ ك‬ٝ‫ ٓغ حٗولخ‬،ّ‫ّ حُي‬ٞ٤ُٔ‫ًخ‬
a. Ulcerative colitis ‫ش؟‬٣ِٞ‫حٓطش حُو‬ٞ‫حُٔظ‬
b. Multiple sclerosis ٢‫ٕ حُظوَك‬ُٞٞ‫خد حُو‬ٜ‫ حُظ‬.a
c. Congenital thymic aplasia ٢‫ل‬٣ُِِٞ‫ِذ ح‬ٜ‫ حُظ‬.b
d. Systemic lupus erythematosus ٢‫ش حُوِو‬٣َ‫ؼظ‬ُٜ‫ ػيّ ط٘ٔؾ حُـيس ح‬.c
‫ش‬٣ُ‫خ‬ٜ‫ش حُـ‬٤ٓ‫ حٌُثزش حُلٔخ‬.d

Quelle maladie autoimmune est caractérisée par l‟absence de


lymphocytes T, de tétanie et d‟hypocalcémie, avec une immunité à
médiation cellulaire réduite?
a. Colite ulcéreuse
b. Sclérose en plaques
c. Aplasie thymique congénitale
d. Lupus érythémateux disséminé

34
72. Atopic dermatitis usually involves: :٢ِ٣ ‫ ٓخ‬٢‫خد حُـِي حُظؤطز‬ٜ‫ٖٔ حُظ‬٠‫ظ‬٣ ‫ػخىس ٓخ‬
ٌَ٘‫ُع ر‬ٞٓ‫ ٓغ كٌش‬١‫لق ؿِي‬١ .a
a. An itchy symmetrically distributed rash َ‫ٓظ٘خظ‬
b. An elevated IgE level IgE ٟٞ‫ حٍطلخع ٓٔظ‬.b
c. Cutaneous infection with staph aureus ‫ش‬٣‫ى‬ٞ‫ٍحص حُؼ٘و‬ٌُٞٔ‫ رخ‬١‫ حٗظخٕ ؿِي‬.c
d. A dry skin ‫ش‬٤‫ز‬ٌُٛ‫ح‬
e. All of the above ‫ ؿلخف حُـِي‬.d
‫لش‬٤‫ل‬ٛ ٙ‫رش أػال‬ٞ‫ ًَ حألؿ‬.e

La dermatite atopique implique habituellement:


a. Une éruption cutanée répartie symétriquement démangeaisons
b. Un taux élevé d‟IgE
c. Infection cutanée par staphylocoque doré
d. Une peau sèche
e. Tout ce qui précède
73. A newborn presents with Hydrocephalus, intracranial calcifications ،ّ‫الىس ٖٓ حٓظٔوخء حَُأ‬ٝ ‫غ‬٣‫ كي‬٢ٗ‫ؼخ‬٣
and chorioretinitis. Your diagnosis is: ‫ش‬٤ٔ٤ُ٘ٔ‫خد ح‬ٜ‫حُظ‬ٝ ‫طٌِٔخص ىحهَ حُـٔـٔش‬
:ٞٛ ‫ي‬ٜ٤‫ ط٘و‬.‫ش‬٤ٌ‫حُ٘ز‬ٝ
a. Neonatal Herpes ١‫ي‬٤ُُٞ‫َرْ ح‬ُٜ‫ ح‬.a
b. Congenital Toxoplasmosis ٢‫ٓخص حُوِو‬ٞ‫ ىحء حُٔو‬.b
c. Congenital Syphilis ٢‫ حُوِو‬١َُِٛ‫ ح‬.c
d. Congenital Varicella ٢‫ حُلٔخم حُوِو‬.d

Un nouveau-né présente une hydrocéphalie, des calcifications


intracrâniennes et une choriorétinite. Votre diagnostic est le suivant :
a. Herpès néonatal
b. Toxoplasmose congénitale
c. Syphilis congénitale
d. Varicelle congénitale
74. Proper Treatment of Purulent Pericarditis requires: ٍٞٓ‫خد حُظخ‬ٜ‫ظطِذ حُؼالؽ حُٔ٘خٓذ الُظ‬٣
:٢‫ل‬٤‫حُو‬
a. Corticosteroids ‫ش‬٣َ٘‫يحص حُو‬٣َٝ٤‫ حُٔظ‬.a
b. Aspirin ٖ٣َ‫ حألٓز‬.b
c. Intravenous immunoglobulin ١‫ي‬٣ٍُٞ‫ ح‬٢‫ٖ حُٔ٘خػ‬٤ُٞ‫ر‬ِٞ‫ حُـ‬.c
d. Surgical Drainage ٢‫ق حُـَحك‬٣َٜ‫ حُظ‬.d

Le traitement approprié de la péricardite purulente nécessite:


a. Corticostéroïdes
b. Aspirine
c. Immunoglobuline intraveineuse
d. Drainage chirurgical

35
75. Causes of Abdominal Neonatal Occlusion, (1 false): 1( ،‫الىس‬ُٞ‫ ح‬٢‫ؼ‬٣‫أٓزخد حٗٔيحى حُزطٖ ُلي‬
:)‫هطؤ‬
a. Small intestines atresia ‫وش‬٤‫ ٍطن حألٓؼخء حُيه‬.a
b. Esophageal atresia ‫ء‬١َُٔ‫ ٍطن ح‬.b
c. Hirschsprung ‫َٗزَٗؾ‬٤ٛ ‫ ىحء‬.c
d. Meconial plug ‫ش‬٤٤‫ ٓيحىس ػو‬.d

Causes de l‟occlusion néonatale abdominale, (1 faux):


a. Atrésie de l‟intestin grêle
b. Atrésie de l‟œsophage
c. Hirschsprung
d. Bouchon méconial
76. In the case of unruptured ectopic pregnancy (a correct answer) ‫َ ٓظِٔم‬٤‫ كخُش كَٔ هخٍؽ حَُكْ ؿ‬٢‫ك‬
a. Laparoscopy is necessary and urgent )‫لش‬٤‫ل‬ٛ ‫(اؿخرش‬
b. Methotrexate can be a treatment even with intraperitoneal َ‫ػخؿ‬ٝ ١ٍَٟٝ ٖ‫َ حُزط‬٤‫ ط٘ظ‬.a
fluid effusion ‫ٌٔخص‬٣َ‫ط‬ٞ‫ؼ‬٤ُٔ‫ٕ ح‬ٌٞ٣ ٕ‫ٌٖٔ أ‬٣ .b
c. The Bhcg allows the follow-up of the conservative treatment َ‫زخد حُٔخث‬ٜٗ‫ ٓغ ح‬٠‫ػالؿخ كظ‬
d. Folic acid is an assistant to Methotrexate ‫لخم‬ُٜ‫ىحهَ ح‬
e. Laparotomy is the only surgical approach. ‫ رٔظخرؼش حُؼالؽ‬Bhcg ‫ٔٔق‬٣ .c
‫حُٔلخكع‬
‫ ٓٔخػي‬ٞٛ ‫ي‬٤ُٞ‫ حُل‬ٞٔ‫ ك‬.d
En cas de grossesse extra-utérine non rompue (une bonne réponse) ‫ٌٔخص‬٣َ‫ط‬ٞ‫ؼ‬٤ُِٔ
a. La laparoscopie est nécessaire et urgente ٢‫ؾ حُـَحك‬ُٜ٘‫ ح‬ٞٛ ٖ‫ كظق حُزط‬.e
b. Le méthotrexate peut être un traitement même avec un .‫ي‬٤‫ك‬ُٞ‫ح‬
épanchement de liquide intrapéritonéal
c. Le Bhcg permet le suivi du traitement conservateur
d. L‟acide folique est un assistant du méthotrexate
e. La laparotomie est la seule approche chirurgicale.

36
77. A pregnant lady at 30 weeks presents with vaginal bleeding ‫ق‬٣ِٗ ٖٓ 30 ‫ع‬ٞ‫ حألٓز‬٢‫يس كخَٓ ك‬٤ٓ ٢ٗ‫طؼخ‬
and frequent and intense uterine contractions after a road ‫ٌٓؼلش رؼي‬ٝ ‫خص ٍكْ ٓظٌٍَس‬ِٜ‫طو‬ٝ ٢ِ‫ز‬ٜٓ
accident. Late decelerations on fetal monitoring are present ًَ ‫ى ٓغ‬ٞ‫ؿ‬َٞٓ‫ئ حُٔظؤه‬١‫ حُظزخ‬.‫ن‬٣َ١ ‫كخىع‬
with each contraction. Which quote is the most valid? ٢ٛ ‫ اؿخرش‬١‫ أ‬.ٖ٤٘‫ ػ٘ي َٓحهزش حُـ‬ٝ‫حٗوزخ‬
a. Use betamimetics to treat preterm labor ‫لش؟‬ٛ َ‫حألًؼ‬
b. Do an emergency caesarean section ‫ظخ ُؼالؽ‬٤‫ حٓظويحّ ٓوِيحص ر‬.a
c. Use a combined peri-spinal ٌَ‫ حُٔز‬ٝ‫حُٔوخ‬
d. It is unlikely that the patient will develop DIC ‫خٍثش‬١ ‫ش‬٣َٜ٤‫ش ه‬٤ِٔ‫ اؿَحء ػ‬.b
e. It is an acute abdomen ١َ‫ى حُلو‬ٞٔ‫ٍ حُؼ‬ٞ‫ حٓظويحّ ك‬.c
‫ٓ٘ظَى‬
‫خد‬ٜ‫َ حُٔلظَٔ إٔ ط‬٤‫ ٖٓ ؿ‬.d
Une femme enceinte à 30 semaines présente des saignements َ‫ش رظوؼَ ٓ٘ظؼَ ىحه‬٠٣َُٔ‫ح‬
vaginaux et des contractions utérines fréquentes et intenses après un ‫ش‬٤‫ػ‬ٝ‫حأل‬
accident de la route. Des décélérations tardives sur la surveillance ‫ رطٖ كخى‬ٚٗ‫ ا‬.e
fœtale sont présentes à chaque contraction. Quelle citation est la plus
valable?
a. Utiliser des bêtamimétiques pour traiter le travail prématuré
b. Faire une césarienne d‟urgence
c. Utiliser une péri-colonne vertébrale combinée
d. Il est peu probable que la patiente développe une CIVD
e. Il s‟agit d‟un abdomen aigu

37
78. A 56-year-old woman with no particular history presents with ‫ن‬٣ٍ‫خ طخ‬ُٜ ْ٤ُ ‫ ػخٓخ‬56 َٔ‫حَٓأس طزِؾ ٖٓ حُؼ‬
recent abdominal bloating. Ultrasound confirms the presence ‫ طئًي‬.‫ ٖٓ حٗظلخم حُزطٖ ٓئهَح‬٢ٗ‫ٖ طؼخ‬٤‫ٓؼ‬
of ascites as well as a 15 cm Dt ovarian cyst, with thick walls, ‫ى حٓظٔوخء‬ٞ‫ؿ‬ٝ ‫ش‬٤‫ط‬ُٜٞ‫م ح‬ٞ‫ؿخص ك‬ُٞٔ‫ح‬
with multiple intracystic vegetations of 2 to 3 cm each. The ، ْٓ 15 َ‫ روط‬ٞ٤‫ْ ٓز‬٤ً ٠ُ‫خكش ا‬ٟ‫رخإل‬
most logical therapeutic proposal among the following would َ‫حرض ٓظؼيىس ىحه‬ٞٗ ‫ ٓغ‬، ‫ٌش‬٤ٔٓ ٕ‫ٓغ ؿيٍح‬
be: ‫ حالهظَحف‬.‫خ‬ٜ٘ٓ ٌَُ ْٓ 3 ٠ُ‫ ا‬2 ٖٓ ْ٤ٌُ‫ح‬
a. Right sided annexectomy by laparoscopy then :ٞٛ ٢ِ٣ ‫ٖ ٓخ‬٤‫ش ٖٓ ر‬٤‫ حألًؼَ ٓ٘طو‬٢‫حُؼالؿ‬
chemotherapy ٖ‫ٖٔ ػ‬٣‫خٍ حُِٔلن حأل‬ٜ‫ حٓظج‬.a
b. Unilateral ovariectomy by laparoscopy followed by ‫َ حُزطٖ ػْ حُؼالؽ‬٤‫ن ط٘ظ‬٣َ١
chemotherapy ٢‫خث‬٤ٔ٤ٌُ‫ح‬
c. Chemotherapy followed by laparoscopy ‫حكي‬ٝ ‫ ٖٓ ؿخٗذ‬ٞ٤‫خٍ حُٔز‬ٜ‫ حٓظج‬.b
d. Hysterectomy with double adnexectomy and ٚ٤ِ٣ ٖ‫َ حُزط‬٤‫ن ط٘ظ‬٣َ١ ٖ‫ػ‬
omentectomy and surgical staging followed by adjuvant ٢‫خث‬٤ٔ٤ٌُ‫حُؼالؽ ح‬
chemotherapy. ٖ‫َ حُزط‬٤‫ ط٘ظ‬ٚ٤ِ٣ ٢‫خث‬٤ٔ٤ٌُ‫ حُؼالؽ ح‬.c
e. Neoadjuvant chemotherapy followed by hysterectomy ٍ‫خ‬ٜ‫خٍ حَُكْ ٓغ حٓظج‬ٜ‫ حٓظج‬.d
with double adnexectomy and omentectomy and surgical ‫هطغ حُؼَد‬ٝ ‫ؽ‬ٝ‫حُِٔلوخص حُِٔى‬
staging ‫ حُؼالؽ‬ٚ٤ِ٣ ٢‫ؾ حُـَحك‬٣ٍ‫حُظي‬ٝ
.‫ حُٔٔخػي‬٢‫خث‬٤ٔ٤ٌُ‫ح‬
‫ي‬٣‫ حُٔٔخػي حُـي‬٢‫خث‬٤ٔ٤ٌُ‫ حُؼالؽ ح‬.e
Une femme de 56 ans sans antécédents particuliers présente des ٍ‫خ‬ٜ‫خٍ حَُكْ ٓغ حٓظج‬ٜ‫ حٓظج‬ٚ٤ِ٣
ballonnements abdominaux récents. L‟échographie confirme la ‫هطغ حُؼَد‬ٝ ‫ؽ‬ٝ‫حُِٔلوخص حُِٔى‬
présence d‟ascite ainsi que d‟un kyste ovarien Dt de 15 cm, aux parois ٢‫ؾ حُـَحك‬٣ٍ‫حُظي‬ٝ
épaisses, avec de multiples végétations intrakystiques de 2 à 3 cm
chacune. La proposition thérapeutique la plus logique parmi les
suivantes serait :
a. Annexe droite par laparoscopie puis chimiothérapie
b. Ovariectomie unilatérale par laparoscopie suivie d‟une
chimiothérapie
c. Chimiothérapie suivie d‟une laparoscopie
d. Hystérectomie avec double annexectomie et omentectomie et
stadification chirurgicale suivie d‟une chimiothérapie
adjuvante.
e. Chimiothérapie néoadjuvante suivie d‟une hystérectomie avec
double annexexectomie et omentectomie et stadification
chirurgicale

38
79. The most common cause of bleeding in the first trimester is َٜٗ‫ حأل‬٢‫ق ك‬٣ُِِ٘ ‫ػخ‬ٞ٤ٗ َ‫حُٔزذ حألًؼ‬
a. Hydatidiform mole ٞٛ ٠ُٝ‫حُؼالػش حأل‬
b. Miscarriage ‫ش‬٣ٍ‫ ػيح‬٠‫ ٍك‬.a
c. Cervical polyp ٝ‫خ‬ٜ‫ حإلؿ‬.b
d. Vaginal infection ‫ش‬٤‫ِش ػ٘و‬٤ِٓ .c
e. E-GEU ٢ِ‫ز‬ٜٓ ٕ‫ حٗظخ‬.d
E-GEU .e
La cause la plus fréquente de saignement au cours du premier
trimestre est
a. la taupe hydatiforme
b. Fausse couche
c. Polype cervical
d. Infection vaginale
e. E-GEU
80. Regarding the effects of the pill: :َٔ‫د ٓ٘غ حُل‬ٞ‫َ كز‬٤‫ظؼِن رظؤػ‬٣ ‫ٔخ‬٤‫ك‬
a. It increases the risk of ovarian cancer ٕ‫خ‬١َٔ‫خرش ر‬ٛ‫ي ٖٓ هطَ حإل‬٣ِ‫ ط‬.a
b. It increases the risk of endometrial cancer ٞ٤‫حُٔز‬
c. It decreases the amount of blood during menstruation ٕ‫خ‬١َٔ‫خرش ر‬ٛ‫ي ٖٓ هطَ حإل‬٣ِ‫ ط‬.b
d. It increases dysmenorrhea ْ‫رطخٗش حَُك‬
e. It decreases the risk of phlebitis ٞ٤‫ش حُيّ أػ٘خء حُل‬٤ًٔ ٖٓ َِ‫ طو‬.c
‫ي ٖٓ ػَٔ حُطٔغ‬٣ِ‫ ط‬.d
‫ي‬٣ٍُٞ‫خد ح‬ٜ‫ طوَِ ٖٓ هطَ حُظ‬.e
En ce qui concerne les effets de la pilule:
a. Il augmente le risque de cancer de l‟ovaire
b. Il augmente le risque de cancer de l‟endomètre
c. Il diminue la quantité de sang pendant la menstruation
d. Il augmente la dysménorrhée
e. Il diminue le risque de phlébite
81. Vaginal lubrication during intercourse comes mainly from ٌَ٘‫ أػ٘خء حُـٔخع ر‬٢ِ‫ز‬ُٜٔ‫ن ح‬٤ُِ‫ حُظ‬٢‫ؤط‬٣
a. Partner‟s secretions ٖٓ ٢ٔ٤‫ٍث‬
b. Bartholin gland ‫ي‬٣َُ٘‫ اكَحُحص ح‬.a
c. Urine ٖ٤ُٞ‫ ؿيس رخٍػ‬.b
d. Transudation through the wall of the vagina ٍٞ‫ حُز‬.c
e. Cervical glands under the influence of estrogen َ‫ز‬ُٜٔ‫ حٍط٘خف ػزَ ؿيحٍ ح‬.d
َ٤‫ ؿيى ػ٘ن حَُكْ طلض طؤػ‬.e
ٖ٤‫ؿ‬َٝ‫ٕ حالٓظ‬َٞٓٛ
La lubrification vaginale pendant les rapports sexuels provient
principalement des
a. Sécrétions du partenaire
b. Glande de Bartholin
c. Urinaire
d. Transsudation à travers la paroi du vagin
e. Glandes cervicales sous l‟influence de l‟œstrogène

39
82. What are the signs that predict the imminence of an eclampsia in a ‫ طظ٘زؤ روَد حالٍطؼخؽ‬٢‫ حُؼالٓخص حُظ‬٢ٛ ‫ٓخ‬
hypertensive pregnant woman? ٢‫ـ‬ٟ ‫خرش رخٍطلخع‬ُٜٔ‫ حَُٔأس حُلخَٓ ح‬ٟ‫ُي‬
a. Abolition of deep tendon reflexes ‫حُيّ؟‬
b. Ringing in the ears ‫وش‬٤ٔ‫ش حُؼ‬٣َ‫ط‬ُٞ‫ى حُلؼَ ح‬ٝ‫ اُـخء ٍى‬.a
c. Weight gain of 4 kg in three days ٖ٤ًٗ‫ٖ حأل‬٤٘١ .b
d. Calf cramps ّ‫خ‬٣‫ ػالػش أ‬٢‫ ًـْ ك‬4 ُُٕٞ‫خىس ح‬٣ُ .c
‫ ٍرِش حُٔخم‬٢‫ ط٘٘ـخص ك‬.d
Quels sont les signes qui prédisent l‟imminence d‟une éclampsie chez
une femme enceinte hypertendue?
a. Abolition des réflexes tendineux profonds
b. Bourdonnements dans les oreilles
c. Prise de poids de 4 kg en trois jours
d. Crampes au mollet

83. For the following parameters choose the one that is increased during ‫ طِىحى أػ٘خء‬٢‫ حهظَ حُظ‬،‫ش‬٤ُ‫َ حُظخ‬٤٣‫رخُ٘ٔزش ُِٔؼخ‬
the evolution of the pregnancy َٔ‫ٍ حُل‬ٞ‫طط‬
a. Hematocrit ‫ض‬٣ًَٞ‫ٔخط‬٤ُٜ‫ ح‬.a
b. Systolic blood pressure ٢ٟ‫ حُيّ حالٗوزخ‬٢‫ـ‬ٟ .b
c. Diastolic blood pressure ٢١‫ حُيّ حالٗزٔخ‬٢‫ـ‬ٟ .c
d. Cardiac output ٢‫ حُ٘ظخؽ حُوِز‬.d

Pour les paramètres suivants, choisissez celui qui est augmenté au


cours de l‟évolution de la grossesse
a. Hématocrite
b. Pression artérielle systolique
c. Pression artérielle diastolique
d. Débit cardiaque

84. Choose from the following drugs the one most associated with a risk ‫خ‬١‫ش حألًؼَ حٍطزخ‬٤ُ‫ش حُظخ‬٣ٝ‫ٖ حألى‬٤‫حهظَ ٖٓ ر‬
of cleft lip ‫هش‬ٞ‫خرش رخُ٘لش حُٔ٘و‬ٛ‫روطَ حإل‬
a. Penicillin ٖ٤ِٔ٘‫ حُز‬.a
b. Anti-inflammatories ‫خد‬ٜ‫خىحص حالُظ‬٠ٓ .b
c. Antiepileptics ‫َع‬ُٜ‫خىحص ح‬٠ٓ .c
d. Anti-emetics ‫ء‬٢‫خىحص حُو‬٠ٓ .d
e. Antihypertensive ٢‫ـ‬٠ُ‫خص ح‬٠‫ هخك‬.e

Choisissez parmi les médicaments suivants celui qui est le plus


associé à un risque de fente labiale
a. Pénicilline
b. Anti-inflammatoires
c. Antiépileptiques
d. Antiémétiques
e. Antihypertenseurs

40
85. Klinefelter syndrome in children is characterized in its typical form ٢‫لخٍ ك‬١‫٘لِظَ ػ٘ي حأل‬٣‫ِ ٓظالُٓش ًال‬٤ٔ‫طظ‬
by: :٢ِ٣ ‫ رٔخ‬٢‫ًؿ‬ُٞٔ٘‫خ ح‬ٌِٜٗ
‫ي‬٣‫ حُ٘ي‬٢ِ‫ حُظوِق حُؼو‬.a
a. Severe mental retardation ‫ى ػالٓخص ٓليىس ػ٘ي‬ٞ‫ؿ‬ٝ ّ‫ ػي‬.b
b. The absence of specific signs at birth ‫الىس‬ُٞ‫ح‬
c. Skin hyperpigmentation ‫زؾ حُـِي‬ٜ‫ ط‬١َ‫ ك‬.c
d. Absence of proper nasal bone ْ٤ُِٔ‫ى ػظْ حألٗق ح‬ٞ‫ؿ‬ٝ ّ‫ ػي‬.d
e. Mortality in the first year of life of 70% ٠ُٝ‫ حُٔ٘ش حأل‬٢‫خص ك‬٤‫ك‬ُٞ‫ ٓؼيٍ ح‬.e
٪ 70 ‫ٖٓ حُؼَٔ ر٘ٔزش‬

Le syndrome de Klinefelter chez les enfants se caractérise dans sa


forme typique par:
a. Retard mental sévère
b. L‟absence de signes spécifiques à la naissance
c. Hyperpigmentation de la peau
d. Absence d‟os nasal approprié
e. Mortalité au cours de la première année de vie de 70%

86. In case of a bilious vomiting in a newborn, the first diagnosis to be ٍ‫لخ‬١‫ ػ٘ي حأل‬١ٝ‫لَح‬ُٜ‫ء ح‬٢‫ كخُش حُو‬٢‫ك‬
ruled out is: ١ٌُ‫ٍ ح‬ٝ‫ حأل‬ٚ٤‫ كبٕ حُظ٘و‬،‫الىس‬ُٞ‫ ح‬٢‫ؼ‬٣‫كي‬
:ٞٛ ٙ‫ـذ حٓظزؼخى‬٣
a. Esophageal atresia ‫ء‬١َُٔ‫ ٍطن ح‬.a
b. Volvulus on common mesentery ‫ش‬٤ِٛ‫ن حأل‬٣ٍ‫ حُٔٔخ‬٠ِ‫ حالٗلظخٍ ػ‬.b
c. Hypertrophic pyloric stenosis ٢ٓ‫وخ‬ٟ ٢‫حر‬ٞ‫ن ر‬٤٠‫ ط‬.c
d. Acute gastroenteritis ‫حألٓؼخء حُلخى‬ٝ ‫خد حُٔؼيس‬ٜ‫ حُظ‬.d
e. Bladder extrophy ‫ٍ حُٔؼخٗش‬ٟٞٔ .e

En cas de vomissements bilieux chez un nouveau-né, le premier


diagnostic à exclure est :
a. Atrésie œsophagienne
b. Volvulus sur mésentère commun
c. Sténose pylorique hypertrophique
d. Gastro-entérite aiguë
e. Extrophie de la vessie

41
87. Which of the following viruses would be responsible for neonatal ٖ‫ال ػ‬ٞ‫ٕ ٓٔج‬ٌٞ٤ٓ ‫ش‬٤ُ‫ٓخص حُظخ‬َٝ٤‫ ٖٓ حُل‬١‫أ‬
encephalitis if the mother has an active infection during childbirth: ‫خرش‬ٜٓ ّ‫ اًح ًخٗض حأل‬١‫ي‬٤ُُٞ‫خد حُيٓخؽ ح‬ٜ‫حُظ‬
:‫الىس‬ُٞ‫ ٗ٘طش أػ٘خء ح‬ٟٝ‫رؼي‬
a. Herpes type 2 2 ‫ع‬ُٞ٘‫َرْ ٖٓ ح‬ُٜ‫ ح‬.a
b. HIV ‫ش‬٣َ٘‫ حُٔ٘خػش حُز‬ٚ‫ّ ٗو‬َٝ٤‫ ك‬.b
c. HPV ١َ٘‫ حُز‬٢ٔ٤ِ‫ٍّ حُل‬ُٞ‫ّ ح‬َٝ٤‫ ك‬.c
d. Influenza (grippe) )‫ذ‬٣َ‫ِٗح (ؿ‬ِٞ‫ حألٗل‬.d
e. Parvovirus ٞ‫ّ رخٍك‬َٝ٤‫ ك‬.e

Lequel des virus suivants serait responsable de l‟encéphalite


néonatale si la mère a une infection active pendant l‟accouchement:

a. Herpès de type 2
b. HIV
c. HPV
d. Influenza (grippe)
e. Parvovirus

88. Regarding esophageal atresia, choose the true answer: ‫ حهظَ حإلؿخرش‬،‫ء‬١َُٔ‫ظؼِن رَطن ح‬٣ ‫ٔخ‬٤‫ك‬
:‫لش‬٤‫ل‬ُٜ‫ح‬
a. There are 6 types of esophageal atresia ‫ء‬١َُٔ‫حع ٖٓ ٍطن ح‬ٞٗ‫ أ‬6 ‫٘خى‬ٛ .a
b. The most common type is type 1 1 ‫ع‬ُٞ٘‫ ح‬ٞٛ ‫ػخ‬ٞ٤ٗ َ‫ع حألًؼ‬ُٞ٘‫ ح‬.b
c. Diagnosis is made by gastric tube not progressing ّ‫ن ػيّ طوي‬٣َ١ ٖ‫ ػ‬ٚ٤‫ظْ حُظ٘و‬٣ .c
d. In the preoperative period, we put a pharyngeal probe of ١‫د حُٔؼي‬ٞ‫حألٗز‬
the plug-in type in continuous aspiration, we maintain a ‫غ‬٠ٗ ،‫ كظَس ٓخ هزَ حُـَحكش‬٢‫ ك‬.d
half-sitting position and we do not feed the child. ‫ى‬ٝ‫ع حُٔٔي‬ُٞ٘‫خ ٖٓ ح‬٤ٓٞ‫ٓٔزخٍح رِؼ‬
e. Look for associated cleft lip malformations ‫غ‬ٟٝ ٠ِ‫ٗلخكع ػ‬ٝ ،َٔ‫ ٓٔظ‬٢‫ر٘ل‬
.َ‫ال ٗطؼْ حُطل‬ٝ ِّٞ‫ق حُـ‬ٜٗ
En ce qui concerne l‟atrésie de l‟œsophage, choisissez la vraie ‫هش‬ٞ‫خص حُ٘لش حُٔ٘و‬ٛٞ٘‫ حرلغ ػٖ ط‬.e
réponse: ‫حُٔظَحكوش‬
a. Il existe 6 types d‟atrésie œsophagienne
b. Le type le plus courant est le type 1
c. Le diagnostic est fait par la sonde gastrique ne progresse pas
d. Dans la période préopératoire, nous mettons une sonde
pharyngée de type plug-in en aspiration continue, nous
maintenons une position semi-assise et nous ne nourrissons
pas l‟enfant.
e. Rechercher les malformations labiales associées

42
89. Regarding omphalocele and laparoschisis, choose the true answer: ،ٖ‫حٗ٘وخم حُزط‬ٝ ‫ش‬٣َُٔ‫ِش ح‬٤‫ظؼِن رخُو‬٣ ‫ٔخ‬٤‫ك‬
:‫لش‬٤‫ل‬ُٜ‫حهظَ حإلؿخرش ح‬
a. The omphalocele is defined by viscera contained in a sac ٍ‫ش ٖٓ هال‬٣َُٔ‫ِش ح‬٤‫ق حُو‬٣َ‫ظْ طؼ‬٣ .a
centered by the umbilical cord ْ٤ً ٢‫ىس ك‬ٞ‫ؿ‬ُٞٔ‫حألك٘خء ح‬
b. Laparoschisis is defined as the exteriorization of viscera ١َُٔ‫ حُلزَ ح‬ٚ‫ٓط‬ٞ‫ظ‬٣
outside the fetal abdomen at the left lateral border of the ‫لخء‬ٟ‫ ا‬ٚٗ‫ؼَف حٗ٘وخم حُزطٖ رؤ‬٣ .b
umbilicus ‫ حألك٘خء‬٠ِ‫ ػ‬٢‫حُطخرغ حُوخٍؿ‬
c. There are malformations associated with laparoschisis ‫ى‬ٝ‫ٖ ػ٘ي حُلي‬٤٘‫هخٍؽ رطٖ حُـ‬
d. In the delivery room, the viscera must be left in the open .‫ َُِٔس‬َٟٔ٤ُ‫ش ح‬٤‫حُـخٗز‬
air to cool ‫خص ٓظَحكوش ٓغ حٗ٘وخم‬ٛٞ٘‫٘خى ط‬ٛ .c
e. Postoperatively, feeding is done as for any new baby ٖ‫حُزط‬
without malformations ‫ـذ طَى‬٣ ،‫الىس‬ُٞ‫ ؿَكش ح‬٢‫ ك‬.d
‫حء حُطِن ُظزَى‬ُٜٞ‫ ح‬٢‫حألك٘خء ك‬
En ce qui concerne l‟omphalocèle et la laparoschisis, choisissez la ‫ش‬٣ٌ‫ طظْ حُظـ‬،‫ش‬٤‫ش حُـَحك‬٤ِٔ‫ رؼي حُؼ‬.e
vraie réponse: ‫ي‬٣‫لَ ؿي‬١ ١‫ حُلخٍ ٓغ أ‬ٞٛ ‫ًٔخ‬
a. L‟omphalocèle est définie par des viscères contenus dans un ‫خص‬ٛٞ٘‫ٕ ط‬ٝ‫ى‬
sac centré par le cordon ombilical
b. La laparoschisis est définie comme l‟extériorisation des
viscères à l‟extérieur de l‟abdomen fœtal au bord latéral
gauche de l‟ombilic
c. Il existe des malformations associées à la laparoschisis
d. Dans la salle d‟accouchement, les viscères doivent être laissés
à l‟air libre pour refroidir
e. Postopératoirement, L‟alimentation se fait comme pour tout
nouveau bébé sans malformations

43
90. For a 40-y old patient with an acute necrotizing pancreatitis of biliary ‫ ػخٓخ‬40 َٔ‫زِؾ ٖٓ حُؼ‬٣ ٞ٣َُٔ ‫رخُ٘ٔزش‬
origin, what is the best therapeutic proposal? ٖٓ ‫خّ حُ٘خهَ حُلخى‬٣ٌَ٘‫خد حُز‬ٜ‫خرخ رخُظ‬ٜٓ
a. The drainage technique of choice is endoscopic ‫َ حهظَحف‬٠‫ أك‬ٞٛ ‫ ٓخ‬، ١ٝ‫لَح‬ٛ ‫ٓ٘٘ؤ‬
retrograde cholangiopancreatography when it is ‫؟‬٢‫ػالؿ‬
available. ٢ٛ ‫ِش‬٠‫ق حُٔل‬٣َٜ‫ش حُظ‬٤٘‫ طو‬.a
b. This technique is extremely easy to perform and is ‫حص‬ٞ٘‫حُو‬ٝ ّ‫خ‬٣ٌَ٘‫َ حُز‬٣ٜٞ‫ط‬
rarely complicated. ‫ن‬٣َ‫ش رخُٔ٘ظخٍ رخُط‬٣ٝ‫لَح‬ُٜ‫ح‬
c. Surgical treatment should be preferred as second line. .‫ٕ ٓظخكش‬ٌٞ‫حَُحؿغ ػ٘يٓخ ط‬
d. Minimally invasive drainage techniques remain poorly ‫ٗخىٍح ٓخ‬ٝ ‫ش‬٣‫ِش ُِـخ‬ٜٓ ‫ش‬٤٘‫ حُظو‬ٌٙٛ .b
used. .‫ٕ ٓؼويس‬ٌٞ‫ط‬
e. Minimally invasive drainage techniques are difficult ٢‫َ حُؼالؽ حُـَحك‬٤٠‫ـذ طل‬٣ .c
to implement .ٕ‫ ػخ‬٢‫ًو‬
‫لش‬٤‫ل‬١ ‫َف‬ُٜ‫خص ح‬٤٘‫ ال طِحٍ طو‬.d
Pour un patient de 40 ans atteint d‟une pancréatite nécrosante aiguë .ّ‫جش حالٓظويح‬٤ٓ َ‫ؿ‬ٞ‫حُظ‬
d‟origine biliaire, quelle est la meilleure proposition thérapeutique ? ‫َف‬ُٜ‫خص ح‬٤٘‫ٌ طو‬٤‫ؼذ ط٘ل‬ُٜ‫ ٖٓ ح‬.e
a. La technique de drainage de choix est la َ‫ؿ‬ٞ‫لش حُظ‬٤‫ل‬١
cholangiopancréatographie rétrograde endoscopique
lorsqu‟elle est disponible.
b. Cette technique est extrêmement facile à réaliser et est
rarement compliquée.
c. Le traitement chirurgical doit être préféré en deuxième
intention.
d. Les techniques de drainage mini-invasives restent peu
utilisées.
e. Les techniques de drainage mini-invasives sont difficiles à
mettre en œuvre

91. Regarding trauma of the abdomen in a patient with a car accident, the ١ٌُ‫ ح‬ٞ٣َُٔ‫ ح‬ٟ‫يٓش حُزطٖ ُي‬ٜ‫ظؼِن ر‬٣ ‫ٔخ‬٤‫ك‬
viscera most commonly affected is: َ‫ كبٕ حألك٘خء حألًؼ‬،‫خٍس‬٤ٓ ‫ ُلخىع‬َٝ‫ظؼ‬٣
:٢ٛ ‫طؤػَح‬
a. The liver ‫ حٌُزي‬.a
b. The stomach ‫ حُٔؼيس‬.b
c. The spleen ٍ‫ حُطلخ‬.c
d. The intestine ‫ حألٓؼخء‬.d
e. The colon ُٕٞٞ‫ حُو‬.e

En ce qui concerne le traumatisme de l‟abdomen chez un patient


victime d‟un accident de voiture, les viscères les plus fréquemment
touchés sont:
a. Le foie
b. L‟estomac
c. La rate
d. L‟intestin
e. Le colon

44
92. A 23 year old man with a one month history of appendicectomy ‫ن‬٣ٍ‫ طخ‬ٚ٣‫ُي‬ٝ ‫ ػخٓخ‬23 َٔ‫زِؾ ٖٓ حُؼ‬٣ َ‫ٍؿ‬
presents severe vomiting and complains of abdominal pain . What ‫ش‬٣‫ى‬ٝ‫خٍ حُِحثيس حُي‬ٜ‫حكي ٖٓ حٓظج‬ٝ َٜٗ
would be the most important question to be asked : ٢‫ ٖٓ آالّ ك‬ٌٞ٘٣ٝ ‫ي‬٣‫ء حُ٘ي‬٢‫ ٖٓ حُو‬٢ٗ‫ؼخ‬٣
:ٚ‫َك‬١ ‫ـذ‬٣ ٍ‫ْ ٓئح‬ٛ‫ أ‬ٞٛ ‫ ٓخ‬.ٖ‫حُزط‬
a. Color of vomit? ‫ء؟‬٢‫ٕ حُو‬ُٞ .a
b. Stool elimination? ‫ حٗطَحف حُزَحُ؟‬.b
‫ حٗطَحف حُـخُ؟‬.c
c. Gas elimination?
‫ ٓئهَح؟‬ُٚٝ‫ ط٘خ‬١ٌُ‫ ٓخ حُطؼخّ ح‬.d
d. What food did he recently ingest? ‫ٕ حُزَحُ؟‬ُٞ .e
e. Color of stool?

Un homme de 23 ans ayant des antécédents d‟appendicectomie d‟un


mois présente de graves vomissements et se plaint de douleurs
abdominales. Quelle serait la question la plus importante à se poser :

a. Couleur du vomi ?
b. Élimination des selles?
c. Élimination des gaz?
d. Quelle nourriture a-t-il récemment ingérée?
e. Couleur des selles?

93. To achieve best results with antibiotic prophylaxis, care must be taken ‫ش‬٣ٞ٤‫خىحص حُل‬٠ُٔ‫َ حُ٘ظخثؾ ٓغ ح‬٠‫ن أك‬٤‫ُظلو‬
when it is given in the course of surgery: ‫خ‬ٜ‫ حُلٌٍ ػ٘ي اػطخث‬٢‫ه‬ٞ‫ـذ ط‬٣ ،‫ش‬٤‫هخث‬ُٞ‫ح‬
:‫أػ٘خء حُـَحكش‬
a. Always precede the surgical procedure, within a maximum of ٢‫ ك‬،‫ش‬٤‫ش حُـَحك‬٤ِٔ‫ طٔزن ىحثٔخ حُؼ‬.a
1h to 1h30, from the time of the injection of the anesthesia ‫ق‬ٜٗٝ ‫ ٓخػش‬٠ُ‫ٕ ٓخػش ا‬ٞ٠‫ؿ‬
b. Perform in the post-operative phase, within a minimum of 30 َ٣‫هض كوٖ حُظوي‬ٝ ٖٓ ،٠ٜ‫ًلي أه‬
min. to 1 h. ‫ َٓكِش ٓخ رؼي‬٢‫خ ك‬ٛ‫ اػطخء‬.b
c. It should be done during the procedure, once the anesthetic ‫وش‬٤‫ ىه‬30 ٕٞ٠‫ ؿ‬٢‫ ك‬،‫حُـَحكش‬
product is injected, within a maximum of 12 hours and / or .َ‫ حأله‬٠ِ‫ ٓخػش ػ‬٠ُ‫ا‬
exceptionally 24 hours. ‫ظْ ًُي أػ٘خء حإلؿَحء‬٣ ٕ‫ـذ أ‬٣ .c
d. Precede the procedure, within a maximum of 1 to 2 days ‫حء‬ٝ‫ رٔـَى كوٖ ى‬،٢‫حُـَحك‬
e. Is best done 6 hours after the incision ‫ ٓخػش‬12 ٕٞ٠‫ ؿ‬٢‫ ك‬،َ٣‫حُظوي‬
‫ ٓخػش‬24 ٝ‫ أ‬/ ٝ ٠ٜ‫ًلي أه‬
.‫خ‬٤‫حٓظؼ٘خث‬
Pour obtenir les meilleurs résultats avec l‟antibioprophylaxie, des ٢‫ ك‬،٢‫ طٔزن حإلؿَحء حُـَحك‬.d
précautions doivent être prises lorsqu‟elle est administrée au cours de ٠ٜ‫خّ ًلي أه‬٣‫ أ‬2 ٠ُ‫ ا‬1 ٕٞ٠‫ؿ‬
la chirurgie: ‫ ٓخػخص‬6 ‫خ رؼي‬ٜ‫َ اػطخث‬٠‫ ٖٓ حألك‬.e
a. Précèdez toujours l‟intervention chirurgicale, dans un délai ٢‫ٖٓ حُ٘ن حُـَحك‬
maximum de 1h à 1h30, à partir du moment de l‟injection de
l‟anesthésie
b. Effectuer dans la phase post-opératoire, dans un délai
minimum de 30 min. à 1 h.
c. Il doit être fait pendant l‟intervention, une fois le produit
anesthésique injecté, dans un délai maximum de 12 heures
et/ou exceptionnellement 24 heures.
d. Précéder l‟intervention, dans un délai maximum de 1 à 2 jours
e. Il est préférable de le faire 6 heures après l‟incision

45
94. A 67-year-old female is admitted to the hospital with a hip fracture ٠ُ‫ ػخٓخ ا‬67 َٔ‫طْ اىهخٍ حَٓأس طزِؾ ٖٓ حُؼ‬
after a fall. Which of the following regimens constitutes appropriate ‫ٍى رؼي‬ُٞ‫ ح‬٢‫خرش رٌَٔ ك‬ٜٓ ٠‫حُٔٔظ٘ل‬
venous thromboembolism prophylaxis for this patient? ‫ش‬٣‫هخ‬ُٞ‫ح‬ ٌَ٘٣ ‫ش‬٤ُ‫ ٖٓ حألٗظٔش حُظخ‬١‫ أ‬.‫خ‬ٜ١ٞ‫ٓو‬
ٌُٜٙ ٢٣‫ي‬٣ٍُٞ‫ ح‬٢ٓ‫ٔخ‬ٜٗ‫حُٔ٘خٓزش ٖٓ حُوؼخٍحال‬
a. Intermittent pneumatic compression devices ‫ش؟‬٠٣َُٔ‫ح‬
b. Subcutaneous unfractionated heparin ‫ حُٔظوطغ‬٢‫حث‬ُٜٞ‫ ح‬٢‫ـ‬٠ُ‫ِس ح‬ٜ‫ أؿ‬.a
c. Subcutaneous low-molecular-weight heparin ‫حُـِي‬ ‫َ حُٔـِأ طلض‬٤‫ٖ ؿ‬٣ٍ‫زخ‬٤ُٜ‫ ح‬.b
d. Warfarin, with a target international normalized ratio ٢‫ج‬٣ِ‫ُٕ حُـ‬ُٞ‫ ح‬ٞ‫ٖ ٓ٘ول‬٣ٍ‫زخ‬٤ُٜ‫ ح‬.c
(INR) of 1.5 to 2.0 ‫طلض حُـِي‬
e. Aspirin 1.5 ٖٓ ) INR ( ‫ٓغ‬ ،ٖ٣ٍ‫حٍكخ‬ُٞ‫ح‬ .d
2.0 ٠ُ‫ا‬
ٖ٣َ‫ حألٓز‬.e
Une femme de 67 ans est admise à l‟hôpital avec une fracture de la
hanche après une chute. Lequel des schémas suivants constitue une
prophylaxie de thromboembolie veineuse appropriée pour ce patient?
a. Dispositifs de compression pneumatique intermittente
b. Héparine sous-cutanée non fractionnée
c. Héparine sous-cutanée de bas poids moléculaire
d. Warfarine, avec un rapport international normalisé (INR)
cible de 1,5 à 2,0
e. Aspirine

95. A 48-year-old man, formerly hypertensive, suddenly suffered from ٢ٗ‫ؼخ‬٣ ٕ‫ ًخ‬،‫ ػخٓخ‬48 َٔ‫زِؾ ٖٓ حُؼ‬٣ َ‫ٍؿ‬
retrosternal chest pain with radiation to the back. Cardiac auscultation ٖٓ ‫ كـؤس‬٠ٗ‫ ػخ‬،ّ‫ حُي‬٢‫ـ‬ٟ ‫ٓخروخ ٖٓ حٍطلخع‬
reveals a diastolic murmur along the left border of the sternum. The .َٜ‫ حُظ‬ٞ‫ِظق ٗل‬٣ ٚ‫يٍ هِق حُو‬ُٜ‫ ح‬٢‫أُْ ك‬
٠ِ‫ش ػ‬٤١‫ ػٖ ٗلوش حٗزٔخ‬٢‫ٌ٘ق حُظٔٔغ حُوِز‬٣
ECG at the time of pain is normal. What is the diagnosis to evoke
‫ حُوِذ‬٢٤‫ طوط‬.ٚ‫ ُِو‬َٟٔ٤ُ‫ى ح‬ٝ‫ٍ حُلي‬ٞ١
first? ١ٌُ‫ ح‬ٚ٤‫ حُظ٘و‬ٞٛ ‫ ٓخ‬.٢‫ؼ‬٤‫ز‬١ ُْ‫هض حأل‬ٝ ٢‫ك‬
‫ال؟‬ٝ‫ أ‬ٍٙ‫خ‬٠‫ـذ حٓظل‬٣
a. Acute pericarditis
‫ٍ حُلخى‬ٞٓ‫خد حُظخ‬ٜ‫ حُظ‬.a
b. A massive pulmonary embolism
َ‫خث‬ٛ ١ٞ‫ٔخّ ٍث‬ٜٗ‫ ح‬.b
c. Aortic dissection
َٜ‫ طِٔن حألر‬.c
d. Myocardial infarction
‫ِش حُوِذ‬٠‫ حكظ٘خء ػ‬.d
e. Pneumothorax
ٍ‫ي‬ُٜ‫حف ح‬َٝ‫ حٓظ‬.e
Un homme de 48 ans, anciennement hypertendu, a soudainement
souffert de douleurs thoraciques rétrosternales avec radiothérapie au
dos. L‟auscultation cardiaque révèle un souffle diastolique le long du
bord gauche du sternum. L‟ECG au moment de la douleur est normal.
Quel est le diagnostic à évoquer en premier ?
a. Péricardite aiguë
b. Une embolie pulmonaire massive
c. Dissection aortique
d. Infarctus du myocarde
e. Pneumothorax

46
96. Which of the following attitudes do you immediately adopt in the ‫ٍ ػ٘ي‬ٞ‫ حُل‬٠ِ‫ ػ‬ٙ‫ش طظز٘خ‬٤ُ‫حهق حُظخ‬ُٞٔ‫ ٖٓ ح‬١‫أ‬
presence of mild epistaxis? ‫ق؟‬٤‫ع ٍػخف هل‬ٝ‫كي‬
‫ ىهخثن‬10 ‫ٖ ُٔيس‬٤‫زؼ‬ٛ‫ رخإل‬٢‫ـ‬ٟ .a
a. Bi-digital compression for 10 minutes ٢ٓ‫ ىًخى أٓخ‬.b
b. Anterior tamponade ٢‫ ىًخى هِل‬.c
c. Posterior tamponade ‫ش‬٣ٞٓ‫ش حُي‬٤‫ػ‬ٝ‫ حأل‬٢‫ ٍر‬.d
d. Vascular ligation ‫ٕ َٓهت‬ُٞ‫غ رخ‬ٟٝ .e
e. Placement of a hemostatic balloon

Laquelle des attitudes suivantes adoptez-vous immédiatement en


présence d‟épistaxis légère?
a. Compression bi-digitale pendant 10 minutes
b. Tamponnade antérieure
c. Tamponnade postérieure
d. Ligature vasculaire
e. Mise en place d‟un ballonnet hémostatique

97. Which of the following clinical signs is suggestive of unilateral َِٗ ٠ُ‫َ ا‬٤٘‫ش ط‬٤ُ‫ش حُظخ‬٣َ٣َُٔ‫ حُؼالٓخص ح‬١‫أ‬
recurrent nerve paralysis? ‫ حُـخٗذ؟‬١‫خد حَُحؿغ أكخى‬ٜ‫حألػ‬
‫ هَِ حُ٘طن‬.a
a. Dysphonia ‫ص ٖٓ حألٗق‬ٞٛ .b
b. A nasal voice ‫ ػَٔ حُزِغ‬.c
c. Dysphagia ْ‫ن حُظ٘ل‬٤ٟ .d
d. Dyspnea ُُٕٞ‫ كويحٕ ح‬.e
e. Weight loss

Lequel des signes cliniques suivants suggère une paralysie nerveuse


unilatérale récurrente?
a. Dysphonie
b. Une voix nasale
c. Dysphagie
d. Dyspnée
e. Perte de poids

47
98. Which of the following items corresponds to a risk factor for the onset َٓ‫حكن ٓغ ػخ‬ٞ‫ظ‬٣ ‫ش‬٤‫ط‬٥‫َ ح‬ٛ‫ ٖٓ حُؼ٘خ‬١‫أ‬
of cholelithiasis? ‫؟‬١ٝ‫لَح‬ٛ ٚ‫ٍ طل‬ٜٞ‫هطَ ُظ‬
١ًٌَُ‫ حُـْ٘ ح‬.a
a. the male gender ٍٞ‫الى حٌُل‬ٜ‫ حٓظ‬.b
b. alcohol consumption 50 ٖٓ َ‫ حُؼَٔ أه‬.c
c. age below 50 ٖ٤‫ط‬َٝ‫ حُز‬٢ُ‫ ػخ‬٢‫ حطزخع ٗظخّ ؿٌحث‬.d
d. a high protein diet ُُٕٞ‫خىس ح‬٣ُ .e
e. overweight

Lequel des éléments suivants correspond à un facteur de risque


d‟apparition de la lithiase biliaire?
a. le sexe masculin
b. consommation d‟alcool
c. âge inférieur à 50 ans
d. un régime riche en protéines
e. surpoids

99. A 40-year-old woman consults because she has had a sensitive ‫خ‬ٜٗ‫َ أل‬٤٘‫ ػخٓخ طٔظ‬40 َٔ‫حَٓأس طزِؾ ٖٓ حُؼ‬
subcutaneous venous cord in the left calf for 1 week. Examination ‫ كٔخّ طلض‬١‫ي‬٣ٍٝ َ‫ ٖٓ كز‬٢ٗ‫ًخٗض طؼخ‬
shows redness of the skin surrounding the vein. Which of the .‫ع‬ٞ‫ أٓز‬1 ‫ ُٔيس‬َٟٔ٤ُ‫ ٍرِش حُٔخم ح‬٢‫حُـِي ك‬
.‫ي‬٣ٍُٞ‫ رخ‬٢٤‫ حكَٔحٍ حُـِي حُٔل‬ٚ‫َ حُلل‬ٜ‫ظ‬٣
following abnormalities is most likely responsible for the disorder?
٠ِ‫ٍ ػ‬ٝ‫ حُٔٔئ‬ٞٛ ‫ش‬٤ُ‫خص حُظخ‬ٛٞ٘‫ ٖٓ حُظ‬١‫أ‬
a. Calf muscle venous pump defect ‫طَحد؟‬ٟ‫ٌح حال‬ٛ ٖ‫حألٍؿق ػ‬
b. Valvular incontinence at the saphenofemoral junction ‫ِش‬٠‫ش ُؼ‬٣‫ي‬٣ٍُٞ‫وش ح‬٠ُٔ‫ذ ح‬٤‫ ػ‬.a
c. Increased deep venous system pressure ‫حُٔخم‬
d. Calf Deep Vein Obstruction َُٛٞٔ‫ٔخٓخص ػ٘ي ح‬ُٜ‫ ِْٓ ح‬.b
e. Thrombosis of a superficial vein ١ٌ‫خكٖ حُلو‬ُٜ‫ح‬
١‫ي‬٣ٍُٞ‫خُ ح‬ٜ‫ حُـ‬٢‫ـ‬ٟ ‫خىس‬٣ُ .c
Une femme de 40 ans consulte parce qu‟elle a un cordon veineux ‫ن‬٤ٔ‫حُؼ‬
sous-cutané sensible dans le mollet gauche depuis 1 semaine. ‫ ٍرِش‬٢‫ن ك‬٤ٔ‫ي حُؼ‬٣ٍُٞ‫ حٗٔيحى ح‬.d
L‟examen montre une rougeur de la peau entourant la veine. Laquelle ‫حُٔخم‬
des anomalies suivantes est la plus probablement responsable du ٢‫ي حُٔطل‬٣ٍُٞ‫ ح‬٢‫ طوؼَ ك‬.e
trouble?
a. Défaut de la pompe veineuse musculaire du mollet
b. Incontinence valvulaire à la jonction saphéno-morale
c. Augmentation de la pression du système veineux profond
d. Obstruction veineuse profonde du mollet
e. Thrombose d‟une veine superficielle

48
100. Which of the following is the MOST common cause of painful rectal ‫ق‬٣ُِ٘ ‫ػخ‬ٞ٤ٗ َ‫ حُٔزذ حألًؼ‬ٞٛ ٢ِ٣ ‫ ٓٔخ‬١‫أ‬
bleeding? ‫ْ حُٔئُْ؟‬٤‫حُٔٔظو‬
a. Internal hemorrhoids ‫ش‬٤ِ‫َ حُيحه‬٤ٓ‫ح‬ٞ‫ حُز‬.a
b. External hemorrhoids ‫ش‬٤‫َ حُوخٍؿ‬٤ٓ‫ح‬ٞ‫ حُز‬.b
c. Diverticulitis ‫خد حَُطؾ‬ٜ‫ حُظ‬.c
d. Anal fissure ٢‫ حُ٘ن حَُ٘ؿ‬.d
e. Rectal foreign body ْ٤‫ حُٔٔظو‬٢‫ذ ك‬٣َ‫ ؿْٔ ؿ‬.e

Lequel des éléments suivants est la cause la plus fréquente de


saignement rectal douloureux?
a. Hémorroïdes internes
b. Hémorroïdes externes
c. Diverticulite
d. Fissure anale
e. Corps étranger rectal

101. The first (in frequency) non-obstetric surgical emergency during ‫ش‬٣‫ي‬٤ُٞ‫َ ط‬٤‫ش ؿ‬٤‫حٍة ؿَحك‬ٞ١ ‫ٍ كخُش‬ٝ‫أ‬
pregnancy is: :٢ٛ َٔ‫حطَ) أػ٘خء حُل‬ٞ‫(رخُظ‬
a. Appendicitis ‫ش‬٣‫ى‬ٝ‫خد حُِحثيس حُي‬ٜ‫ حُظ‬.a
b. cholecystitis ‫خد حَُٔحٍس‬ٜ‫ حُظ‬.b
c. Intestinal obstruction ١ٞ‫ حٗٔيحى ٓؼ‬.c
d. volvulus ٍ‫ حٗلظخ‬.d
e. Perforated ulcer ‫ هَكش ٓؼوزش‬.e

La première urgence chirurgicale non obstétricale (en fréquence)


pendant la grossesse est:
a. Appendicite
b. Cholécystite
c. Obstruction intestinale
d. Volvulus
e. Ulcère perforé

102. In a 45-year-old man which of the following is the MOST common ‫ ٓٔخ‬١‫ أ‬،‫ ػخٓخ‬45 َٔ‫زِؾ ٖٓ حُؼ‬٣ َ‫ ٍؿ‬٢‫ك‬
presentation of gallstones? ‫خص‬٤ٜ‫ػخ ُل‬ٞ٤ٗ َ‫ حألًؼ‬َٝ‫ حُؼ‬ٞٛ ٢ِ٣
a. Acute pancreatitis ‫حَُٔحٍس؟‬
b. Biliary colic ‫خّ حُلخى‬٣ٌَ٘‫خد حُز‬ٜ‫ حُظ‬.a
c. Ascending cholangitis ١ٝ‫لَح‬ُٜ‫ ح‬ٚ‫ حُٔـ‬.b
d. Gallbladder empyema ‫خػيس‬ُٜ‫ش ح‬٣ٝ‫لَح‬ُٜ‫ش ح‬٤٘‫خد حأله‬ٜ‫ حُظ‬.c
‫ِش حَُٔحٍس‬٤‫ ىر‬.d

Chez un homme de 45 ans, laquelle des présentations suivantes est la


plus courante des calculs biliaires?
a. Pancréatite aiguë
b. Coliques biliaires
c. Cholangite ascendante
d. Empyème de la vésicule biliaire

49
103. A 51-year-old patient is seen in your office several weeks after a ٢‫ ػخٓخ ك‬51 َٔ‫ش طزِؾ ٖٓ حُؼ‬٠٣َٓ ‫ي‬ٛ‫ط٘خ‬
parathyroidectomy for a parathyroid adenoma. She is complaining of ‫خٍ ؿخٍحص‬ٜ‫غ ٖٓ حٓظج‬٤‫خىطي رؼي ػيس أٓخر‬٤‫ػ‬
parethesias. Physical examination reveals contraction of the right .‫ ؿخٍحص حُيٍم‬٢‫ي ك‬٤ٔ‫ٍّ ك‬ٝ ‫حُيٍم رٔزذ‬
facial muscles when you tap lightly over the right side of the patient‟s ٢ٗ‫ حُزي‬ٚ‫ٌ٘ق حُلل‬٣ .ٌٍٓ ٖٓ ٌٞ٘‫خ ط‬ٜٗ‫ا‬
face. Which of the following is the most likely diagnosis? َ‫ ػ٘ي حُ٘و‬٠٘ٔ٤ُ‫ ح‬ٚ‫ؿ‬ُٞ‫الص ح‬٠‫ ػ‬ِٚ‫ػٖ طو‬
ٚ‫ؿ‬ٝ ٖٓ ٖٔ٣‫ حُـخٗذ حأل‬٠ِ‫رَكن ػ‬
a. Hypokalemia ٚ٤‫ حُظ٘و‬ٞٛ ٢ُ‫ ٖٓ حُظخ‬١‫ أ‬.‫ش‬٠٣َُٔ‫ح‬
b. Hypercalcemia ‫لخ؟‬٤‫حألًؼَ طَؿ‬
c. Hyperkalemia ّ‫ّ حُي‬ٞ٤ٓ‫طخ‬ٞ‫ ر‬ٚ‫ ٗو‬.a
d. Hypocalcemia ّ‫ّ حُي‬ٞ٤ُٔ‫ ًخ‬١َ‫ ك‬.b
e. Hyponatremia ّ‫ّ حُي‬ٞ٤ٓ‫طخ‬ٞ‫ ر‬١َ‫ ك‬.c
ّ‫ّ حُي‬ٞ٤ُٔ‫ ًخ‬ٚ‫ ٗو‬.d
ّ‫ّ حُي‬ٞ٣‫ى‬ٞٛ ٚ‫ ٗو‬.e
Un patient de 51 ans est vu dans votre cabinet plusieurs semaines
après une parathyroïdectomie pour un adénome parathyroïdien. Elle
se plaint de parérésie. L‟examen physique révèle une contraction des
muscles faciaux droits lorsque vous tapotez légèrement sur le côté
droit du visage du patient. Lequel des énoncés suivants est le
diagnostic le plus probable?
a. Hypokaliémie
b. Hypercalcémie
c. Hyperkaliémie
d. Hypocalcémie
e. Hyponatrémie

104. What is the first-line test for a peptic ulcer? ‫ش؟‬٤ٔ٠ُٜ‫ٍ ُِوَكش ح‬ٝ‫ حأل‬٢‫ حهظزخٍ حُو‬ٞٛ ‫ٓخ‬
‫ش‬٣‫ذ حُٔؼيس حُوخػي‬٤‫ أٗخر‬.a
a. Basal gastric tubing ‫ش‬٣‫خ هخػي‬٤ٔ٤٘٣َ‫ ؿخٓظ‬.b
b. Basal gastrinemia ٢‫ش ك‬٤‫حر‬ٞ‫خص حُز‬٣ُِٞٔ‫ ػٖ ح‬١َ‫ حُظل‬.c
c. Research of Helicobacter Pylori in the antral region ‫ش‬٣ٍ‫حُٔ٘طوش حُـخ‬
d. Abdominal computed tomography ‫ٓذ‬ٞ‫ حُٔل‬٢‫َ حُٔوطؼ‬٣ٜٞ‫ حُظ‬.d
e. Searching for hyperthyroidism ٖ‫ُِزط‬
‫ حُـيس‬١‫ ٗ٘خ‬١َ‫ حُزلغ ػٖ ك‬.e
‫ش‬٤‫حُيٍه‬
Quel est le test de première intention pour un ulcère peptique?
a. Tubulure gastrique basale
b. Gastrinémie basale
c. Recherche sur Helicobacter Pylori dans la région antrale
d. Tomodensitométrie abdominale
e. Recherche de l‟hyperthyroïdie

50
105. What is the most common cause of pancreatitis in a 32-y old male? ‫خد‬ٜ‫ػخ الُظ‬ٞ٤ٗ َ‫ حُٔزذ حألًؼ‬ٞٛ ‫ٓخ‬
a. Cholelithiasis 32 َٔ‫ٖ ٖٓ حُؼ‬٤‫ٍ حُزخُـ‬ًٌُٞ‫ ح‬ٟ‫خّ ُي‬٣ٌَ٘‫حُز‬
b. Alcoholism ‫ػخٓخ؟‬
c. Abdominal trauma ١ٝ‫لَح‬ٛ ٚ‫ طل‬.a
d. Penetrating peptic ulcer ٍٞ‫ اىٓخٕ حٌُل‬.b
e. Salicylate poisoning ٖ‫ حُزط‬٢‫يٓش ك‬ٛ .c
‫ش ٓؼوزش‬٤ٔ٠ٛ ‫ هَكش‬.d
Quelle est la cause la plus fréquente de pancréatite chez un homme ‫الص‬٤ٔ٤ُ‫ حُظْٔٔ رخُٔخ‬.e
âgé de 32 ans?
a. Cholélithiase
b. Alcoolisme
c. Traumatisme abdominal
d. Ulcère peptique pénétrant
e. Empoisonnement au salicylate

106. A 35-y old man complains of bone pain. Among these benign tumoral ّ‫ ٖٓ آال‬ٌٞ٘٣ ‫ ػخٓخ‬35 َٔ‫زِؾ ٖٓ حُؼ‬٣ َ‫ٍؿ‬
lesions, which is the one that corresponds to pain with nocturnal ‫ش‬٤ٗ‫خ‬١َُٔ‫كخص ح‬٥‫ ح‬ٌٙٛ ٖ٤‫ ٖٓ ر‬.ّ‫ حُؼظخ‬٢‫ك‬
recrudescence? ٢ٓ‫حكن ٓغ أُْ حٗظٌخ‬ٞ‫ طظ‬٢‫ حُظ‬٢ٛ ‫ ٓخ‬،‫يس‬٤ٔ‫حُل‬
a. Osteogenic exostosis ‫؟‬٢ِ٤ُ
b. Osteoid osteoma ٢ٔ‫ ػَٕ ػظ‬.a
c. Non-osteogenic fibroma ٢ٗ‫ٍّ ػظٔخ‬ٝ .b
d. The essential bone cyst ٢ٔ‫َ ػظ‬٤‫ ؿ‬٢‫ل‬٤ُ ٍّٝ .c
e. The isolated chondroma ٢ٓ‫ حألٓخ‬٢ٔ‫ْ حُؼظ‬٤ٌُ‫ ح‬.d
ٍِٝ‫ حُٔؼ‬٢‫ك‬َٝ٠‫ٍّ حُـ‬ُٞ‫ ح‬.e
Un homme de 35 ans se plaint de douleurs osseuses. Parmi ces
lésions tumorales bénignes, laquelle est celle qui correspond à une
douleur avec recrudescence nocturne ?
a. Exostose ostéogénique
b. Ostéome ostéoïde
c. Fibrome non ostéogénique
d. Le kyste osseux essentiel
e. Le chondrome isolé

107. Which of the following organisms is the major cause of most ٢ٔ٤‫ حُٔزذ حَُث‬٢ٛ ‫ش‬٤ُ‫خص حُظخ‬٣ٞ٠‫ ٖٓ حُؼ‬١‫أ‬
travelers‟ diarrhea? ‫ٖ؟‬٣َ‫خٍ ٓؼظْ حُٔٔخك‬ٜٓ‫إل‬
a. E.coli ‫ش‬٤ُٗٞٞ‫ش حُو‬٤ٌ٣َٗ‫ حإل‬.a
b. Campylobacter ‫لش‬٤‫ حُؼط‬.b
c. Vibrio ‫ٔش‬٠ُ‫ ح‬.c
d. Giardia ‫خ‬٣‫خٍى‬٤‫ حُـ‬.d
e. Shigella ‫ال‬٤‫ـ‬٤ُ٘‫ ح‬.e

Lequel des organismes suivants est la principale cause de la diarrhée


de la plupart des voyageurs?
a. E. coli
b. Campylobacter
c. Vibrio
d. Giardia
e. Shigella

51
108. Acute hyperkalemia is associated with which of the following ٖٓ ١‫ّ حُيّ حُلخى رؤ‬ٞ٤ٓ‫طخ‬ٞ‫ ر‬١َ‫ ك‬٢‫َطز‬٣
electrocardiographic changes? ‫ش حُوِذ؟‬٤‫َر‬ًٜ ٢٤‫ طوط‬٢‫ش ك‬٤ُ‫َحص حُظخ‬٤‫حُظـ‬
QRS ‫غ‬٤ٓٞ‫ ط‬.a
a. QRS widening ST ‫ ٓوطغ‬٢‫خُش ك‬١‫ ا‬.b
b. Prolongation of the ST segment PR ‫ كظَس‬٢‫ ك‬ٝ‫ حٗولخ‬.c
c. A decrease in the PR interval ‫ رخٍُس‬U ‫ؿخص‬ٞٓ .d
d. Prominent U waves T ‫ؿش‬ٞٓ ‫ق‬٤‫ طٔط‬.e
e. T-wave flattening

L‟hyperkaliémie aiguë est associée à lequel des changements


électrocardiographiques suivants?
a. Élargissement du QRS
b. Allongement du segment ST
c. Diminution de l‟intervalle PR
d. Ondes U proéminentes
e. Aplatissement de l‟onde T

109. Which of the following locations is the type of cancer most often ١ٌُ‫خٕ ح‬١َُٔ‫ع ح‬ٞٗ ٞٛ ‫ش‬٤ُ‫حهغ حُظخ‬ُٞٔ‫ ٖٓ ح‬١‫أ‬
giving rise to liver metastases? َ‫ٍ ٗوخث‬ٜٞ‫ ظ‬٠ُ‫خٕ ا‬٤‫ أؿِذ حألك‬٢‫ ك‬١‫ئى‬٣
‫ش؟‬٣‫ًزي‬
a. Uterine adenocarcinoma ٢ٔ‫ش ٍك‬٣‫خٗش ؿي‬١َٓ .a
b. Squamous cell carcinoma of the esophagus ٢‫ش ك‬٤‫خ حُلَٗل‬٣‫خٕ حُوال‬١َٓ .b
c. Colorectal adenocarcinoma ‫ء‬١َُٔ‫ح‬
d. Osteosarcoma ُٕٞٞ‫ حُو‬٢‫ش ك‬٣‫خٗش ؿي‬١َٓ .c
e. Testicular seminoma ْ٤‫حُٔٔظو‬ٝ
‫ش‬٤ٔ‫ٓخ ػظ‬ًٍٞ‫ ٓخ‬.d
Parmi les endroits suivants, lequel est le type de cancer qui donne le ١ٞ٤ٜ‫ ه‬١ٞ٘ٓ ٍّٝ .e
plus souvent lieu à des métastases hépatiques?
a. Adénocarcinome utérin
b. Carcinome épidermoïde de l‟œsophage
c. Adénocarcinome colorectal
d. Ostéosarcome
e. Séminome testiculaire

52
110. During acute renal failure, the hydro-electrolyte abnormality to look ًٌٝٗ ٕ‫ كب‬،‫ حُلخى‬١ٌُِٞ‫أػ٘خء حُلَ٘ ح‬
for urgently is: ٚ٘‫ـذ حُزلغ ػ‬٣ ١ٌُ‫ش ح‬٤‫ظخص حُٔخث‬٤ٌَُُٜ‫ح‬
:ٞٛ َ‫رٌَ٘ ػخؿ‬
a. Hyponatremia ّ‫ّ حُي‬ٞ٣‫ى‬ٞٛ ٚ‫ ٗو‬.a
b. Hyperkalemia ّ‫ّ حُي‬ٞ٤ٓ‫طخ‬ٞ‫ ر‬١َ‫ ك‬.b
c. Hypocalcemia ّ‫ّ حُي‬ٞ٤ُٔ‫ ًخ‬ٚ‫ ٗو‬.c
d. Hyperphosphatemia ّ‫ٓلخص حُي‬ٞ‫ ك‬١َ‫ ك‬.d
e. Global dehydration ّ‫ ؿلخف ػخ‬.e

Lors d‟insuffisance rénale aiguë, l‟anomalie hydroélectrolytique à


rechercher d‟urgence est :
a. Hyponatrémie
b. Hyperkaliémie
c. Hypocalcémie
d. Hyperphosphatémie
e. Déshydratation globale

111. In which ECG leads myocardial infarction by thrombosis of the ‫ حُوِذ‬٢٤‫خُ طوط‬ٜ‫ ؿ‬٢‫َ ك‬٤ٛٞ‫ ِٓي ط‬١‫ أ‬٢‫ك‬
proximal anterior descending artery manifests itself: ٖ‫ِش حُوِذ ػ‬٠‫ حكظ٘خء ػ‬٠ِ‫ظـ‬٣ ٠ُ‫ب‬٤‫َرخث‬ٌُٜ‫ح‬
٢ٓ‫خٕ حُ٘خٍُ حألٓخ‬٣َُ٘‫ ح‬٢‫ن هؼخٍ ك‬٣َ١
a. D2 D3 VF :‫ذ‬٣َ‫حُو‬
b. V1 V2 V3 V4 D2 D3 VF .a
c. V7 V8 V9 V1 V2 V3 V4 .b
d. D1 VL V7 V8 V9 .c
e. V5 V6 V7 D1 VL .d
V5 V6 V7 .e
Dans lequel l‟ECG conduit à un infarctus du myocarde par thrombose
de l‟artère descendante antérieure proximale se manifeste:
a. D2 D3 VF
b. V1 V2 V3 V4
c. V7 V8 V9
d. D1 VL
e. V5 V6 V7

53
112. What is the most probable etiology to evoke in front of voluminous ٍ‫خ‬٠‫ حُٔٔززخص حألًؼَ حكظٔخال ُالٓظل‬٢ٛ ‫ٓخ‬
asymptomatic bilateral hilar and mediastinal lymphadenopathies ‫ؤش‬٠ُ‫ش ح‬٤‫ش حُؼ٘خث‬٤‫أٓخّ حػظالٍ حُؼوي حُِٔل‬
associated with preservation of general condition and erythema ‫ش‬٤‫حػظالٍ حُؼوي حُِٔل‬ٝ ٝ‫ٔش حألػَح‬٣‫ػي‬
‫ حُلخُش‬٠ِ‫ش حَُٔطزطش رخُللخظ ػ‬٤‫ل‬ُٜ٘ٔ‫ح‬
nodosum discovered in a 25-year-old young woman?
‫ حَٓأس‬٢‫ش حٌُٔظ٘لش ك‬٣‫ حُؼوي‬٢ٓ‫حُلٔخ‬ٝ ‫حُؼخٓش‬
a. Tuberculosis ‫ ػخٓخ؟‬25 َٔ‫ٗخرش طزِؾ ٖٓ حُؼ‬
b. Sarcoidosis َُٔ‫ ح‬.a
c. Non-Hodgkin's lymphoma ‫ي‬٣ًٍٞ‫ حُٔخ‬.b
d. Hodgkin's disease ٖ٤ٌ‫ىؿ‬ٞٛ‫ٓخ حُال‬ٞ‫ٔل‬٤ُ .c
e. Virus ٖ٤ٌ‫ىؿ‬ٞٛ ‫ ىحء‬.d
َّٝ٤‫ ك‬.e
Quelle est l‟étiologie la plus probable à évoquer devant de
volumineuses lymphadénopathies hilaires et médiastinales bilatérales
asymptomatiques associées à la préservation de l‟état général et à
l‟érythème noueux découverts chez une jeune femme de 25 ans ?
a. Tuberculose
b. Sarcoïdose
c. Lymphome non hodgkinien
d. Maladie de Hodgkin
e. Virus

113. What is the most common cause of hearing loss in a previously ‫ػخ ُلويحٕ حُٔٔغ‬ٞ٤ٗ َ‫ حُٔزذ حألًؼ‬ٞٛ ‫ٓخ‬
healthy patient? ‫ْ ٓخروخ؟‬٤ِٓ ٞ٣َٓ ٟ‫ُي‬
a. Barotrauma ٢‫ـط‬٠ُ‫ق ح‬َُٟ‫ ح‬.a
b. Cerumen impaction ‫ٔخم‬ُٜ‫ حٗل٘خٍ ح‬.b
c. Neuronitis ‫خد‬ٜ‫خد حألػ‬ٜ‫ حُظ‬.c
d. Otitis media ٠‫ٓط‬ُٞ‫خد حألًٕ ح‬ٜ‫ حُظ‬.d
e. Tympanic membrane perforation ٢ِ‫ حٗؼوخد حُـ٘خء حُطز‬.e

Quelle est la cause la plus fréquente de perte auditive chez un patient


auparavant en bonne santé?
a. Barotraumatisme
b. Impaction du cérumen
c. Neuronite
d. Otite moyenne
e. Perforation de la membrane tympanique

54
114. Among the following drugs, which would potentially cause a problem ‫ش ٖٓ حُٔلظَٔ إٔ طٔزذ‬٤ُ‫ش حُظخ‬٣ٝ‫ ٖٓ حألى‬١‫أ‬
of impotence: :٢ٔ٘‫ٌِٓ٘ش حُؼـِ حُـ‬
a. Neuroleptic ٕ‫خ‬ٌُٛ‫خى ح‬٠ٓ .a
b. Antibiotic ١ٞ٤‫خى ك‬٠ٓ .b
c. Iron supplement ‫ي‬٣‫ ٓظٔٔخص حُلي‬.c
d. Calcium supplement ّٞ٤ُٔ‫ ٓظٔٔخص حٌُخ‬.d
e. Aspirin ٖ٣َ‫ حألٓز‬.e

Parmi les médicaments suivants, qui causeraient potentiellement un


problème d‟impuissance :
a. neuroleptique
b. Antibiotique
c. Supplément de fer
d. Supplément de calcium
e. Aspirine

115. A 70-year-old man presents to the emergency department with ٠ُ‫َ ا‬٠‫ل‬٣ ‫ ػخٓخ‬70 َٔ‫زِؾ ٖٓ حُؼ‬٣ َ‫ٍؿ‬
meningeal signs, headache, and fever. A biological blood test and a ‫يحع‬ٛٝ ‫ش‬٤‫حٍة رؼالٓخص ٓلخث‬ٞ‫هْٔ حُط‬
lumbar puncture are performed. On the cerebrospinal fluid: (by mm3) ٢‫ؿ‬ُٞٞ٤‫ حُيّ حُز‬ٚ‫ظْ اؿَحء كل‬٣ .٠ٔ‫ك‬ٝ
5,000 nucleated elements and 2 red blood cells, the formula found :٢‫ حُٔخثَ حُ٘وخػ‬٠ِ‫ ػ‬.٢٘‫حُزٍِ حُوط‬ٝ
98% of neutrophils. Proteinorachia is at 1.67 g / L and glycorachia at ‫خ‬٣‫ هال‬2ٝ ‫حس‬ٞٗ َٜ٘‫ ػ‬5000 )ْٓ3 ٍ‫(رٔويح‬
2.3 mmol / L. The current blood glucose is 7.5 mmol / L. What is the ٖٓ ٪98 ‫ـش‬٤ُٜ‫ؿيص ح‬ٝ ،‫ىّ كَٔحء‬
most likely diagnosis? َ‫ ُظ‬/ ْ‫ ؿ‬1.67 ‫ٖ ػ٘ي‬٤‫ط‬َٝ‫ حُز‬.‫حُٔؼظيالص‬
a. Herpetic meningoencephalitis ‫ ٗٔزش‬.َ‫ ُظ‬/ ٍٞٔ٤ِٓ 2.3 ‫ُػ٘ي‬ًِٞٞ‫حُـ‬ٝ
b. Bacterial meningitis / ٍٞٔ٤ِٓ 7.5 ٢ٛ ‫ش‬٤ُ‫ حُيّ حُلخ‬٢‫ُ ك‬ًِٞٞ‫حُـ‬
c. Viral encephalitis ‫ حألًؼَ حكظٔخال؟‬ٚ٤‫ حُظ٘و‬ٞٛ ‫ ٓخ‬.َ‫ُظ‬
d. Cerebral toxoplasmosis ٢ٔ‫َر‬ُٜ‫حُيٓخؽ ح‬ٝ ‫خ‬٣‫خد حُٔلخ‬ٜ‫ حُظ‬.a
e. Guillain-Barré syndrome ٢ٓٞ‫خ حُـَػ‬٣‫خد حُٔلخ‬ٜ‫ حُظ‬.b
٢َٓٝ٤‫خد حُيٓخؽ حُل‬ٜ‫ حُظ‬.c
Un homme de 70 ans se présente à l‟urgence avec des signes ٢‫ٓخص حُيٓخؿ‬ٞ‫ ىحء حُٔو‬.d
méningés, des maux de tête et de la fièvre. Un test sanguin biologique ٚ٣ٍ‫الٕ رخ‬٤‫ ٓظالُٓش ؿ‬.e
et une ponction lombaire sont effectués. Sur le liquide céphalo-
rachidien : (par mm3) 5 000 éléments nucléés et 2 globules rouges, la
formule a trouvé 98% des neutrophiles. Proteinorachia est à 1,67 g / L
et glycorachia à 2,3 mmol / L. La glycémie actuelle est de 7,5 mmol /
L. Quel est le diagnostic le plus probable?
a. Méningo-encéphalite herpétique
b. Méningite bactérienne
c. Encéphalite virale
d. Toxoplasmose cérébrale
e. Syndrome de Guillain-Barré

55
116. A 68-year-old man presents with haematuria, dysuria and urinary ‫ِش‬٤‫ ٖٓ ر‬٢ٗ‫ؼخ‬٣ ‫ ػخٓخ‬68 َٔ‫زِؾ ٖٓ حُؼ‬٣ َ‫ٍؿ‬
urgency for 72 hours. Clinical examination shows an enlarged and 72 ‫ٍ ُٔيس‬ٞ‫اُلخف حُز‬ٝ ٍٞ‫ػَٔ حُز‬ٝ ‫ش‬٣ٞٓ‫ى‬
tender prostate. Cystoscopy shows urethral stricture and hemorrhagic ْ‫و‬٠‫ ط‬١َ٣َُٔ‫ ح‬ٚ‫َ حُلل‬ٜ‫ظ‬٣ .‫ٓخػش‬
َٜ‫ظ‬٣ ‫َ حُٔؼخٗش‬٤‫ ط٘ظ‬.‫خ‬ٜٓ‫ال‬٣‫ا‬ٝ ‫ٓظخص‬َٝ‫حُز‬
cystitis. One hour after the cytoscopy, he has chills, with fever and
.٢‫خد حُٔؼخٗش حُِ٘ك‬ٜ‫حُظ‬ٝ ٍٞ‫ حُز‬َٟ‫ن ٓـ‬٤٠‫ط‬
drowsiness. Her temperature is 35.6°C, her blood pressure is 60/20 ٢ٗ‫ؼخ‬٣ ،١ِٞ‫َ حُو‬٤‫حكيس ٖٓ حُظ٘ظ‬ٝ ‫رؼي ٓخػش‬
mmHg, her heart rate is 120/min and her respiratory rate is 24/min. ‫ ىٍؿش‬.ّ‫حُ٘ؼخ‬ٝ ٠ٔ‫ ٓغ حُل‬،‫َس‬٣َ‫ٖٓ ه٘ؼ‬
His skin is cold, mottled and cyanotic. He is delusional. A Foley ٚٓ‫ ى‬٢‫ـ‬ٟٝ ،‫ش‬٣ٞ‫ ىٍؿش ٓج‬35.6 ٚ‫كَحٍط‬
catheter is placed. What is the most likely diagnosis? ‫خ‬ٜ‫َرخص هِز‬ٟ ٍ‫ٓؼي‬ٝ ،‫ ْٓ ُثزن‬20/60
ٙ‫ ؿِي‬.‫وش‬٤‫ ىه‬/ 24 ٚٔ‫ٓؼيٍ ط٘ل‬ٝ ‫وش‬٤‫ ىه‬/ 120
a. Acute gastrointestinal bleeding ‫غ‬ٟٝ ْ‫ظ‬٣ .ْٛ‫ح‬ٝ ٚٗ‫ ا‬.‫ٍِٓم‬ٝ ٖ‫َٓه‬ٝ ‫رخٍى‬
b. Acute myocardial infarction َ‫ حألًؼ‬ٚ٤‫ حُظ٘و‬ٞٛ ‫ ٓخ‬.٢ُٞ‫هٔطَس ك‬
c. Septic shock ‫لخ؟‬٤‫طَؿ‬
d. Pulmonary embolism ‫ كخى‬١ٞ‫ ٓؼ‬١‫ق ٓؼي‬٣ِٗ .a
e. Bladder rupture ‫ش كخى‬٤‫ِش هِز‬٠‫ حكظ٘خء ػ‬.b
‫ش‬٤ٗ‫يٓش اٗظخ‬ٛ .c
Un homme de 68 ans présente une hématurie, une dysurie et une ١ٞ‫ٔخّ ٍث‬ٜٗ‫ ح‬.d
urgence urinaire pendant 72 heures. L‟examen clinique montre une ‫ طِٔم حُٔؼخٗش‬.e
prostate élargie et sensible. La cystoscopie montre une sténose
urétrale et une cystite hémorragique. Une heure après la cytoscopie, il
a des frissons, de la fièvre et de la somnolence. Sa température est de
35,6 °C, sa tension artérielle est de 60/20 mmHg, sa fréquence
cardiaque est de 120/min et sa fréquence respiratoire est de 24/min.
Sa peau est froide, tachetée et cyanosée. Il délire. Un cathéter Foley
est placé. Quel est le diagnostic le plus probable?
a. Saignement gastro-intestinal aigu
b. Infarctus aigu du myocarde
c. Choc septique
d. Embolie pulmonaire
e. Rupture de la vessie

56
117. A construction worker was working outside on a job site and forgot to ‫هغ‬ٞٓ ٢‫ حُوخٍؽ ك‬٢‫ؼَٔ ك‬٣ ‫ًخٕ ػخَٓ ر٘خء‬
wear his steel toe boot. He stepped on a rusty nail that went through ّ‫ ىح‬.١ً‫ال‬ٞ‫ حُل‬ٚ‫ حٍطيحء كٌحث‬٢ٔٗٝ َٔ‫ػ‬
his shoe and pierced his foot resulting in a dirty wound. Patient states ‫حهظَم‬ٝ ٚ‫ية حهظَم كٌحث‬ٛ ٍ‫ ٓٔٔخ‬٠ِ‫ػ‬
ًٌَ٣ .‫ ؿَف ٓظٔن‬٠ُ‫ ا‬ٟ‫ ٓٔخ أى‬ٚٓ‫هي‬
that he does not think that he had a tetanus shot in 20+ years and does
‫ ُوخف‬٠ِ‫َ ػ‬ٜ‫ ك‬ٚٗ‫ؼظوي أ‬٣ ‫ ال‬ٚٗ‫ أ‬ٞ٣َُٔ‫ح‬
not think he received the initial three vaccines as a child. In addition ٠‫ طِو‬ٚٗ‫ؼظوي أ‬٣ ‫ال‬ٝ + 20 ‫حص‬ٞ٘ٓ ٢‫حٌُِحُ ك‬
to X-ray and appropriate antibiotic therapy, which of the following is .‫لال‬١ ٕ‫ش ػ٘يٓخ ًخ‬٤ُٝ‫حُِوخكخص حُؼالػش حأل‬
correct? ‫حُؼالؽ‬ٝ ‫ش‬٤٘٤ُٔ‫ حألٗؼش ح‬٠ُ‫خكش ا‬ٟ‫رخإل‬
a. Give patient tetanus toxoid 0.5 ml and 250 unit tetanus ٢ُ‫ ٖٓ حُظخ‬١‫ أ‬،‫ش‬٣ٞ٤‫خىحص حُل‬٠ُٔ‫حُٔ٘خٓذ رخ‬
immunoglobulin ‫ق؟‬٤‫ل‬ٛ
b. Give patient tetanus toxoid 0.5 ml 0.5 ُ‫لخٕ حٌُِح‬٣ً ٞ٣َُٔ‫ اػطخء ح‬.a
ٖ٤ُٞ‫ر‬ِٞ‫كيس حُـ‬ٝ 250ٝ َٓ
c. Give patient nothing as you do not treat until symptoms begin
ُ‫ ٌُِِح‬٢‫حُٔ٘خػ‬
d. Give patient immunoglobulin 250 unit alone ُ‫لخٕ حٌُِح‬٣ً ٞ٣َُٔ‫ اػطخء ح‬.b
َٓ 0.5
Un travailleur de la construction travaillait à l‟extérieur sur un ‫ء ألٗي‬٢ٗ ١‫ أ‬ٞ٣َُٔ‫ ح‬٢‫ ال طؼط‬.c
chantier et a oublié de porter sa botte à embout d‟acier. Il a marché ٝ‫ طزيأ حألػَح‬٠‫ال طؼخُؾ كظ‬
sur un clou rouillé qui a traversé sa chaussure et lui a percé le pied, ce ٖ٤ُٞ‫ر‬ِٞ‫ حُـ‬ٞ٣َُٔ‫ اػطخء ح‬.d
qui lui a causé une blessure sale. Le patient déclare qu‟il ne pense pas ‫خ‬ٛ‫كي‬ٝ ‫كيس‬ٝ 250 ٢‫حُٔ٘خػ‬
qu‟il a eu un vaccin contre le tétanos en 20+ ans et ne pense pas qu‟il
a reçu les trois premiers vaccins dans son enfance. En plus de la
radiographie et de l‟antibiothérapie appropriée, lequel des éléments
suivants est correct?
a. Donnez au patient 0,5 ml d‟anatoxine tétanique et 250
unités d‟immunoglobuline antitétanique
b. Donnez au patient 0,5 ml d‟anatoxine tétanique
c. Ne donnez rien au patient car vous ne traitez pas jusqu‟à
l‟apparition des symptômes
d. Donnez au patient 250 unités d‟immunoglobuline seule

57
118. A 4- year- old child is brought to the emergency room by her parents ‫حص‬ٞ٘ٓ 4 َٔ‫لِش طزِؾ ٖٓ حُؼ‬١ ٍ‫خ‬٠‫ظْ اك‬٣
on a Friday night because they are concerned about rabies. A bat was ‫ِش‬٤ُ ‫خ‬ٜ٣‫حُي‬ٝ َ‫حٍة ٖٓ هز‬ٞ‫ ؿَكش حُط‬٠ُ‫ا‬
present in the child‟s bedroom when they arrived at their country ٕ‫ ًخ‬.‫ٕ ر٘ؤٕ ىحء حٌُِذ‬ٞ‫ْ هِو‬ٜٗ‫حُـٔؼش أل‬
‫ّ حُطلِش ػ٘يٓخ‬ٞٗ ‫ ؿَكش‬٢‫ىح ك‬ٞ‫ؿ‬ٞٓ ٕ‫حُولخ‬
home that evening. It started flying around the head of the girl when
.‫ ًُي حُٔٔخء‬٢‫ ك‬٢‫ل‬٣َُ‫ْ ح‬ُِٜ٘ٓ ٠ُ‫ح ا‬ِٞٛٝ
she entered her room and it ruffled her hair. The parents heard her ‫ٍ ٍأّ حُلظخس ػ٘يٓخ ىهِض‬ٞ‫َ ك‬٣‫ظطخ‬٣ ‫ريأ‬
scream, ran up to her room, and shooed the bat out the window. ٕ‫حُيح‬ُٞ‫ ٓٔغ ح‬.‫خ‬َٛ‫أُػؾ ٗؼ‬ٝ ‫خ‬ٜ‫ؿَكظ‬
Which is the most appropriate intervention at this time? ‫أهَؿخ‬ٝ ،‫خ‬ٜ‫ ؿَكظ‬٠ُ‫خ ا‬٠ًٍٝ ،‫خ‬ٜ‫َحه‬ٛ
a. Administer rabies immunoglobulin (RIG) only ٢‫ حُظيهَ حألٗٔذ ك‬ٞٛ ‫ ٓخ‬.‫حُولخٕ ٖٓ حُ٘خكٌس‬
b. Administer rabies vaccine and rabies immunoglobulin (RIG) ‫هض؟‬ُٞ‫ٌح ح‬ٛ
c. Reassure the parents that there is no risk of rabies ‫ ُيحء‬٢‫ٖ حُٔ٘خػ‬٤ُٞ‫ر‬ِٞ‫ اػطخء حُـ‬.a
٢‫) كو‬RIG(
d. Administer rabies vaccine only
‫ اػطخء ُوخف ىحء حٌُِذ‬.b
‫ ُيحء‬٢‫ٖ حُٔ٘خػ‬٤ُٞ‫ر‬ِٞ‫حُـ‬ٝ
)RIG(
Un enfant de 4 ans est amené à l‟urgence par ses parents un vendredi ‫ؿي‬ٞ٣ ‫ ال‬ٚٗ‫ٖ رؤ‬٣‫حُي‬ُٞ‫ٔؤٗش ح‬١ .c
soir parce qu‟ils sont préoccupés par la rage. Une chauve-souris était ‫هطَ ٖٓ ىحء حٌُِذ‬
présente dans la chambre de l‟enfant lorsqu‟ils sont arrivés à leur ٢‫ اػطخء ُوخف ىحء حٌُِذ كو‬.d
maison de campagne ce soir-là. Il a commencé à voler autour de la
tête de la fille quand elle est entrée dans sa chambre et cela a
ébouriffé ses cheveux. Les parents l‟ont entendue crier, ont couru
jusqu‟à sa chambre et ont chassé la chauve-souris par la fenêtre.
Quelle est l‟intervention la plus appropriée en ce moment?
a. Administrer l‟immunoglobuline antirabique (RIG)
seulement
b. Administrer le vaccin antirabique et l‟immunoglobuline
antirabique (RIG)
c. Rassurer les parents qu‟il n‟y a aucun risque de rage
d. Administrer le vaccin antirabique seulement

58
119. A 28-year-old man is evaluated for 2 days history of fever and chills. ‫ ػخٓخ ُٔيس‬28 َٔ‫زِؾ ٖٓ حُؼ‬٣ َ‫ْ ٍؿ‬٤٤‫ظْ طو‬٣
He denies any cough, sputum production, abdominal pain, diarrhea or ٢‫٘ل‬٣ٝ .‫َس‬٣َ‫حُو٘ؼ‬ٝ ٠ٔ‫ن حُل‬٣ٍ‫ٖ ٖٓ طخ‬٤ٓٞ٣
dysuria. He is known to have Hodgkin‟s lymphoma, received a cycle ٝ‫ حُزطٖ أ‬٢‫ آالّ ك‬ٝ‫ اٗظخؽ حُو٘غ أ‬ٝ‫ ٓؼخٍ أ‬١‫أ‬
ٚٗ‫ف أ‬َٝ‫ ٖٓ حُٔؼ‬.ٍٞ‫ ػَٔ حُز‬ٝ‫خٍ أ‬ٜٓ‫ا‬
of chemotherapy 5 days ago. He is febrile 38.5 and blood pressure
،ٖ٤ٌ‫ىؿ‬ٞٛ ‫ش‬٣ٝ‫ٔلخ‬٤ُِ‫خٕ حُـيى ح‬١َٔ‫خد ر‬ٜٓ
123/67. Physical exam showed a normal oral cavity and skin, normal .ّ‫خ‬٣‫ أ‬5 َ‫ هز‬٢‫خث‬٤ٔ٤ٌُ‫ٍس ٖٓ حُؼالؽ ح‬ٝ‫ ى‬٠‫طِو‬ٝ
cardiopulmonary and abdominal exam Labs tests showed WBC=1500 َٜ‫ أظ‬.67/123 ّ‫ حُي‬٢‫ـ‬ٟٝ 38.5 ‫حُلَحٍس‬
cells/mm3 with absolute neutrophilic count of 200 cells/mm3. Blood ،٢‫ؼ‬٤‫ز‬١ ‫حُـِي‬ٝ ْ‫ف حُل‬ٞ‫ إٔ ؿ‬٢ٗ‫ حُزي‬ٚ‫حُلل‬
and urine cultures are collected. Which of the following is the most ٢‫ؼ‬٤‫ز‬١ ٢٘‫حُزط‬ٝ ١ٞ‫ حَُث‬٢‫ حُوِز‬ٚ‫حُلل‬ٝ
appropriate initial empirical therapy pending the results of the WBC = ٕ‫ش أ‬٣َ‫ حُٔوز‬ٙٞ‫َص حُلل‬ٜ‫أظ‬
cultures? ‫ ٓغ ػيى حُٔؼظيالص‬3 ْٓ / ‫ش‬٤ِ‫ ه‬1500
‫ظْ ؿٔغ ٍُع‬٣ .3 ْٓ / ‫ش‬٤ِ‫ ه‬200 ‫حُٔطِن‬
a. Piperacillin/tazobactam
‫ أٗٔذ ػالؽ‬ٞٛ ٢ِ٣ ‫ ٓٔخ‬١‫ أ‬.ٍٞ‫حُز‬ٝ ّ‫حُي‬
b. Meropenem and micafungin ‫ حٗظظخٍ ٗظخثؾ حٍُِع؟‬٢‫ ك‬٢ُٝ‫ أ‬٢‫ز‬٣َ‫طـ‬
c. Cefepime and vancomycin ّ‫رخًظخ‬ُٝ‫ طخ‬/ ٖ٤ِ٤ٓ‫َح‬٤‫ز‬٤‫ ر‬.a
d. Linezolid ٖ٤‫ٗـ‬ٞ‫ٌخك‬٤ٓٝ ْ٤٘٤‫ر‬َٝ٤ٓ .b
ٖ٤ٔ٣‫ٓخ‬ٌٞٗ‫كخ‬ٝ ْ٤‫ز‬٤‫ل‬٤ٓ .c
Un homme de 28 ans est évalué pour 2 jours d‟antécédents de fièvre ‫ي‬٤ُِٝ٘٤ُ .d
et de frissons. Il nie toute toux, production d‟expectorations, douleurs
abdominales, diarrhée ou dysurie. Il est connu pour avoir un
lymphome de Hodgkin, a reçu un cycle de chimiothérapie il y a 5
jours. Il est fébrile 38,5 et tension artérielle 123/67. L‟examen
physique a montré une cavité buccale et une peau normales, un
examen cardiopulmonaire et abdominal normal Les tests de
laboratoire ont montré WBC = 1500 cellules / mm3 avec un nombre
absolu de neutrophiles de 200 cellules / mm3. Des cultures de sang et
d‟urine sont prélevées. Laquelle des options suivantes est la thérapie
empirique initiale la plus appropriée en attendant les résultats des
cultures?
a. Pipéracilline/tazobactam
b. Méropénème et micafonguine
c. Céfépime et vancomycine
d. Linézolide

59
120. A 72-year-old man presents to the emergency room with 3-day ٠ُ‫َ ا‬٠‫ل‬٣ ‫ ػخٓخ‬72 َٔ‫زِؾ ٖٓ حُؼ‬٣ َ‫ٍؿ‬
history of high-grade fever, chills and dyspnea. He reports productive ٖٓ ّ‫خ‬٣‫ أ‬3 ‫ن ُٔيس‬٣ٍ‫حٍة ٓغ طخ‬ٞ‫ؿَكش حُط‬
cough and decrease urine output for the last 3 days. Upon .ْ‫ن حُظ٘ل‬٤ٟٝ ‫َس‬٣َ‫حُو٘ؼ‬ٝ ‫يس‬٣‫ حُ٘ي‬٠ٔ‫حُل‬
ٍٞ‫ اٗظخؽ حُز‬ٝ‫حٗولخ‬ٝ ‫أرِؾ ػٖ حُٔؼخٍ حُٔ٘ظؾ‬
presentation, patient was ill looking and lethargic. His body weight is
ٞ٣َُٔ‫ ًخٕ ح‬،َٝ‫ ػ٘ي حُؼ‬.ّ‫خ‬٣‫ أ‬3 َ‫ه‬٥
70 kg. Vital signs are temperature 39°C, BP= 83/45, HR=115 bpm, .ْ‫ ًـ‬70 ٚٔٔ‫ُٕ ؿ‬ٝ .‫هخٓال‬ٝ َٜ‫ت حُٔظ‬٤ٓ
RR= 27 bpm and SpO2= 91% on 5l/min oxygen. Physical exam was 39 ‫ ىٍؿش حُلَحٍس‬٢ٛ ‫ش‬٣ٞ٤‫حُؼالٓخص حُل‬
significant for localized crackles at the right lung base and clear left HR = 115 ،BP = 83/45 ،‫ش‬٣ٞ‫ىٍؿش ٓج‬
lung, and lower extremities skin mottling with no ٢‫ش ك‬٠‫ ٗز‬RR = 27 ،‫وش‬٤‫ حُيه‬٢‫ش ك‬٠‫ٗز‬
edema.LABORATORY TESTS: * WBC=15000 cells/mm3 * ‫وش‬٤‫ ىه‬/ َ‫ ُظ‬5 ٠ِ‫ ػ‬٪SpO2 = 91 ٝ ‫وش‬٤‫حُيه‬
Hemoglobin 11 g/dL * Platelets 90000/mm3 * Creatinine = ‫ رخٍُح ًٌََُس‬٢ٗ‫ حُزي‬ٚ‫ ًخٕ حُلل‬.ٖ٤‫أًٔـ‬
‫حَُثش‬ٝ ٠٘ٔ٤ُ‫ هخػيس حَُثش ح‬٢‫ش ك‬٤‫ؼ‬ٟٞٓ
2.4 mg/dl * Lactate=6 mmol/LChest x-ray showed right lower
‫ش‬٤ِ‫َحف حُٔل‬١‫طزوغ ؿِي حأل‬ٝ ،‫ش‬٤‫خك‬ٛ َٟٔ٤ُ‫ح‬
consolidation.In addition to starting IV antibiotics, what is the most ‫ * ًَحص‬:‫ش‬٣َ‫ حُٔوز‬ٙٞ‫ حُلل‬.‫ًٓش‬ٝ ٕٝ‫ري‬
appropriate next step in managing this patient? * 3 ْٓ / ‫ش‬٤ِ‫ ه‬15000 = ‫خء‬٠٤‫حُيّ حُز‬
a. Infuse 2L of 4% albumin solution ‫لخثق‬ُٜ‫ِظَ * ح‬٤ٔ٣‫ ى‬/ ْ‫ ؿ‬11 ٖ٤‫ر‬ِٞ‫ؿ‬ٞٔ٤ُٜ‫ح‬
b. Infuse 2L of lactated ringer‟s solution = ٖ٤٘٤‫خط‬٣ٌَُ‫ * ح‬3 ْٓ / 90000 ‫ش‬٣ٞٓ‫حُي‬
c. Start IV norepinephrine ٍٞٔ٤ِٓ 6 = ‫ِظَ * حُالًظخص‬٤ٔ٣‫ ى‬/ ْ‫ ٓـ‬2.4
d. Start IV hydrocortisone ‫ِيح‬ٜ‫يٍ ط‬ُِٜ ‫ش‬٤٘٤ُٔ‫َص حألٗؼش ح‬ٜ‫ أظ‬.ٍ /
‫ ريء‬٠ُ‫خكش ا‬ٟ‫ رخإل‬.‫ش‬٤ِ‫ حُٔل‬٠٘ٔ٤ُ‫ش ح‬ٜ‫ حُـ‬٢‫ك‬
‫س‬ٞ‫ حُوط‬٢ٛ ‫ ٓخ‬،‫ش‬٣‫ي‬٣ٍُٞ‫ش ح‬٣ٞ٤‫خىحص حُل‬٠ُٔ‫ح‬
Un homme de 72 ans se présente à la salle d‟urgence avec des ‫؟‬ٞ٣َُٔ‫ٌح ح‬ٛ َ٤‫ طير‬٢‫ش حألٗٔذ ك‬٤ُ‫حُظخ‬
antécédents de fièvre, de frissons et de dyspnée pendant 3 jours. Il ٍِٞ‫ ُظَ ٖٓ ٓل‬2 ‫ذ‬٣َٔ‫ ط‬.a
rapporte une toux productive et une diminution de la production ٪4 ٖ٤ٓٞ‫حألُز‬
d‟urine au cours des 3 derniers jours. Lors de la présentation, le َ‫٘ـ‬٣ٍ ٍِٞ‫ٓل‬ ٖٓ َ‫ُظ‬ 2 ‫ذ‬٣َٔ‫ ط‬.b
patient était mal regardé et léthargique. Son poids corporel est de 70 ‫الًظخص‬
١‫ي‬٣ٍُٞ‫ٖ ح‬٣َ‫٘ل‬٤‫ٍار‬ُٞ٘‫ حُزيء رخ‬.c
kg. Les signes vitaux sont la température 39°C, BP = 83/45, HR = 115
١‫ي‬٣ٍُٞ‫ٕ ح‬ِٝ٤‫ٍط‬ًٍٞٝ‫ي‬٤ُٜ‫ حُزيء رخ‬.d
bpm, RR = 27 bpm et SpO2= 91% avec 5l/min d‟oxygène. L‟examen
physique était significatif pour les crépitements localisés à la base du
poumon droit et du poumon gauche clair, et les marbrures cutanées
des membres inférieurs sans œdème. TESTS DE LABORATOIRE: *
GLOBULES BLANCS = 15000 cellules / mm3 * Hémoglobine 11 g /
dL * Plaquettes 90000 / mm3 * Créatinine = 2,4 mg / dl * Lactate = 6
mmol / LCla radiographie a montré une consolidation inférieure
droite En plus de commencer les antibiotiques IV, quelle est la
prochaine étape la plus appropriée dans la prise en charge de ce
patient?
a. Infuser 2L de solution d‟albumine à 4%
b. Infuser 2L de solution de sonneur lactate
c. Début de la noradrénaline IV
d. Début de l‟hydrocortisone IV

60
121. A 65-year-old man presents due to retrosternal oppressive chest pain ‫َ رٔزذ‬٠‫ل‬٣ ‫ ػخٓخ‬65 َٔ‫زِؾ ٖٓ حُؼ‬٣ َ‫ٍؿ‬
of 3 months duration. The pain is episodic mainly precipitated by .َٜٗ‫ أ‬3 ‫ ُٔيس‬ٚ‫يٍ ٓئُْ هِق حُو‬ٛ ‫ن‬٤ٟ
exertion and subsides after 2-5 minutes of rest. He underwent stress ٌٍ‫ ػ٘ي ر‬٢ٔ٤‫ليع رٌَ٘ ٍث‬٣ ٢َٟ‫حألُْ ػ‬
.‫ ىهخثن ٖٓ حَُحكش‬5-2 ‫يأ رؼي‬ٜ٣ٝ ‫ي‬ٜ‫حُـ‬
electrocardiogram. 10 minutes after starting the test, he has the same
‫ رؼي‬.٢‫َرخث‬ٌُٜ‫ ح‬٢‫ي حُوِز‬ٜ‫ حُـ‬ٚ‫غ ُلل‬٠‫ه‬
chest pain and his EKG shows 2-mm ST depression and T-wave ٖٓ ٢ٗ‫ؼخ‬٣ ‫ ريأ‬،ٍ‫ ىهخثن ٖٓ ريء حالهظزخ‬10
inversion in leads II, III, aVF. Patient is planned for coronary ‫ش حُوِذ‬٤‫َر‬ًٜ ٢‫َ ٓوط‬ٜ‫ظ‬٣ٝ ٍ‫ي‬ُٜ‫ٗلْ أُْ ح‬
angiography. The coronary angiography will most probably show ّ‫حٗؼٌخ‬ٝ ْٓ2 ST ٝ‫ حٗولخ‬ٚ‫ ر‬ٙ‫حُوخ‬
stenosis of which of the following vessels? ْ‫ظ‬٣ .aVFٝ IIIٝ II ‫ حألٓالى‬٢‫ ك‬T ‫ؿش‬ٞٓ
a. Circumflex artery .‫ش‬٤‫ش حُظخؿ‬٤‫ػ‬ٝ‫َ حأل‬٣ٜٞ‫ ُظ‬ٞ٣َُِٔ ٢٤‫حُظوط‬
b. Right coronary artery ‫ش‬٤‫ػ‬ٝ‫َ حأل‬٣ٜٞ‫َ ط‬ٜ‫ظ‬٣ ٕ‫ٖٓ حَُٔؿق أ‬
‫ش؟‬٤ُ‫ش حُظخ‬٤‫ػ‬ٝ‫ ٖٓ حأل‬١‫ن أ‬٤٠‫ش ط‬٤‫حُظخؿ‬
c. Left main coronary artery
٢‫ط‬٤‫خٕ حُٔل‬٣َُ٘‫ ح‬.a
d. Left anterior descending artery ٖٔ٣‫ حأل‬٢‫خٕ حُظخؿ‬٣َُ٘‫ ح‬.b
َٔ٣‫ حأل‬٢ٔ٤‫ حَُث‬٢‫خٕ حُظخؿ‬٣َُ٘‫ ح‬.c
Un homme de 65 ans se présente en raison d‟une douleur thoracique ٍُ‫َٔ حُ٘خ‬٣‫ حأل‬٢ٓ‫خٕ حألٓخ‬٣َُ٘‫ ح‬.d
oppressante rétrosternale d‟une durée de 3 mois. La douleur est
épisodique principalement précipitée par l‟effort et disparaît après 2-5
minutes de repos. Il a subi un électrocardiogramme d‟effort. 10
minutes après le début du test, il a la même douleur thoracique et son
électrocardiogramme montre une dépression ST de 2 mm et une
inversion d‟onde T dans les dérivations II, III, aVF. Le patient est
prévu pour une coronarographie. L‟angiographie coronaire montrera
très probablement une sténose de lequel des vaisseaux suivants?
a. Artère circonflexe
b. Artère coronaire droite
c. Artère coronaire principale gauche
d. Artère descendante antérieure gauche

61
122. A 62-year-old man presents to the emergency department with sudden ٠ُ‫َ ا‬٠‫ل‬٣ ‫ ػخٓخ‬62 َٔ‫زِؾ ٖٓ حُؼ‬٣ َ‫ٍؿ‬
onset of severe chest pain, radiating to the neck and arms and ٢‫ي ك‬٣‫ش ٓلخؿجش ألُْ ٗي‬٣‫حٍة ٓغ ريح‬ٞ‫هْٔ حُط‬
associated with diaphoresis and dizziness. On physical exam, patient ٖ٤‫حٌٍُحػ‬ٝ ‫ حَُهزش‬ٞ‫ِظق ٗل‬٣ ،ٍ‫ي‬ُٜ‫ح‬
٢‫ ك‬.‫هش‬ٝ‫ى‬ٝ َ٣ِ‫ظَحكن ٓغ طؼَم ؿ‬٣ٝ
is in pain. His vital signs are blood pressure 89/55, heart rate 60 bpm,
.ُْ‫ ٖٓ حأل‬ٞ٣َُٔ‫ ح‬٢ٗ‫ؼخ‬٣ ،٢ٗ‫ حُزي‬ٚ‫حُلل‬
and oxygen saturation 96% on room air. The rest of physical exam is ،55/89 ّ‫ حُي‬٢‫ـ‬ٟ ٢ٛ ‫ش‬٣ٞ٤‫ حُل‬ٚ‫ػالٓخط‬
normal. EKG showed ST elevation in I, aVL, V5 and V6 and normal ،‫وش‬٤‫ حُيه‬٢‫ش ك‬٠‫ ٗز‬60 ‫َرخص حُوِذ‬ٟ ٍ‫ٓؼي‬ٝ
PR interval. The main underlying pathogenic process of this patient‟s .‫حء حُـَكش‬ٞٛ ٠ِ‫ ػ‬٪96 ٖ٤‫ط٘زغ حألًٔـ‬ٝ
presentation is: ٢‫َ ٓوط‬ٜ‫ أظ‬.٢‫ؼ‬٤‫ز‬١ ٢ٗ‫ حُزي‬ٚ‫ش حُلل‬٤‫رو‬
a. Coronary artery stenosis 5Vٝ aVLٝ I ٢‫ ك‬ST ‫ش حُوِذ حٍطلخع‬٤‫َر‬ًٜ
b. Rupture of coronary artery atheroma ‫ش‬٤ٔ٤‫ش حَُث‬٤ِٔ‫ حُؼ‬.‫ش‬٤‫ؼ‬٤‫ز‬١ PR ‫كظَس‬ٝ 6Vٝ
ٞ٣َُٔ‫ٌح ح‬ٛ ‫ ػ٘ي‬٢َُٟٔ‫ ح‬َٝ‫حُٔٔززش ُِؼ‬
c. Coronary artery spasm
:٢ٛ
d. Inflammation of the pericardium ٢‫خٕ حُظخؿ‬٣َُ٘‫ن ح‬٤٠‫ ط‬.a
٢‫خٕ حُظخؿ‬٣َُ٘‫يس ح‬٤ٜ‫ طِٔم ػ‬.b
٢‫خٕ حُظخؿ‬٣َُ٘‫ ط٘٘ؾ ح‬.c
Un homme de 62 ans se présente à l‟urgence avec l‟apparition ٍٞٓ‫خد حُظخ‬ٜ‫ حُظ‬.d
soudaine d‟une douleur thoracique intense, irradiant vers le cou et les
bras et associée à une diaphorèse et à des étourdissements. À
l‟examen physique, le patient ressent de la douleur. Ses signes vitaux
sont une pression artérielle de 89/55, une fréquence cardiaque de 60
bpm et une saturation en oxygène de 96% sur l‟air ambiant. Le reste
de l‟examen physique est normal. L‟ECG a montré une élévation ST
dans I, aVL, V5 et V6 et un intervalle PR normal. Le principal
processus pathogène sous-jacent de la présentation de ce patient est:
a. Sténose de l‟artère coronaire
b. Rupture de l‟athérome de l‟artère coronaire
c. Spasme de l‟artère coronaire
d. Inflammation du péricarde

62
123. A 55-year-old woman presents with 3 weeks of fever and malaise. 3 ٌ٘ٓ ٢ٗ‫ ػخٓخ طؼخ‬55 َٔ‫حَٓأس طزِؾ ٖٓ حُؼ‬
She also reports worsening dyspnea. She is known to have moderate ‫ ًٔخ‬.‫ػي‬ٞ‫ٍ رخُظ‬ٞ‫حُ٘ؼ‬ٝ ٠ٔ‫غ ٖٓ حُل‬٤‫أٓخر‬
aortic stenosis due to bicuspid aortic valve. She was previously ‫ف‬َٝ‫ ٖٓ حُٔؼ‬.ْ‫ن حُظ٘ل‬٤ٟ ْ‫أرِـض ػٖ طلخه‬
َٜ‫ حألر‬٢‫ ك‬٢ٓٞ‫ن ٓظ‬٤٠‫ ٖٓ ط‬٢ٗ‫خ طؼخ‬ٜٗ‫أ‬
asymptomatic. On physical examination, temperature 38.1°C, heart
‫ ًخٗض‬.‫ حَُ٘ف‬٢‫ ػ٘خث‬١َٜ‫ٔخّ حألر‬ُٜ‫رٔزذ ح‬
rate 90 bpm, blood pressure 110/67. Cardiac examination is ٚ‫ ػ٘ي حُلل‬.ٝ‫ٕ أػَح‬ٝ‫ حُٔخرن ري‬٢‫ك‬
significant for systolic murmur on the right upper sternal border that ،‫ش‬٣ٞ‫ ىٍؿش ٓج‬38.1 ‫ ىٍؿش حُلَحٍس‬،٢ٗ‫حُزي‬
has increased in intensity since the last visit. Transthoracic ،‫وش‬٤‫ حُيه‬٢‫ش ك‬٠‫ ٗز‬90 ‫َرخص حُوِذ‬ٟ ٍ‫ٓؼي‬
echocardiography shows a mobile mass on the aortic valve with new ‫ حُوِذ‬ٚ‫َ كل‬ٜ‫ظ‬٣ .67/110 ّ‫ حُي‬٢‫ـ‬ٟ
aortic regurgitation. Blood cultures results are pending. Awaiting ‫ش‬٣ِٞ‫ش حُؼ‬٤ٜ‫ى حُو‬ٝ‫ حُلي‬٠ِ‫ش ػ‬٤ٟ‫ٗلوش حٗوزخ‬
blood culture results, what is the most appropriate empirical ‫خٍس‬٣ُِ‫خ ٌٓ٘ ح‬ٜ‫ ُحىص ٗيط‬٢‫ حُظ‬٠٘ٔ٤ُ‫ح‬
َ‫ حُوِذ ػز‬ٟ‫ي‬ٛ ٢٤‫َ طوط‬ٜ‫ظ‬٣ .‫َس‬٤‫حأله‬
antibiotics regimen for treating this patient?
١َٜ‫ٔخّ حألر‬ُٜ‫ ح‬٠ِ‫يٍ ًظِش ٓظلًَش ػ‬ُٜ‫ح‬
a. Ceftriaxone and gentamycin ّ‫ ٗظخثؾ ٍُع حُي‬.َٜ‫ حألر‬٢‫ي ك‬٣‫ٓغ هِْ ؿي‬
b. Vancomycin and ceftriaxone ٞٛ ‫ ٓخ‬،ّ‫ حٗظظخٍ ٗظخثؾ ٍُع حُي‬٢‫ ك‬.‫ٓؼِوش‬
c. Vancomycin and oxacillin ‫ش حألٗٔذ‬٤‫ز‬٣َ‫ش حُظـ‬٣ٞ٤‫خىحص حُل‬٠ُٔ‫ٗظخّ ح‬
d. Oxacillin and gentamycin ‫؟‬ٞ٣َُٔ‫ٌح ح‬ٛ ‫ُؼالؽ‬
ٖ٤ٔ٣‫ؿ٘ظخٓخ‬ٝ ًٕٞٔ‫خ‬٣َ‫لظ‬٤ٓ .a
ًٕٞٔ‫خ‬٣َ‫لظ‬٤ٓٝ ٖ٤ٔ٣‫ٓخ‬ٌٞٗ‫ كخ‬.b
Une femme de 55 ans présente 3 semaines de fièvre et de malaise. ٖ٤ِ٤ٓ‫ًٔخ‬ٝ‫أ‬ٝ ٖ٤ٔ٣‫ٓخ‬ٌٞٗ‫ كخ‬.c
Elle signale également une aggravation de la dyspnée. Elle est connue ٖ٤ٔ٤ٓ‫ؿ٘ظخ‬ٝ ٖ٤ِ٤ٓ‫ًٔخ‬ٝ‫ أ‬.d
pour avoir une sténose aortique modérée due à une valve aortique
bicuspide. Elle était auparavant asymptomatique. À l‟examen
physique, température 38,1 °C, fréquence cardiaque 90 bpm, pression
artérielle 110/67. L‟examen cardiaque est significatif pour le souffle
systolique sur le bord sternal supérieur droit qui a augmenté en
intensité depuis la dernière visite. L‟échocardiographie
transthoracique montre une masse mobile sur la valve aortique avec
une nouvelle régurgitation aortique. Les résultats des hémocultures
sont en attente. En attendant les résultats de l‟hémoculture, quel est le
régime antibiotique empirique le plus approprié pour traiter ce
patient?
a. Ceftriaxone et gentamycine
b. Vancomycine et ceftriaxone
c. Vancomycine et oxacilline
d. Oxacilline et gentamycine

63
124. A 60-year-old man is evaluated in the emergency department for a ٢‫ ػخٓخ ك‬60 َٔ‫زِؾ ٖٓ حُؼ‬٣ َ‫ْ ٍؿ‬٤٤‫ظْ طو‬٣
two-day history of acute pain and swelling in the left knee. He denied ُْ‫ٖ ٖٓ حأل‬٤ٓٞ٣ ‫ن ُٔيس‬٣ٍ‫حٍة ُظخ‬ٞ‫هْٔ حُط‬
any history of skin rash, trauma, or recent intraarticular procedures. ١‫ أ‬٠‫ٗل‬ٞٛ .َٟٔ٤ُ‫ حًَُزش ح‬٢‫ٍّ ك‬ٞ‫حُظ‬ٝ ‫حُلخى‬
ٝ‫يٓش أ‬ُٜ‫ ح‬ٝ‫ أ‬١‫ن ٖٓ حُطلق حُـِي‬٣ٍ‫طخ‬
He is married and monogamous. He has type II diabetes mellitus and
‫ؽ‬ِٝ‫ ٓظ‬ٞٛ .َٜ‫َس ىحهَ حُٔل‬٤‫اؿَحءحص أه‬
osteoarthritis. Medications are metformin and acetaminophen. On ٢ُٔ٘‫ ٖٓ ح‬١ٌَُٔ‫ ىحء ح‬ٚ٣‫ ُي‬.‫حؽ‬ُِٝ‫ ح‬١‫أكخى‬ٝ
physical examination, temperature is 38.2°C and heart rate is ٖ٤ٍٓٞ‫ظل‬٤ُٔ‫ ح‬٢ٛ ‫ش‬٣ٝ‫ حألى‬.ّ‫٘خٗش ػظخ‬ٛٝ 2
92/minute. Left knee is swollen and warm with overlying erythema ‫ ىٍؿش‬،٢ٗ‫ حُزي‬ٚ‫ ػ٘ي حُلل‬.ٖ٤‫ك‬ٞ٘٤ٓ‫ظخ‬٤ٓ‫حأل‬ٝ
and tender to palpation. Synovial fluid aspiration shows a purulent ‫َرخص‬ٟ ٍ‫ٓؼي‬ٝ ‫ش‬٣ٞ‫ ىٍؿش ٓج‬38.2 ‫حُلَحٍس‬
fluid with WBC count of 85000/mm3 and 95% neutrophils. Synovial ‫ ٓ٘ظلوش‬َٟٔ٤ُ‫ حًَُزش ح‬.‫وش‬٤‫ ىه‬/ 92 ‫حُوِذ‬
fluid culture is pending. The most likely causative organism of this َٜ‫ظ‬٣ .ْ‫الّ ػ٘ي حُـ‬٣‫ا‬ٝ ٢ٓ‫ىحكجش ٓغ كٔخ‬ٝ
‫خ ٓغ ػيى‬٤‫ل‬٤‫ ٓخثال ه‬٢ِ٤ُُِ‫ حُٔخثَ ح‬٢‫ٗل‬
patient‟s disease is:
٪95ٝ 3 ْٓ / 85000 ‫خء‬٠٤‫خص ىّ ر‬٣ًَ
a. Staphylococcus aureus .‫ ٓؼِن‬٢ِ٤ُُِ‫ ٍُع حُٔخثَ ح‬.‫ٓؼظيالص‬
b. Group B streptococcus ‫ٌح‬ٛ ‫لخ ُيحء‬٤‫ش حُٔٔززش حألًؼَ طَؿ‬٣ٞ٠‫حُؼ‬
c. Neisseria gonorrhea :٢ٛ ٞ٣َُٔ‫ح‬
d. Coagulase negative staphylococcus ‫ش‬٤‫ز‬ٌُٛ‫ش ح‬٣‫ى‬ٞ‫ٍحص حُؼ٘و‬ٌُٞٔ‫ ح‬.a
‫ش‬٣‫ػش د حُؼوي‬ٞٔ‫ حُٔـ‬.b
‫ش‬٤٘‫ش حُز‬٣َٔ٤ُ٘‫ ح‬.c
‫ش‬٤‫ش ِٓز‬٣‫ى‬ٞ‫ٍحص ػ٘و‬ٌٞٓ .d
Un homme de 60 ans est évalué au service des urgences pour des
ُ‫ال‬ٞ‫أؿ‬ٌُٞ‫ح‬
antécédents de douleur aiguë et d‟enflure du genou gauche depuis
deux jours. Il a nié tout antécédent d‟éruption cutanée, de traumatisme
ou de procédures intra-articulaires récentes. Il est marié et monogame.
Il souffre de diabète sucré de type II et d‟arthrose. Les médicaments
sont la metformine et l‟acétaminophène. À l‟examen physique, la
température est de 38,2 °C et la fréquence cardiaque est de 92/minute.
Le genou gauche est enflé et chaud avec un érythème sus-jacent et
sensible à la palpation. L‟aspiration du liquide synovial montre un
liquide purulent avec un nombre de globules blancs de 85000/mm3 et
95% de neutrophiles. La culture fluide synoviale est en attente.
L‟organisme causal le plus probable de la maladie de ce patient est:
a. Staphylococcus aureus
b. streptocoque du groupe B
c. Neisseria gonorrhea
d. Staphylocoque coagulase négatif

64
125. An 80-year-old woman is evaluated for a 1-month history of dyspnea ‫ ػخٓخ‬80 َٔ‫ْ حَٓأس طزِؾ ٖٓ حُؼ‬٤٤‫ظْ طو‬٣
on moderate exertion and fatigue. She has a history of hypertension ٌٍ‫ن حُظ٘لْ ػ٘ي ر‬٤ٟ ٖٓ َٜٗ 1 ‫ن‬٣ٍ‫ُظخ‬
treated with hydrochlorothiazide. Upon examination, her BP=125/80 ٖٓ ‫ن‬٣ٍ‫خ طخ‬ٜ٣‫ ُي‬.‫طؼذ‬ٝ ٢ٓٞ‫ى ٓظ‬ٜٞ‫ٓـ‬
‫ؼخُؾ‬٣ ١ٌُ‫ حُيّ ح‬٢‫ـ‬ٟ ‫حٍطلخع‬
with HR of 85. Jugular veins are mildly distended. The lung exam
BP = ،ٚ‫ ػ٘ي حُلل‬.‫ي‬٣ُ‫خ‬٤‫ػ‬ًٍٍِٝٞٝ‫ي‬٤ٜ‫ر‬
shows fine bilateral basilar crackles. Systolic murmur grade II/VI is ‫ش‬٤‫ىحؿ‬ُٞ‫ٍىس ح‬ٝ‫ حأل‬.85 HR ‫ ٓغ‬125/80
noted at the right upper sternal border. Echocardiogram showed ‫ حَُثش‬ٚ‫َ كل‬ٜ‫ظ‬٣ .‫ق‬٤‫ٓ٘ظلوش رٌَ٘ هل‬
Ejection fraction of 30% with aortic stenosis and left ventricular ‫الكع‬٣ .‫وش‬٤‫ش حُـخٗذ ىه‬٤‫ش ػ٘خث‬٣‫ًَحًَ هخػي‬
dilatation. Which of the following medications is shown to decrease ‫ى‬ٝ‫ حُلي‬٢‫ ك‬VI / II ‫ش ٖٓ حَُطزش‬٤ٟ‫ٗلوش اٗوزخ‬
the mortality of this patient? ٟ‫ي‬ٛ ٢‫َ ٓوط‬ٜ‫ أظ‬.٠٘ٔ٤ُ‫خ ح‬٤ِ‫ش حُؼ‬٤ٜ‫حُو‬
a. Enalapril ‫ن‬٤٠‫ ٓغ ط‬٪30 ‫خ ر٘ٔزش‬٣‫َى‬١ ‫حُوِذ ؿِءح‬
‫ش‬٣ٝ‫ ٖٓ حألى‬١‫ أ‬.َٔ٣‫ٖ حأل‬٤‫ٓغ حُزط‬ٞ‫ط‬ٝ َٜ‫حألر‬
b. Verapamil
‫ٌح‬ٛ ‫خص ػ٘ي‬٤‫ك‬ُٞ‫وَِ ٓؼيٍ ح‬٣ ٚٗ‫َ أ‬ٜ‫ظ‬٣ ‫ش‬٤ُ‫حُظخ‬
c. Digoxin ‫؟‬ٞ٣َُٔ‫ح‬
d. Furosemide َ٣َ‫ اٗخالر‬.a
َ٤ٓ‫َحرخ‬٤‫ ك‬.b
ٖ٤ًٔٞ‫ـ‬٣‫ ى‬.c
Une femme de 80 ans est évaluée pour des antécédents de dyspnée de ‫ي‬٤ٔ٤ٍٓٝٞ‫ ك‬.d
1 mois à l‟effort modéré et à la fatigue. Elle a des antécédents
d‟hypertension traitée à l‟hydrochlorothiazide. À l‟examen, son BP =
125/80 avec HR de 85. Les veines jugulaires sont légèrement
distendues. L‟examen pulmonaire montre de fins crépitements
basilaires bilatéraux. Le souffle systolique de grade II/VI est noté au
bord sternal supérieur droit. L‟échocardiogramme a montré une
fraction d‟éjection de 30% avec sténose aortique et dilatation
ventriculaire gauche. Lequel des médicaments suivants est montré
pour diminuer la mortalité de ce patient?
a. Enalapril
b. Vérapamil
c. Digoxine
d. Furosémide

65
126. A 25-year-old man presents with sudden onset of left chest pain. The ‫ش‬٣‫ ٖٓ ريح‬٢ٗ‫ؼخ‬٣ ‫ ػخٓخ‬25 َٔ‫زِؾ ٖٓ حُؼ‬٣ َ‫ٍؿ‬
pain started after vigorous exercise, increases on inspiration, but is not ‫ ريأ حألُْ رؼي‬.َٔ٣‫يٍ حأل‬ُٜ‫ ح‬٢‫ٓلخؿجش ألُْ ك‬
affected by position. He is previously healthy and smokes ‫ ال‬ٌُٚ٘ ،‫ن‬٤ُٜ٘‫ِىحى ػ٘ي ح‬٣ٝ ،١ٞ‫ٖ ه‬٣َٔ‫ط‬
٢‫يس ك‬٤‫لش ؿ‬ٜ‫ظٔظغ ر‬٣ ٕ‫ ًخ‬.‫ش‬٤‫ؼ‬ُٟٞ‫ظؤػَ رخ‬٣
occasionally. On physical examination his temperature is 36.8°C,
ٚ‫ ػ٘ي حُلل‬.َ‫ه‬٥ ٖ٤‫يهٖ ٖٓ ك‬٣ٝ ‫حُٔخرن‬
blood pressure is 120/80 mm Hg, oxygen saturation of 97% on room ‫ ىٍؿش‬36.8 ٚ‫ٕ ىٍؿش كَحٍط‬ٌٞ‫ ط‬،٢ٗ‫حُزي‬
air. 1Chest examination reveals decreased tactile fremitus, ،‫ ِْٓ ُثزن‬80/120 ّ‫ حُي‬٢‫ـ‬ٟٝ ،‫ش‬٣ٞ‫ٓج‬
hyperresonance, and diminished breath sounds in the left ‫حء‬ٞٛ ٠ِ‫ ػ‬٪97 ‫ٖ ر٘ٔزش‬٤‫ط٘زغ حألًٔـ‬ٝ
hemithorax. What is the most likely diagnosis? ٝ‫يٍ ػٖ حٗولخ‬ُٜ‫ ح‬ٚ‫ٌ٘ق كل‬٣.‫حُـَكش‬
a. Spontaneous pneumothorax ٚ‫ط٘خه‬ٝ ٖ٤َُٗ‫ ح‬١َ‫ك‬ٝ ٢ُِٔٔ‫ْ ح‬٤ٔ‫حُل‬
b. Pulmonary embolism .ٍ‫ي‬َُِٜٔ٣‫ق حأل‬ُٜ٘‫ ح‬٢‫حص حُظ٘لْ ك‬ٞٛ‫أ‬
‫لخ؟‬٤‫ حألًؼَ طَؿ‬ٚ٤‫ حُظ٘و‬ٞٛ ‫ٓخ‬
c. Acute pericarditis
١ٞ‫يٍ حُؼل‬ُٜ‫حف ح‬َٝ‫ حٓظ‬.a
d. Costochondritis ١ٞ‫ٔخّ ٍث‬ٜٗ‫ ح‬.b
‫ٍ حُلخى‬ٞٓ‫خد حُظخ‬ٜ‫ حُظ‬.c
٢‫ِؼ‬٠ُ‫ف ح‬َٝ٠‫خد حُـ‬ٜ‫ حُظ‬.d
Un homme de 25 ans présente une douleur soudaine à la poitrine
gauche. La douleur a commencé après un exercice vigoureux,
augmente avec l‟inspiration, mais n‟est pas affectée par la position. Il
est auparavant en bonne santé et fume occasionnellement. À l‟examen
physique, sa température est de 36,8 ° C, sa pression artérielle est de
120/80 mm Hg, sa saturation en oxygène de 97% sur l‟air ambiant.
1L‟examen thoracique révèle une diminution du fremitus tactile, une
hyperrésonance et une diminution des bruits respiratoires dans
l‟hémithorax gauche. Quel est le diagnostic le plus probable?
a. Pneumothorax spontané
b. Embolie pulmonaire
c. Péricardite aiguë
d. Costochondrite

66
127. A 40-year-old woman presents with severe retrosternal dull chest pain ٢ٛٝ ‫َص‬٠‫ ػخٓخ ك‬40 َٔ‫حَٓأس طزِؾ ٖٓ حُؼ‬
that started two hours prior to presentation. She is previously healthy. ٚ‫ حُو‬٢‫ق ك‬ٟ‫ح‬ٝ َ٤‫ي ؿ‬٣‫ ٖٓ أُْ ٗي‬٢ٗ‫طؼخ‬
She also reports right calf pain that started two days ago. Her only ٖٓ ٖ٤‫يٍ ريأ هزَ ٓخػظ‬ُِٜ ٢‫حُوِل‬
.‫ حُٔخرن‬٢‫يس ك‬٤‫لش ؿ‬ٜ‫ ًخٗض ر‬.‫خ‬ٍٛٞ٠‫ك‬
medication is oral contraceptive pills (OCP). On physical
‫ ريأ‬٠٘ٔ٤ُ‫ ٍرِش حُٔخم ح‬٢‫ًٔخ أرِـض ػٖ أُْ ك‬
examination, her blood pressure is 130/80, heart rate of 92 bpm and ‫د ٓ٘غ‬ٞ‫ كز‬ٞٛ ‫ي‬٤‫ك‬ُٞ‫خ ح‬ٛ‫حإ‬ٝ‫ ى‬.ٖ٤ٓٞ٣ َ‫هز‬
oxygen saturation of 92% on room air. Cardiopulmonary exam is ٚ‫ حُلل‬٢‫ ك‬.)OCP( ْ‫ن حُل‬٣َ١ ٖ‫حُلَٔ ػ‬
normal. Right calf tenderness, hotness and swelling were noted. CT ٍ‫ٓؼي‬ٝ ،80/130 ‫خ‬ٜٓ‫ ى‬٢‫ـ‬ٟ ‫زِؾ‬٣ ،٢ٗ‫حُزي‬
angiography of the pulmonary arteries shows pulmonary emboli in the ‫ط٘زغ‬ٝ ‫وش‬٤‫ حُيه‬٢‫ش ك‬٠‫ ٗز‬92 ‫َرخص حُوِذ‬ٟ
right and left pulmonary arteries. Lab tests, including troponin and .‫حء حُـَكش‬ٞٛ ٠ِ‫ ػ‬٪92 ‫ٖ ر٘ٔزش‬٤‫حألًٔـ‬
BNP, are normal. In addition to stopping OCP, which of the following ّ‫ال‬٣‫كع ا‬ُٞ .٢‫ؼ‬٤‫ز‬١ ١ٞ‫ حَُث‬٢‫ حُوِز‬ٚ‫حُلل‬
َٜ‫ظ‬٣ .ٍّٞ‫ط‬ٝ ‫ٗش‬ٞ‫ٓو‬ٝ ٠٘ٔ٤ُ‫ حُٔخم ح‬٢‫ك‬
is the most appropriate management at this time?
‫ٓذ‬ٞ‫ حُٔل‬٢‫ش حُٔوطؼ‬٤‫ػ‬ٝ‫َ حأل‬٣ٜٞ‫ط‬
a. Anticoagulation with warfarin ٖ٤٣‫ حَُ٘ح‬٢‫ ك‬١ٞ‫ٔخّ ٍث‬ٜٗ‫ش ح‬٣ٞ‫ٖ حَُث‬٤٣‫َُِ٘ح‬
b. Anticoagulation with enoxaparin ٙٞ‫ حُلل‬.َٟٔ٤ُ‫ح‬ٝ ٠٘ٔ٤ُ‫ش ح‬٣ٞ‫حَُث‬
c. Intravenous thrombolytic agent (alteplase) ،BNPٝ ٖ٤ٗٞ‫ر‬َٝ‫ ًُي حُظ‬٢‫ رٔخ ك‬،‫ش‬٣َ‫حُٔوز‬
d. Insertion of inferior vena cava filter ‫ ٓٔخ‬١‫ أ‬،OCP ‫وخف‬٣‫ ا‬٠ُ‫خكش ا‬ٟ‫ رخإل‬.‫ش‬٤‫ؼ‬٤‫ز‬١
‫هض؟‬ُٞ‫ٌح ح‬ٛ ٢‫َ حألٗٔذ ك‬٤‫ حُظير‬ٞٛ ٢ِ٣
ٖ٣ٍ‫حٍكخ‬ٝ ‫خى طوؼَ ىّ ٓغ‬٠ٓ .a
ٖ٣ٍ‫ًٔخرخ‬ٞ٘٣‫خى طوؼَ ىّ ٓغ ا‬٠ٓ .b
Une femme de 40 ans présente une douleur thoracique sourde
)ُ‫ (أُظزال‬١‫ي‬٣ٍٝ ‫ ػخَٓ كخٍ ُِوؼَس‬.c
rétrosternale sévère qui a commencé deux heures avant la ‫ف‬ٞ‫ي حألؿ‬٣ٍُٞ‫ اىهخٍ َٓٗق ح‬.d
présentation. Elle est auparavant en bonne santé. Elle signale ٢ِ‫حُٔل‬
également une douleur au mollet droit qui a commencé il y a deux
jours. Son seul médicament est la pilule contraceptive orale (OCP). À
l‟examen physique, sa tension artérielle est de 130/80, sa fréquence
cardiaque de 92 bpm et sa saturation en oxygène de 92 % sur l‟air
ambiant. L‟examen cardiopulmonaire est normal. Une sensibilité au
mollet droit, une chaleur et un gonflement ont été notés.
L‟angiographie CT des artères pulmonaires montre des embolies
pulmonaires dans les artères pulmonaires droite et gauche. Les tests
de laboratoire, y compris la troponine et la BNP, sont normaux. En
plus d‟arrêter OCP, laquelle des situations suivantes est la gestion la
plus appropriée à l‟heure actuelle?
a. Anticoagulation à la warfarine
b. Anticoagulation à l‟énoxaparine
c. Agent thrombolytique intraveineux (altéplase)
d. Insertion d‟un filtre de la veine cave inférieure

67
128. A 28-year-old man is evaluated for a six-month history of episodic ‫ ػخٓخ‬28 َٔ‫زِؾ ٖٓ حُؼ‬٣ َ‫ْ ٍؿ‬٤٤‫ظْ طو‬٣
dyspnea, cough and wheezes. As a child, he had asthma and allergies, ْ‫ن حُظ٘ل‬٤ٟ ٖٓ َٜٗ‫ن ُٔيس ٓظش أ‬٣ٍ‫رظخ‬
but he has been asymptomatic since early teenage years. His recent ،‫لال‬١ ٕ‫ ػ٘يٓخ ًخ‬.ِ٣ُ‫حأل‬ٝ ٍ‫حُٔؼخ‬ٝ ٢‫ر‬ُٞ٘‫ح‬
ُْ ٌُٚ٘ ،‫ش‬٤ٓ‫حُلٔخ‬ٝ ٞ‫ ٖٓ حَُر‬٢ٗ‫ؼخ‬٣ ٕ‫ًخ‬
symptoms started after upper respiratory infections, and since then he
‫وش‬ٛ‫حص حَُٔح‬ٞ٘ٓ ٌ٘ٓ ٝ‫ أػَح‬ٚ٤ِ‫َ ػ‬ٜ‫طظ‬
is having the symptoms 2-3 times a week but not daily and are often ‫خرخص‬ٜ‫َس رؼي حُظ‬٤‫ حأله‬ٟٚ‫ ريأص أػَح‬.‫حُٔزٌَس‬
triggered by exercise or exposure to cold air. He is also awakened ٖ٤‫ٌٓ٘ ًُي حُل‬ٝ ،١ِٞ‫ حُؼ‬٢ٔ‫خُ حُظ٘ل‬ٜ‫حُـ‬
with asthma symptoms 2 to 3 nights a month. Physical examination ‫ع‬ٞ‫ حألٓز‬٢‫ َٓحص ك‬3-2 ٝ‫ ٖٓ حألػَح‬٢ٗ‫ؼخ‬٣
reveals scattered wheezing in both lung fields. Spirometry: FEV1= ٖ‫خ ػ‬ِٛ٤‫ظْ طلل‬٣ ‫ؿخُزخ ٓخ‬ٝ ‫خ‬٤ٓٞ٣ ْ٤ُ ٌُٖٝ
80% of predicted, with a 15 % improvement after inhaled albuterol. ‫حء‬ُِٜٞ َٝ‫ حُظؼ‬ٝ‫ش أ‬ٟ‫خ‬٣َُ‫ن ٓٔخٍٓش ح‬٣َ١
Which of the following is the most appropriate therapy for this ٖٓ ٞ‫ ٍر‬ٝ‫وع ٓغ أػَح‬٤‫ٔظ‬٣ ٚٗ‫ ًٔخ أ‬.‫حُزخٍى‬
ٚ‫ٌ٘ق حُلل‬٣ .َُٜ٘‫ ح‬٢‫خٍ ك‬٤ُ 3 ٠ُ‫ ا‬2
patient?
.‫ حَُثش‬٢ِ‫ ًال كو‬٢‫ِ ٓظ٘خػَ ك‬٣ُ‫ ػٖ أ‬٢ٗ‫حُزي‬
a. Daily Inhaled long-acting beta-agonist (LABA) ،‫هغ‬ٞ‫ ٖٓ حُٔظ‬FEV1 = 80% :ْ‫خّ حُظ٘ل‬٤‫ه‬
b. Daily Inhaled low-dose corticosteroids (ICS) and as- ‫ رؼي حٓظ٘٘خم‬٪15 ‫ٓغ طلٖٔ ر٘ٔزش‬
needed albuterol ‫ حُؼالؽ حألٗٔذ‬ٞٛ ٢ِ٣ ‫ ٓٔخ‬١‫ أ‬.ٍَٝ٤‫ط‬ٞ‫أُز‬
c. As-needed albuterol ‫؟‬ٞ٣َُٔ‫ٌح ح‬ُٜ
d. Daily Inhaled low-dose corticosteroids and long-acting ٍٞ‫ِش حُٔلؼ‬٣ٞ١ ‫ظخ‬٤‫ ٓوِيحص ر‬.a
beta-agonist )LABA( ‫خ‬٤ٓٞ٣ ‫ش‬٤‫حٓظ٘٘خه‬
ٖٓ ‫ش‬٠‫ ؿَػش ٓ٘ول‬.b
Un homme de 28 ans est évalué pour des antécédents de dyspnée ‫ش‬٤‫يحص حالٓظ٘٘خه‬٣َٝ٤‫ٓظ‬ٌٞ٤‫ٍط‬ٌُٞ‫ح‬
‫ٍ ػ٘ي‬َٝ٤‫ط‬ٞ‫حألُز‬ٝ )ICS( ‫خ‬٤ٓٞ٣
épisodique, de toux et de respiration sifflante de six mois. Enfant, il
‫حُلخؿش‬
souffrait d‟asthme et d‟allergies, mais il est asymptomatique depuis le ‫ٍ ػ٘ي حُلخؿش‬َٝ٤‫ط‬ٞ‫ أُز‬.c
début de l‟adolescence. Ses symptômes récents ont commencé après ٖٓ ‫ش‬٠‫ ؿَػش ٓ٘ول‬.d
des infections des voies respiratoires supérieures, et depuis lors, il a ‫ش‬٤‫يحص حالٓظ٘٘خه‬٣َٝ٤‫ٓظ‬ٌٞ٤‫ٍط‬ٌُٞ‫ح‬
les symptômes 2-3 fois par semaine, mais pas tous les jours et sont ٍٞ‫ِش حُٔلؼ‬٣ٞ١ ‫ظخ‬٤‫خ ٓوِيحص ر‬٤ٓٞ٣
souvent déclenchés par l‟exercice ou l‟exposition à l‟air froid. Il est
également réveillé avec des symptômes d‟asthme 2 à 3 nuits par mois.
L‟examen physique révèle une respiration sifflante dispersée dans les
deux champs pulmonaires. Spirométrie : VEMS 1 = 80 % des
prévisions, avec une amélioration de 15 % après inhalation
d‟albutérol. Lequel des traitements suivants est le plus approprié pour
ce patient?
a. Bêta-agoniste à action prolongée inhalé quotidiennement
(BAAP)
b. Corticostéroïdes à faible dose inhalés (CSI) et albutérol au
besoin
c. Albutérol quotidiennement
d. Corticostéroïdes à faible dose inhalé et bêta-agoniste à action
prolongée

68
129. A 70 year old male patient is brought to emergency department with 70 َٔ‫زِؾ ٖٓ حُؼ‬٣ ًًَ ٞ٣َٓ ٍ‫خ‬٠‫ظْ اك‬٣
rectorrhagia which he had for 12 hours. He denied abdominal pain or ْ٤‫حٍة ٓغ ِٗف حُٔٔظو‬ٞ‫ هْٔ حُط‬٠ُ‫ػخٓخ ا‬
prior GI bleeding. No hematemesis. No melena. On physical ّ‫ آال‬٠‫ ٗل‬.‫ ٓخػش‬12 ‫خ ُٔيس‬ٜ٘ٓ ٢ٗ‫ؼخ‬٣ ٕ‫ ًخ‬٢‫حُظ‬
ُ‫خ‬ٜ‫ حُـ‬٢‫ق ٓخرن ك‬٣ِٗ ٝ‫ حُزطٖ أ‬٢‫ك‬
examination, he looks pale, in distress with non-icteric sclera. Rectal
١ٞ‫ؿي طـ‬ٞ٣ ‫ال‬.١ٞٓ‫ء ى‬٢‫ؿي ه‬ٞ٣ ‫ ال‬.٢ٔ٠ُٜ‫ح‬
examination showed bloody stools. Vital signs showed HR of 130 ،‫ ٗخكزخ‬ٝ‫زي‬٣ ،٢ٗ‫ حُزي‬ٚ‫ ػ٘ي حُلل‬.‫ى‬ٞٓ‫أ‬
beats per min, systolic blood pressure of 60 mm Hg. Laboratory tests َٜ‫ أظ‬.‫ش‬٤ٗ‫َهخ‬٣ َ٤‫ِزش ؿ‬ٛ ‫رلخُش ًَد ٓغ‬
showed a hematocrit of 30 percent and mean corpuscular volume of ‫َص‬ٜ‫ أظ‬.١ٞٓ‫ْ رَحُ ى‬٤‫ حُٔٔظو‬ٚ‫كل‬
70 fL, creatinine of 2 mg per dl. The most appropriate initial test after ٢‫ش ك‬٠‫ ٗز‬130 HR ‫ش‬٣ٞ٤‫حُؼالٓخص حُل‬
stabilization is: ِْٓ 60 ٢ٟ‫ حُيّ حالٗوزخ‬٢‫ـ‬ٟٝ ،‫وش‬٤‫حُيه‬
a. Urgent gastroscopy ‫ش‬٣َ‫ حُٔوز‬ٙٞ‫َص حكل‬ٜ‫ أظ‬.‫ُثزن‬
ْ‫ كـ‬٢ٓٞ‫ٓظ‬ٝ % 30 ‫ض ر٘ٔزش‬٣ًَٞ‫ٔخط‬٤ُٜ‫ح‬
b. Urgent surgery
2 ٖ٤٘٤‫خط‬٣ٌَُ‫ح‬ٝ ،fL 70 ‫ٔخص‬٤ٔ‫حُـ‬
c. Urgent unprepared colonoscopy ‫ حألٗٔذ رؼي‬٢ُٝ‫ حأل‬ٚ‫ حُلل‬.َٓ‫ى‬/‫ِٓؾ‬
d. Nuclear technetium scan :ٞٛ ‫ض‬٤‫حُظؼز‬
َ‫َ ٓؼيس ػخؿ‬٤‫ ط٘ظ‬.a
‫ ؿَحكش ػخؿِش‬.b
Un patient de 70 ans est amené au service des urgences avec une ِٜ‫َ ٓـ‬٤‫ٕ ػخؿَ ؿ‬ُٞٞ‫َ ه‬٤‫ ط٘ظ‬.c
rectorragie qu‟il a eue pendant 12 heures. Il a nié toute douleur ‫ٓٔزوخ‬
١ُٝٞ٘‫ّ ح‬ٞ٤‫ظ‬٤ٌ٘‫ ٓٔق حُظ‬.d
abdominale ou saignement gastro-intestinal antérieur. Pas
d‟hématémèse. Pas de méléna. À l‟examen physique, il semble pâle,
en détresse avec une sclérotique non ictérique. L‟examen rectal a
révélé des selles sanglantes. Les signes vitaux ont montré une HR de
130 battements par minute, une pression artérielle systolique de 60
mm Hg. Les tests de laboratoire ont montré un hématocrite de 30% et
un volume corpusculaire moyen de 70 fL, une créatinine de 2 mg par
dl. Le test initial le plus approprié après stabilisation est :
a. Gastroscopie urgente
b. Chirurgie urgente
c. Coloscopie urgente non préparée
d. Scanner au technétium nucléaire

69
130. A 62-year-old woman has chronic renal and hepatic failure secondary َ٘‫خرش رل‬ٜٓ ‫ ػخٓخ‬62 َٔ‫حَٓأس طزِؾ ٖٓ حُؼ‬
to autosomal dominant polycystic kidney disease. She is presently on ٠ٌُِ‫ ح‬َُٝٔ ٢ُ‫ ِٖٓٓ طخ‬١‫ًزي‬ٝ ١ًِٞ
the waiting list for a liver and kidney transplant and has noted .‫ حُٔخثي‬١‫ حُـٔي‬٢‫زـ‬ُٜ‫ٔخص ح‬٤ٌُ‫حُٔظؼيى ح‬
‫ش‬٤ِٔ‫ هخثٔش حالٗظظخٍ إلؿَحء ػ‬٠ِ‫خ ػ‬٤ُ‫ كخ‬٢ٛٝ
increasing abdominal girth. On pelvic examination, the ovaries are
٢٤‫خىس ٓل‬٣ُ ‫الكظض‬ٝ ‫ش‬٤ًِٝ ‫ٍُع ًزي‬
approximately 6 cm in diameter and feel firm and irregular. An intra- َ‫زِؾ هط‬٣ ،ٝٞ‫ حُل‬ٚ‫ ػ٘ي كل‬.ٖ‫حُزط‬
abdominal fluid wave can be palpated. The serum concentration of َ٤‫ؿ‬ٝ ‫الرش‬ٛ ًٝٝ ْٓ 6 ٢ُ‫ح‬ٞ‫ٖ ك‬٤٠٤‫حُٔز‬
CA-125 is elevated. What should be the next step in management of َ‫ؿش حُٔخثَ ىحه‬ٞٓ ْٔ‫ٌٖٔ طل‬٣ .ٖ٤٤‫ؼ‬٤‫ز‬١
this patient? .‫ َٓطلغ‬CA-125 َُٜٔ‫ِ ح‬٤ًَ‫ ط‬.ٖ‫حُزط‬
a. Open laparotomy َ٤‫ طير‬٢‫ش ك‬٤ُ‫س حُظخ‬ٞ‫ٕ حُوط‬ٌٞ‫ـذ إٔ ط‬٣ ‫ٓخًح‬
b. CT scan of the abdomen and pelvis ‫ش؟‬٠٣َُٔ‫ ح‬ٌٙٛ
ٖ‫ كظق حُزط‬.a
c. Repeat CA- 125 Antigen
‫ٓذ‬ٞ‫ حُٔل‬٢‫َ حُٔوطؼ‬٣ٜٞ‫ حُظ‬.b
d. Liver transplant ٝٞ‫حُل‬ٝ ٖ‫ُِزط‬
CA-125 ‫ي‬٠‫ طٌَحٍٓٔظ‬.c
Une femme de 62 ans souffre d‟insuffisance rénale et hépatique ‫ ٍُع حٌُزي‬.d
chronique secondaire à une polykystose rénale autosomique
dominante. Elle est actuellement sur la liste d‟attente pour une greffe
du foie et des reins et a noté une augmentation de la circonférence
abdominale. À l‟examen pelvien, les ovaires mesurent environ 6 cm
de diamètre et sont fermes et irréguliers. Une onde de liquide intra-
abdominale peut être palpée. La concentration sérique de CA-125 est
élevée. Quelle devrait être la prochaine étape de la prise en charge de
ce patient?
a. ouverte laparotomie
b. TDM de l‟abdomen et du bassin
c. Répéter de l‟antigène CA-125
d. la greffe du foie

70
131. A 25-year-old woman, gravida 2 para 1, at 8 weeks gestation comes ‫الىس‬ٝ 2 َٔ‫ ك‬،‫ ػخٓخ‬25 َٔ‫حَٓأس طزِؾ ٖٓ حُؼ‬
to the physician to initiate prenatal care. Her blood type is 0 negative ‫ذ‬٤‫ حُطز‬٠ُ‫ ا‬٢‫غ ٖٓ حُلَٔ طؤط‬٤‫ أٓخر‬8 ٢‫ ك‬،1
and the father's blood type is 0 positive. Her first pregnancy was 0 ‫خ‬ٜٓ‫ِش ى‬٤ٜ‫ ك‬.‫الىس‬ُٞ‫ش ٓخ هزَ ح‬٣‫ُزيء حَُػخ‬
َٜ‫ أظ‬.‫ش‬٤‫ـخر‬٣‫ ا‬0 ‫ِش ىّ حألد‬٤ٜ‫ك‬ٝ ‫ٓخُزش‬
significant for placental abruption at the time of delivery. She
‫هض‬ٝ ٢‫ٔش ك‬٤ُ٘ٔ‫خٍ ح‬ٜ‫ٍ حٗل‬ٝ‫خ حأل‬ِٜٔ‫ك‬
received the standard dose of anti-D immune globulin at 28 weeks ٖٓ ‫ش‬٤ٓ‫خ‬٤‫ طِوض حُـَػش حُو‬.‫الىس‬ُٞ‫ح‬
during her first pregnancy and again 1 day postpartum. The patient ٢‫ ك‬D ٍ ‫خى‬٠ُٔ‫ ح‬٢‫ٖ حُٔ٘خػ‬٤ُٞ٤‫ر‬ِٞ‫حُـ‬
has no medical problems and no history of blood transfusions. Her َٟ‫َٓس أه‬ٝ ٍٝ‫خ حأل‬ِٜٔ‫ هالٍ ك‬28 ‫ع‬ٞ‫حألٓز‬
anti-D antibody titer is currently 1:32. Which of the following is the ٖٓ ‫ش‬٠٣َُٔ‫ ح‬٢ٗ‫ ال طؼخ‬.‫الىس‬ُٞ‫ّ ٖٓ ح‬ٞ٣ 1 ‫رؼي‬
most likely explanation for this patient's finding? .ّ‫خص ٗوَ حُي‬٤ِٔ‫ن ٖٓ ػ‬٣ٍ‫ال طخ‬ٝ ‫ش‬٤‫ز‬١ ًَ‫ٓ٘خ‬
a. Inadequate dose of anti-D immune globulin after her first ‫خ‬٤ُ‫ كخ‬D ٍ ‫خىس‬٠ُٔ‫خٍ حألؿٔخّ ح‬٤‫زِؾ ػ‬٣
َ‫َ حألًؼ‬٤ٔ‫ حُظل‬ٞٛ ٢ُ‫ ٖٓ حُظخ‬١‫ أ‬.1:32
delivery
‫ش؟‬٠٣َُٔ‫ ح‬ٌٙٛ ‫ـش‬٤‫لخ ُ٘ظ‬٤‫طَؿ‬
b. Inadequate dose of anti-D immune globulin during her first ٖ٤ُٞ٤‫ر‬ِٞ‫ش ٖٓ حُـ‬٤‫َ ًخك‬٤‫ ؿَػش ؿ‬.a
pregnancy ‫خ‬ٜ‫الىط‬ٝ ‫ رؼي‬D ٍ ‫خى‬٠ُٔ‫ ح‬٢‫حُٔ٘خػ‬
c. No anti-D immune globulin prophylaxis between the ٠ُٝ‫حأل‬
pregnancies ٖ٤ُٞ٤‫ر‬ِٞ‫ش ٖٓ حُـ‬٤‫َ ًخك‬٤‫ ؿَػش ؿ‬.b
d. Premature administration of anti-D immune globulin after her ‫خ‬ِٜٔ‫ أػ٘خء ك‬D ٍ ‫خى‬٠ُٔ‫ ح‬٢‫حُٔ٘خػ‬
first pregnancy ٍٝ‫حأل‬
‫خى‬٠ٓ ٢‫ٖ ٓ٘خػ‬٤ُٞ٤‫ر‬ِٞ‫ؿي ؿ‬ٞ٣ ‫ ال‬.c
َٔ‫ٖ كخالص حُل‬٤‫ ر‬D ٍ
Une femme de 25 ans, gravida 2 para 1, à 8 semaines de gestation
٢‫ٖ حُٔ٘خػ‬٤ُٞ٤‫ر‬ِٞ‫ اػطخء حُـ‬.d
vient chez le médecin pour commencer les soins prénatals. Son
‫حٕ رؼي‬ٝ‫ هزَ حأل‬D ٍ ‫خى‬٠ُٔ‫ح‬
groupe sanguin est négatif de 0 et le groupe sanguin de son père est de
ٍٝ‫خ حأل‬ِٜٔ‫ك‬
0 positif. Sa première grossesse était importante pour le décollement
placentaire au moment de l‟accouchement. Elle a reçu la dose
standard d‟immunoglobuline anti-D à 28 semaines au cours de sa
première grossesse et de nouveau 1 jour après l‟accouchement. Le
patient n‟a aucun problème médical et aucun antécédent de
transfusions sanguines. Son titre d‟anticorps anti-D est actuellement
de 1:32. Laquelle des explications suivantes est la plus probable de la
découverte de ce patient?
a. Dose insuffisante d‟immunoglobuline anti-D après son
premier accouchement
b. Dose insuffisante d‟immunoglobuline anti-D pendant sa
première grossesse
c. Pas de prophylaxie des immunoglobulines anti-D entre les
grossesses
d. Administration prématurée d‟immunoglobulines anti-D après
sa première grossesse

71
132. A 45-year-old woman presents to her physician's office complaining ٠ُ‫َا‬٠‫ ػخٓخ طل‬45 َٔ‫حَٓأس طزِؾ ٖٓ حُؼ‬
of night sweats and insomnia. She states that for the past month she ٢ِ٤ُِ‫ ٖٓ حُظؼَم ح‬ٌٞ٘‫ ط‬٢ٛٝ ‫خ‬ٜ‫ز‬٤‫ز‬١ ‫خىس‬٤‫ػ‬
has woken up completely soaked with perspiration on several َُٜ٘‫وظض هالٍ ح‬٤‫خ حٓظ‬ٜٗ‫ٍ ا‬ٞ‫طو‬ٝ .‫حألٍم‬ٝ
‫ ػيس‬٢‫ حُؼَم ك‬٢‫ ؿخٍهش طٔخٓخ ك‬٢ٟ‫حُٔخ‬
occasions. She has had irregular menstrual periods for the past six
َ٤‫ ؿ‬ٞ٤‫ ٖٓ كظَحص ك‬٢ٗ‫ ًخٗض طؼخ‬.‫ٓ٘خٓزخص‬
months. She consumes one ounce of alcohol nightly before going to .‫ش‬٤ٟ‫َ حُٔظش حُٔخ‬ٜٗ‫ٓ٘ظظٔش هالٍ حأل‬
the bed, and quit smoking 5 years ago. She has a history of َ‫ال هز‬٤ُ ٍٞ‫حكيس ٖٓ حٌُل‬ٝ ‫ش‬ٜٗٝ‫ِي أ‬ٜ‫طٔظ‬
hypertension controlled with hydrochlorothiazide. She denies illicit ٌ٘ٓ ٖ٤‫أهِؼض ػٖ حُظيه‬ٝ ،ٕ‫ حُلَح‬٠ُ‫خد ا‬ٌُٛ‫ح‬
drug use. Her temperature is 36.C (98 F), blood pressure is 140/90 ّ‫ حُي‬٢‫ـ‬ٟ ‫ن ٖٓ حٍطلخع‬٣ٍ‫خ طخ‬ٜ٣‫ ُي‬.‫حص‬ٞ٘ٓ 5
mmHg, pulse is 80/min, and respirations are 14/min. Physical ٢ٛٝ .‫ي‬٣ُ‫خ‬٤‫ػ‬ًٍٍِٝٞٝ‫ي‬٤ٜ‫ ر‬ٚ٤ِ‫طَ ػ‬٤ٔ‫ ط‬١ٌُ‫ح‬
examination shows no abnormalities. Which of the following is the .‫ػش‬َُٝ٘ٔ‫َ ح‬٤‫ حُٔويٍحص ؿ‬٢١‫ طؼخ‬٢‫ط٘ل‬
98( ‫ش‬٣ٞ‫ ىٍؿش ٓج‬36 ‫خ‬ٜ‫ىٍؿش كَحٍط‬
best next step in management?
،‫ ْٓ ُثزن‬90/140 ّ‫ حُي‬٢‫ـ‬ٟٝ ،)‫ض‬٣‫خ‬َٜٜٗ‫ك‬
a. Measure serum TSH and FSH .‫وش‬٤‫ ىه‬/ 14 ْ‫حُظ٘ل‬ٝ ،‫وش‬٤‫ ىه‬/ 80 ٞ‫حُ٘ز‬ٝ
b. Obtain a urine toxicology screen ‫ ٓٔخ‬١‫ أ‬.‫خص‬ٛٞ٘‫ ط‬١‫َ أ‬ٜ‫ظ‬٣ ‫ ال‬٢ٗ‫ حُزي‬ٚ‫حُلل‬
c. Reassure her that she is reaching menopause َ٤‫ حُظير‬٢‫ش ك‬٤ُ‫س طخ‬ٞ‫َ هط‬٠‫ أك‬ٞٛ ٢ِ٣
d. Prescribe a short course of oral hormone replacement therapy ‫؟‬٢‫حُؼالؿ‬
٢ُِٜٔ‫ ح‬FSHٝ TSH ّ‫خ‬٤‫ ه‬.a
Une femme de 45 ans se présente au cabinet de son médecin en se ٍٞ‫ش حُز‬٤ٔٓ ٕ‫خ‬٤‫ ر‬٠ِ‫ٍ ػ‬ٜٞ‫ حُل‬.b
ٖٓ ٠ُ‫ِض ا‬ٛٝ ‫خ‬ٜٗ‫خ رؤ‬ٜ‫ٔؤٗظ‬١ .c
plaignant de sueurs nocturnes et d‟insomnie. Elle affirme que depuis
ّ‫ؤ‬٤ُ‫ح‬
un mois, elle s‟est réveillée complètement trempée de transpiration à ‫َس ٖٓ حُؼالؽ‬٤ٜ‫ٍس ه‬ٝ‫ق ى‬ٛٝ .d
plusieurs reprises. Elle a eu des règles irrégulières au cours des six ْ‫ن حُل‬٣َ١ ٖ‫ِش ػ‬٣‫ٗخص حُزي‬َُٜٞٓ‫رخ‬
derniers mois. Elle consomme une once d‟alcool tous les soirs avant
d‟aller au lit et a arrêté de fumer il y a 5 ans. Elle a des antécédents
d‟hypertension contrôlée par l‟hydrochlorothiazide. Elle nie avoir
consommé des drogues illicites. Sa température est de 36.C (98 F), sa
pression artérielle est de 140/90 mmHg, son pouls est de 80/min et sa
respiration est de 14/min. L‟examen physique ne montre aucune
anomalie. Laquelle des étapes suivantes est la meilleure prochaine
étape dans la gestion?
a. Mesurer la TSH et la FSH sériques
b. Obtenir un dépistage de toxicologie urinaire
c. Rassurez-la qu‟elle atteint de la ménopause
d. Prescrire un traitement hormonal substitutif oral de courte
durée

72
133. A 38-year-old nulliparous woman complains of lower abdominal ّ‫ ٖٓ ػي‬ٌٞ٘‫ ػخٓخ ط‬38 َٔ‫حَٓأس طزِؾ ٖٓ حُؼ‬
discomfort and chronic constipation. She has cramping with her .ُِٖٓٔ‫حإلٓٔخى ح‬ٝ ٖ‫ أٓلَ حُزط‬٢‫حَُحكش ك‬
menses that causes her to miss a day of work each month. Menses ‫ص‬ٞ‫خ طل‬ِٜ‫ـؼ‬٣ ١ٌُ‫ ح‬ٞ٤‫خ ط٘٘ؾ ٓغ حُل‬ٜ٣‫ُي‬
ٞ٤‫ليع حُل‬٣ .َٜٗ ًَ َٔ‫ّ ٖٓ حُؼ‬ٞ٣
occur regularly at 28-day intervals with no bleeding or spotting
ٖ٤‫غ ر‬٤‫ طزو‬ٝ‫ق أ‬٣ِٗ ٕٝ‫ٓخ ى‬ٞ٣ 28 ًَ ّ‫رخٗظظخ‬
between cycles. Past medical history is significant for asthma and an ٠ُ‫َ ا‬٤٘٣ ‫ حُٔخرن‬٢‫ن حُطز‬٣ٍ‫ حُظخ‬.‫ٍحص‬ٝ‫حُي‬
appendectomy at age 12. Her mother had breast cancer at age 55, and .12 ٖٓ ٢‫ش ك‬٣‫ى‬ٝ‫خٍ حُِحثيس حُي‬ٜ‫حٓظج‬ٝ ٞ‫حَُر‬
her maternal grandmother died of ovarian cancer at age 66. Vital ،55 ٖٓ ٢‫ ك‬١‫خٕ حُؼي‬١َٔ‫خ ر‬ٜ‫حُيط‬ٝ ‫زض‬٤ٛ‫أ‬
signs are within normal limits. Pelvic examination shows an enlarged ٞ٤‫خٕ حُٔز‬١َٓ ‫خ رٔزذ‬ٜٓ‫خ أل‬ٜ‫ض ؿيط‬٤‫ك‬ٞ‫ط‬ٝ
uterus that is irregular and mobile with a prominent posterior mass. ‫ش‬٣ٞ٤‫ حُؼالٓخص حُل‬.‫ ػخٓخ‬66 ِٛ‫٘خ‬٣ َٔ‫ػٖ ػ‬
Which of the following is the best next step in management of this ٚ‫َ كل‬ٜ‫ظ‬٣ .‫ش‬٤‫ؼ‬٤‫ى حُطز‬ٝ‫ٖٔ حُلي‬ٟ
‫ٓظلَى‬ٝ ْ‫َ ٓ٘ظظ‬٤‫وْ ؿ‬٠‫ ٍكْ ٓظ‬ٝٞ‫حُل‬
patient?
َ٠‫ أك‬ٞٛ ٢ِ٣ ‫ ٓٔخ‬١‫ أ‬.‫ش رخٍُس‬٤‫ٓغ ًظِش هِل‬
a. Pelvic ultrasound ‫ش؟‬٠٣َُٔ‫ ح‬ٌَٙٛ٤‫ طير‬٢‫ش ك‬٤ُ‫س طخ‬ٞ‫هط‬
b. CT scan of the abdomen and pelvis ‫ش‬٤‫ط‬ُٜٞ‫م ح‬ٞ‫ؿخص ك‬ُٞٔ‫َ رخ‬٣ٜٞ‫ حُظ‬.a
c. Endometrial biopsy ٝٞ‫ُِل‬
d. BRCA 1 and BRCA2 genetic testing ‫ٓذ‬ٞ‫ حُٔل‬٢‫َ حُٔوطؼ‬٣ٜٞ‫ حُظ‬.b
ٝٞ‫حُل‬ٝ ٖ‫ُِزط‬
ْ‫ هِػش رطخٗش حَُك‬.c
BRCA 1 ٢٘٤‫ حالهظزخٍ حُـ‬.d
Une femme nullipare de 38 ans se plaint d‟une gêne abdominale basse
BRCA2ٝ
et d‟une constipation chronique. Elle a des crampes avec ses règles
qui lui font manquer une journée de travail chaque mois. Les
menstruations se produisent régulièrement à des intervalles de 28
jours sans saignement ni spotting entre les cycles. Les antécédents
médicaux sont significatifs pour l‟asthme et une appendicectomie à
l‟âge de 12 ans. Sa mère a eu un cancer du sein à l‟âge de 55 ans et sa
grand-mère maternelle est décédée d‟un cancer de l‟ovaire à l‟âge de
66 ans. Les signes vitaux sont dans les limites normales. L‟examen
pelvien montre un utérus élargi qui est irrégulier et mobile avec une
masse postérieure proéminente. Laquelle des étapes suivantes est la
meilleure prochaine étape dans la prise en charge de ce patient?
a. Échographie pelvienne
b. TDM de l‟abdomen et du bassin
c. Biopsie de l‟endomètre
d. Test génétique BRCA 1 et BRCA2

73
134. A 16-year-old female presents to the ER complaining of left lower ٠ُ‫َ ا‬٠‫ ػخٓخ طل‬16 َٔ‫حَٓأس طزِؾ ٖٓ حُؼ‬
quadrant abdominal pain that started suddenly 24 hours ago. The pain ٢‫ ٖٓ آالّ ك‬ٌٞ٘‫ ط‬٢ٛٝ ‫حٍة‬ٞ‫ؿَكش حُط‬
does not radiate and is 5/10 in severity. She denies having fevers, ‫َٔ ريأص كـؤس‬٣‫ حأل‬٢ِ‫ حَُرغ حُٔل‬٢‫حُزطٖ ك‬
٢‫ ك‬10/5 ٞٛٝ ‫ِظق‬٣ ‫ حألُْ ال‬.‫ ٓخػش‬24 َ‫هز‬
vomiting, dysuria, diarrhea or vaginal bleeding. Her last menstrual
ٝ‫ء أ‬٢‫ حُو‬ٝ‫ أ‬٠ٔ‫خ رخُل‬ٜ‫خرظ‬ٛ‫ ا‬٢‫ ط٘ل‬.ٚ‫ٗيط‬
period was two weeks ago. She takes no medications. On physical .٢ِ‫ز‬ُٜٔ‫ق ح‬٣ُِ٘‫ ح‬ٝ‫خٍ أ‬ٜٓ‫ حإل‬ٝ‫ٍ أ‬ٞ‫ػَٔ حُز‬
examination, her temperature is 37.2° C, blood pressure is 110/65 ‫ ال‬.ٖ٤‫ػ‬ٞ‫خ هزَ أٓز‬ُٜ ‫ش‬٣َٜٗ ‫ٍس‬ٝ‫ًخٗض آهَ ى‬
mmHg, pulse is 80/min and respirations are 14/min. There is mild left ٌٕٞ‫ ط‬،٢ٗ‫ حُزي‬ٚ‫ حُلل‬٢‫ ك‬.‫ش‬٣ٝ‫ أى‬١‫ٍ أ‬ٝ‫طظ٘خ‬
lower quadrant tenderness without rebound or rigidity, and the ٢‫ـ‬ٟٝ ،‫ش‬٣ٞ‫ ىٍؿش ٓج‬37.2 ‫خ‬ٜ‫ىٍؿش كَحٍط‬
remainder of the examination is unremarkable. Which of the / 80 ٞ‫حُ٘ز‬ٝ ،‫ ْٓ ُثزن‬65/110 ّ‫حُي‬
following is the most likely diagnosis? ّ‫ال‬٣‫٘خى ا‬ٛ .‫وش‬٤‫ ىه‬/ 14 ْ‫حُظ٘ل‬ٝ ،‫وش‬٤‫ىه‬
‫ٕ حٍطيحى‬ٝ‫َٔ ى‬٣‫ حأل‬٢ِ‫ حَُرغ حُٔل‬٢‫ق ك‬٤‫هل‬
a. Diverticulitis
١‫ أ‬.‫ظ‬ٞ‫َ ِٓل‬٤‫ ؿ‬ٚ‫ش حُلل‬٤‫رو‬ٝ ،‫الرش‬ٛ ٝ‫أ‬
b. Midcycle pain ‫لخ؟‬٤‫ حألًؼَ طَؿ‬ٚ٤‫ حُظ٘و‬ٞٛ ٢ُ‫ٖٓ حُظخ‬
c. Pelvic inflammatory disease ‫خد حَُطؾ‬ٜ‫ حُظ‬.a
d. Acute appendicitis ‫ٍس‬ٝ‫ق حُي‬ٜ‫ أُْ ٓ٘ظ‬.b
ٝٞ‫خد حُل‬ٜ‫ حُظ‬َٝٓ .c
‫ش حُلخى‬٣‫ى‬ٝ‫خد حُِحثيس حُي‬ٜ‫ حُظ‬.d
Une jeune fille de 16 ans se présente à l‟urgence en se plaignant de
douleurs abdominales dans le quadrant inférieur gauche qui ont
commencé soudainement il y a 24 heures. La douleur ne rayonne pas
et est de gravité 5/10. Elle nie avoir de la fièvre, des vomissements, de
la dysurie, de la diarrhée ou des saignements vaginaux. Ses dernières
règles ont eu lieu il y a deux semaines. Elle ne prend aucun
médicament. À l‟examen physique, sa température est de 37,2 °C, sa
pression artérielle est de 110/65 mmHg, son pouls est de 80/min et sa
respiration est de 14/min. Il y a une légère sensibilité du quadrant
inférieur gauche sans rebond ni rigidité, et le reste de l‟examen n‟est
pas remarquable. Lequel des énoncés suivants est le diagnostic le plus
probable?
a. Diverticulite
b. Douleur en milieu de cycle
c. Maladie inflammatoire pelvienne
d. Appendicite aiguë

74
135. A 3-year-old girl is at a health supervision visit. The girl was ‫خٍس‬٣ُ ٢‫حص ك‬ٞ٘ٓ 3 َٔ‫كظخس طزِؾ ٖٓ حُؼ‬
diagnosed with bilateral grade IV vesicoureteral reflux after a urinary ‫خرش حُلظخس‬ٛ‫ ا‬ٚ٤‫ طْ ط٘و‬.٢‫ل‬ُٜ‫ُإلَٗحف ح‬
tract infection 6 months ago. The girl has been otherwise healthy, ‫ ٖٓ حُيٍؿش حَُحرؼش‬٢‫ حُلخُز‬٢ٗ‫رـٍِ حُٔؼخ‬
.َٜٗ‫ أ‬6 َ‫ش هز‬٤ُٞ‫ حُٔٔخُي حُز‬ٟٝ‫رؼي ػي‬
with normal growth parameters and development. She has been toilet
ُٞٔ٘‫َ ح‬٤٣‫ ٓغ ٓؼخ‬،‫يس‬٤‫لش ؿ‬ٜ‫ًخٗض حُلظخس ر‬
trained since age 2 years. Her physical examination findings are ٠ِ‫خ ػ‬ٜ‫ز‬٣ٍ‫ ُوي طْ طي‬.ٍٞ‫حُظط‬ٝ ٢‫ؼ‬٤‫حُطز‬
normal. Urinalysis shows: specific gravity: 1.020pH: 6.0and no ‫ ٗظخثؾ‬.‫حص‬ٞ٘ٓ 2 ٖٓ ٌ٘ٓ ٝ‫حٓظويحّ حَُٔكخ‬
protein, blood, leukocyte esterase, or nitrites. Of the following, it is َ٤ِ‫َ طل‬ٜ‫ظ‬٣ .‫ش‬٤‫ؼ‬٤‫ز‬١ ‫خ‬ُٜ ٢ٗ‫ حُزي‬ٚ‫حُلل‬
MOST likely that in this disorder ‫ال‬ٝpH: 6.01.020 :٢‫ػ‬ُٞ٘‫ حُؼوَ ح‬:ٍٞ‫حُز‬
a. antibiotic prophylaxis can be stopped after 1 year without ّ‫خص حُي‬٣ًَ ُ‫ آظَح‬ٝ‫ ىّ أ‬ٝ‫ٖ أ‬٤‫ط‬َٝ‫ؿي ر‬ٞ٣
urinary tract infection ‫لخ‬٤‫ حألًؼَ طَؿ‬٢ِ٣ ‫ ٓٔخ‬.‫ض‬٣َ‫ حُ٘ظ‬ٝ‫خء أ‬٠٤‫حُز‬
:‫طَحد‬ٟ‫ٌح حال‬ٛ ٢‫ إٔ ك‬ٞٛ
b. the primary form is uncommon in children
‫خىحص‬٠ُٔ‫ش رخ‬٣‫هخ‬ُٞ‫وخف ح‬٣‫ٌٖٔ ا‬٣ .a
c. bladder and bowel dysfunction is uncommon ٟٝ‫ٕ ػي‬ٝ‫ ٓ٘ش ى‬1 ‫ش رؼي‬٣ٞ٤‫حُل‬
d. spontaneous resolution is unlikely ‫ش‬٤ُٞ‫حُٔٔخُي حُز‬
‫َ ٗخثغ ػ٘ي‬٤‫ ؿ‬ٞٛ ٢ٓ‫ حٌَُ٘ حألٓخ‬.b
ٍ‫لخ‬١‫حأل‬
Une fillette de 3 ans est en visite de surveillance médicale. La fillette ‫َ ٗخثغ‬٤‫حألٓؼخء ؿ‬ٝ ‫ؼق حُٔؼخٗش‬ٟ .c
a reçu un diagnostic de reflux vésico-urétéral bilatéral de grade IV ‫َ َٓؿق‬٤‫ ؿ‬٢‫ حُوَحٍ حُظِوخث‬.d
après une infection des voies urinaires il y a 6 mois. La fille a été par
ailleurs en bonne santé, avec des paramètres de croissance et de
développement normaux. Elle a été formée à la propreté depuis l‟âge
de 2 ans. Les résultats de son examen physique sont normaux.
L‟analyse d‟urine montre: densité : 1.020pH: 6.0et pas de protéines,
de sang, d‟estérase leucocytaire ou de nitrites. Parmi les éléments
suivants, il est TRÈS probable que dans ce trouble,
a. la prophylaxie antibiotique peut être arrêtée après 1 an sans
infection des voies urinaires
b. la forme primaire est rare chez les enfants
c. la dysfonction de la vessie et de l‟intestin est rare
d. la résolution spontanée est peu probable

75
136. The timing, sequence, and rationale behind the introduction of solid ٢‫حألٓخّ حُٔ٘طو‬ٝ َِٔٔ‫حُظ‬ٝ ‫ض‬٤‫ه‬ٞ‫ؼي حُظ‬٣
foods is an important topic at the routine 4-month health maintenance ‫ٔخ‬ٜٓ ‫ػخ‬ٟٞٞٓ ‫ِزش‬ُٜ‫ؼٔش ح‬١‫ٍحء اىهخٍ حأل‬ٝ
visit. An evidence-based discussion on infant nutrition should focus 4 ‫ش ُٔيس‬٤٘٤‫ط‬َُٝ‫ش ح‬٤‫ل‬ُٜ‫خٗش ح‬٤ُٜ‫خٍس ح‬٣ُ ٢‫ك‬
٠ِ‫ـذ إٔ طًَِ حُٔ٘خه٘ش حُوخثٔش ػ‬٣ .َٜٗ‫أ‬
on the recommendations for infants at 4 months of age. Of the
‫خص‬٤ٛٞ‫ حُظ‬٠ِ‫غ ػ‬َُٟ‫ش ح‬٣ٌ‫ٍ طـ‬ٞ‫حألىُش ك‬
following, the statement that MOST accurately reflects the latest ٖ٤‫ ٖٓ ر‬.َٜٗ‫ أ‬4 َٔ‫ ػ‬٢‫غ ك‬َُٟ‫ش رخ‬ٛ‫حُوخ‬
recommendations is: ‫ؼٌْ ريهش أكيع‬٣ ١ٌُ‫خٕ ح‬٤‫ كبٕ حُز‬،٢ِ٣ ‫ٓخ‬
a. adding a small amount of cereal in the bedtime bottle is not :ٞٛ ‫خص‬٤ٛٞ‫حُظ‬
harmful and has been demonstrated to aid sleeping through the ‫د‬ٞ‫َس ٖٓ حُلز‬٤‫ـ‬ٛ ‫ش‬٤ًٔ ‫خكش‬ٟ‫ ا‬.a
night, especially in large infants ‫خٍس‬ٟ ‫ٔض‬٤ُ ُّٞ٘‫هض ح‬ٝ ‫ ُؿخؿش‬٢‫ك‬
b. between 6 and 11 months of age, 8 ounces of 100% fruit juice ُّٞ٘‫ ح‬٠ِ‫خ طٔخػي ػ‬ٜٗ‫هي ػزض أ‬ٝ
‫غ‬َُٟ‫ش ػ٘ي ح‬ٛ‫ هخ‬،َ٤ُِ‫حٍ ح‬ٞ١
is recommended, in addition to complementary foods, to meet
ٍ‫حٌُزخ‬
the daily requirements for vitamin C intake 11ٝ 6 ٖ٤‫ْ ر‬ٍٛ‫ف أػٔخ‬ٝ‫ٖ طظَح‬٣ٌُ‫ ح‬.b
c. delaying the introduction of solid foods beyond 4 to 6 months 8 ّ‫ رخٓظويح‬٠ٛٞ٣ ،‫َح‬ٜٗ
of age has not been shown to prevent the development of food ‫ش ر٘ٔزش‬ًٜ‫َ حُلخ‬٤ٜ‫خص ٖٓ ػ‬ٜٗٝ‫أ‬
allergies ‫ؼٔش‬١‫ حأل‬٠ُ‫خكش ا‬ٟ‫ رخإل‬،٪100
d. Start all types of solid food as soon as early as possible ‫ش‬٤ٓٞ٤ُ‫خؿخص ح‬٤‫ش حالكظ‬٤‫ ُظِز‬،‫ش‬٤ِ٤ٌٔ‫حُظ‬
C ٖ٤ٓ‫ظخ‬٤‫ٍ ك‬ٝ‫ٖٓ ط٘خ‬
‫ِزش رؼي‬ُٜ‫ؼٔش ح‬١‫َ اىهخٍ حأل‬٤‫ طؤه‬.c
‫ؼزض‬٣ ُْ َٔ‫َ ٖٓ حُؼ‬ٜٗ‫ أ‬6 ٠ُ‫ ا‬4
Le moment, la séquence et la raison d‟être de l‟introduction des ‫ش‬٤‫ش حُـٌحث‬٤ٓ‫ٍ حُلٔخ‬ٞ‫ٔ٘غ طط‬٣ ٚٗ‫أ‬
aliments solides sont un sujet important lors de la visite de routine de ‫ِزش‬ُٜ‫ؼٔش ح‬١‫حع حأل‬ٞٗ‫غ أ‬٤ٔ‫ حُزيء رـ‬.d
maintien de la santé de 4 mois. Une discussion fondée sur des ٌٖٔٓ ‫هض‬ٝ ‫ أهَد‬٢‫ك‬
données probantes sur la nutrition des nourrissons devrait se
concentrer sur les recommandations pour les nourrissons de 4 mois.
Parmi les éléments suivants, l‟énoncé qui reflète le plus fidèlement les
dernières recommandations est:
a. l‟ajout d‟une petite quantité de céréales dans le biberon au
coucher n‟est pas nocif et il a été démontré qu‟il aide à dormir
toute la nuit, en particulier chez les grands nourrissons
b. âgés de 6 à 11 mois, 8 onces de jus de fruits à 100% sont
recommandées, en plus des aliments complémentaires, pour
répondre aux besoins quotidiens en vitamine C
c. retarder l‟introduction d‟aliments solides au-delà de l‟âge de 4
à 6 mois n‟a pas été démontré pour prévenir le développement
d‟allergies alimentaires
d. Commencez tous les types d‟aliments solides dès que possible

76
137. An otherwise healthy neonate in the newborn nursery has a well- ‫خٗش‬٠‫ ك‬٢‫ى ك‬ٞ‫ؿ‬ٞٓ ْ٤ِٓ ‫الىس‬ُٞ‫غ ح‬٣‫لَ كي‬١
developed rugated scrotum without palpable testes. The phallus is 2 ٍٞ‫ؽ ٓظط‬ٞٔٓ ٖ‫ل‬ٛ ْ٤ً ٚ٣‫الىس ُي‬ُٞ‫ ح‬٢‫ؼ‬٣‫كي‬
cm in length with a urethral meatus at the base of the penile shaft just ٍٞ١ ‫زِؾ‬٣ .ٖ٤‫ٓظ‬ِٞٔٓ ٖ٤‫ظ‬٤ٜ‫ٕ ه‬ٝ‫ري‬
‫ ػ٘ي هخػيس‬٢ِ٤ِ‫ٔخم اك‬ٛ ‫ ْٓ ٓغ‬2 ‫ذ‬٤٠‫حُو‬
above the scrotum. Of the following, the disorder most likely
.‫لٖ ٓزخَٗس‬ُٜ‫ْ ح‬٤ً ‫م‬ٞ‫ذ ك‬٤٠‫ى حُو‬ٞٔ‫ػ‬
associated with this condition is َ‫طَحد حألًؼ‬ٟ‫ كبٕ حال‬،٢ِ٣ ‫ٖ ٓخ‬٤‫ٖٓ ر‬
a. congenital adrenal hyperplasia ٞٛ ‫ حُلخُش‬ٌٜٙ‫ ر‬٢‫لخ حَُٔطز‬٤‫طَؿ‬
b. imperforate anus ٢‫ش حُوِو‬٣َ‫وْ حُـيس حٌُظ‬٠‫ ط‬.a
c. vesicoureteral reflux ‫رش‬ٞ‫َ ٓؼو‬٤‫ كظلش َٗؽ ؿ‬.b
d. renal agenesis ٢‫ حُلخُز‬٢ٗ‫ ؿٍِ حُٔؼخ‬.c
١ٌُِٞ‫ ػيّ حُظوِن ح‬.d

Un nouveau-né par ailleurs en bonne santé dans la pouponnière a un


scrotum rugueux bien développé sans testicules palpables. Le phallus
mesure 2 cm de long avec un méat urétral à la base de la tige pénienne
juste au-dessus du scrotum. Parmi les éléments suivants, le trouble le
plus probablement associé à cette affection est
a. l‟hyperplasie congénitale des surrénales
b. anus imperforé
c. reflux vésico-urétéral
d. agénésie rénale

77
138. A 3-day-old neonate born at 28 weeks„ gestation has a patent ductus ّ‫خ‬٣‫ أ‬3 َٔ‫زِؾ ٖٓ حُؼ‬٣ ‫الىس‬ُٞ‫غ ح‬٣‫لَ كي‬١
arteriosus that was identified by echocardiography. The ductus is ‫ ه٘خس‬ٚ٣‫ ٖٓ حُلَٔ ُي‬28 ‫ع‬ٞ‫ حألٓز‬٢‫ى ك‬ُٞٞٓٝ
reported to have left-to-right (aorta-to-pulmonary artery) blood ٢٤‫ن طوط‬٣َ١ ٖ‫خ ػ‬ٛ‫ي‬٣‫ش ٓخٌُش طْ طلي‬٤ٗ‫خ‬٣َٗ
ٖٓ ّ‫ طْ حإلرالؽ ػٖ طيكن حُي‬.‫ حُوِذ‬ٟ‫ي‬ٛ
flow. The neonate is currently supported with continuous positive
٠ُ‫ ا‬١َٜ‫خٕ حألر‬٣َُ٘‫ٖ (ٖٓ ح‬٤ٔ٤ُ‫ ح‬٠ُ‫ٔخٍ ا‬٤ُ‫ح‬
airway pressure of +5 on 21% FiO2 and full enteral feeds. The ‫الىس‬ُٞ‫غ ح‬٣‫ظْ ىػْ كي‬٣ .)١ٞ‫خٕ حَُث‬٣َُ٘‫ح‬
neonate just completed a 48-hour course of antibiotics. Of the َٔ‫ ٓٔظ‬٢‫ـخر‬٣‫ ا‬٢‫حث‬ٞٛ َٟ‫ ٓـ‬٢‫ـ‬٠‫خ ر‬٤ُ‫كخ‬
following, the best management for this neonate at this time is to .‫ش ًخِٓش‬٣ٞ‫ش ٓؼ‬٣ٌ‫طـ‬ٝ FiO2 ٪21 ٠ِ‫ ػ‬5+
a. monitor 48 ‫خ‬ٜ‫ٍس ٓيط‬ٝ‫ ى‬ٞ‫الىس ُِظ‬ُٞ‫غ ح‬٣‫أًَٔ كي‬
b. administer indomethacin ‫ٖ ٓخ‬٤‫ ٖٓ ر‬.‫ش‬٣ٞ٤‫خىحص حُل‬٠ُٔ‫ٓخػش ٖٓ ح‬
c. refer for surgical ligation ‫الىس‬ُٞ‫غ ح‬٣‫ٌح حُلي‬ُٜ ‫َ ػالؽ‬٠‫ كبٕ أك‬،٢ِ٣
ٞٛ ‫هض‬ُٞ‫ٌح ح‬ٛ ٢‫ك‬
d. repeat echocardiography in 48 hours
‫ َٓحهزش‬.a
ٖ٤ٓ‫ظخ‬٤ٓٝ‫ اػطخء حإلٗي‬.b
Un nouveau-né de 3 jours né à 28 semaines de gestation a une ٢‫ حُـَحك‬٢‫ حَُر‬٠ُ‫ع ا‬ٞ‫ حَُؿ‬.c
persistance du canal artériel qui a été identifiée par ٍ‫ حُوِذ هال‬ٟ‫ي‬ٛ ٢٤‫ ُظٌَحٍ طوط‬.d
échocardiographie. Le canal aurait un flux sanguin de gauche à droite .‫ ٓخػش‬48
(aorte à artère pulmonaire). Le nouveau-né est actuellement soutenu
par une pression positive continue des voies respiratoires de +5 sur
21% de FiO2 et des alimentations entérales complètes. Le nouveau-
né vient de terminer un traitement antibiotique de 48 heures. Parmi les
éléments suivants, la meilleure prise en charge de ce nouveau-né à
l‟heure actuelle est de
a. surveiller
b. l‟administration d‟indométacine
c. référer pour la ligature chirurgicale
d. répéter l‟échocardiographie dans 48 heures

78
139. A patient has been diagnosed with achalasia. He refused surgery .‫ رظؼٌٍ حالٍطوخء‬ٞ٣َٓ ‫خرش‬ٛ‫ ا‬ٚ٤‫طْ ط٘و‬
initially, preferring to try non-operative therapy. He tried life style ‫ال طـَرش‬٠‫ ٓل‬،‫ش‬٣‫ حُزيح‬٢‫ حُـَحكش ك‬ٞ‫ٍك‬
modification, calcium channel blockers, botulin toxin injection, and ٢ٔٗ َ٣‫ٍ طؼي‬ٝ‫ كخ‬.٢‫َ حُـَحك‬٤‫حُؼالؽ ؿ‬
ٖ‫كو‬ٝ ،ّٞ٤ُٔ‫حص حٌُخ‬ٞ٘‫َحص ه‬ٛ‫كخ‬ٝ ،‫خس‬٤‫حُل‬
endoscopic pneumatic dilatation. None of the treatments alleviated his
٢‫حث‬ُٜٞ‫ٓغ ح‬ٞ‫حُظ‬ٝ ،ٖ٤ُٞ‫ط‬ٞ‫ٖ حُز‬٤ًٔٞ‫ط‬
symptoms. What are his surgical options? ٖٓ ‫ ٖٓ حُؼالؿخص‬١‫ ُْ طولق أ‬.ٍ‫رخُٔ٘ظخ‬
a. Modified Heller myotomy with partial fundoplication ‫ش؟‬٤‫ حُـَحك‬ٚ‫خٍحط‬٤‫ ه‬٢ٛ ‫ ٓخ‬.ٟٚ‫أػَح‬
b. Esophagectomy ‫ش‬٤٘‫َِ ٓغ طؼ‬٤ٛ َ٠‫غ ػ‬٠‫َ ر‬٣‫ طؼي‬.a
c. Surgical esophagomyotomy proximal to the LES ‫ش‬٤‫حُوخع حُـِث‬
d. Nissen fundoplication ‫ء‬١َُٔ‫خٍ ح‬ٜ‫ حٓظج‬.b
‫ذ‬٣َ‫ حُو‬٢‫ء حُـَحك‬١َُٔ‫غ ح‬٠‫ ر‬.c
LES ٖٓ
Un patient a reçu un diagnostic d‟achalasie. Il a d‟abord refusé la ٖ٤ٔ٤ُ٘ ‫ش حُوخع‬٤٘‫ طؼ‬.d
chirurgie, préférant essayer une thérapie non chirurgicale. Il a essayé
la modification du mode de vie, les inhibiteurs calciques, l‟injection
de toxine botulique et la dilatation pneumatique endoscopique. Aucun
des traitements n‟a soulagé ses symptômes. Quelles sont ses options
chirurgicales?
a. Myotomie de Heller modifiée avec fundoplicature partielle
b. Œsophagectomie chirurgicale
c. Oesophagomyotomie chirurgicale proximale à la LES
d. Nissen fundoplicature

140. A 72-year-old man complains of bilateral thigh and buttock ٖٓ ٌٞ٘٣ ‫ ػخٓخ‬72 َٔ‫زِؾ ٖٓ حُؼ‬٣ َ‫ٍؿ‬
claudication of several months‟ duration. He was told by his physician .َٜٗ‫حألٍىحف ُؼيس أ‬ٝ ٌ‫ حُلو‬٢‫ ك‬٢‫ػَؽ ػ٘خث‬
that the angiogram revealed findings indicating that he has Leriche ٖ‫ش ً٘ق ػ‬٤‫ػ‬ٝ‫َ حأل‬٣ٜٞ‫ إٔ ط‬ٚ‫ز‬٤‫ز‬١ َٙ‫أهز‬
.ٖ٣َ٤ُ ‫خد رٔظالُٓش‬ٜٓ ٚٗ‫ أ‬٠ُ‫َ ا‬٤٘‫ٗظخثؾ ط‬
syndrome. What does this patient have?
‫؟‬ٞ٣َُٔ‫ٌح ح‬ٛ ٟ‫ٓخًح ُي‬
a. Iliac artery aneurysm ٕ‫خ‬٣َُ٘‫ ح‬٢‫ش ك‬٣ٞٓ‫ش حُي‬٤‫ػ‬ٝ‫ طٔيى حأل‬.a
b. Tibial occlusive disease ٢‫حُلَهل‬
c. Femoropopliteal occlusive disease ‫د‬ٞ‫ حٗٔيحى حُظ٘ز‬َٝٓ .b
d. Aortoiliac occlusive disease ٢٠‫ حٗٔيحى حُلوٌ حُٔؤر‬َٝٓ .c
١َٜ‫خٕ حألر‬٣َُ٘‫ حٗٔيحى ح‬َٝٓ .d
٢‫حُلَهل‬
Un homme de 72 ans se plaint d‟une claudication bilatérale de la
cuisse et des fesses de plusieurs mois. Son médecin lui a dit que
l‟angiographie avait révélé des résultats indiquant qu‟il souffrait du
syndrome de Leriche. Qu‟est-ce que ce patient a?
a. Anévrisme de l‟artère iliaque
b. Maladie occlusive tibiale
c. Maladie occlusive fémoropoplitée
d. Maladie occlusive aortoilaïque

79
141. A 44-year-old man presents to the emergency department ٠ُ‫َ ا‬٠‫ل‬٣ ‫ ػخٓخ‬44 َٔ‫زِؾ ٖٓ حُؼ‬٣ َ‫ٍؿ‬
complaining of 3 hours of severe epigastric pain that radiates to his ٖٓ ‫ ٓخػخص‬3 ٖٓ ٌٞ٘٣ ٞٛٝ ‫حٍة‬ٞ‫هْٔ حُط‬
back. He denies experiencing similar episodes in the past but remarks .َٜٙ‫ ظ‬ٞ‫ِظق ٗل‬٣ ١ٌُ‫ي ح‬٣‫ ٗي‬٢‫ك‬َٞٓٗ ُْ‫أ‬
،٢ٟ‫ حُٔخ‬٢‫رخص ٓٔخػِش ك‬ُٞ٘ َٟٚ‫ طؼ‬٢‫٘ل‬٣ٝ
that he has been diagnosed previously with gallstones but elected to
‫ ٓخروخ‬ٜٚ٤‫ طْ ط٘و‬ٚٗ‫ أ‬٠ُ‫َ ا‬٤٘٣ ٌُٚ٘
not have a cholecystectomy. Examination and laboratory findings are ‫ حهظخٍ ػيّ اؿَحء‬ٌُٚ٘ ‫خص حَُٔحٍس‬٤ٜ‫رل‬
consistent with acute pancreatitis, and he is admitted to the hospital. ‫حُ٘ظخثؾ‬ٝ ٚ‫ حُلل‬.‫خٍ حَُٔحٍس‬ٜ‫ش حٓظج‬٤ِٔ‫ػ‬
Two months following discharge, he continues to have abdominal ،‫خّ حُلخى‬٣ٌَ٘‫خد حُز‬ٜ‫ش طظلن ٓغ حُظ‬٣َ‫حُٔوز‬
pain and elevated amylase levels. CT of the abdomen shows a ٖٓ ٖ٣َٜٗ ‫ رؼي‬.٠‫ حُٔٔظ٘ل‬٠ُ‫ ا‬ُٚ‫ظْ اىهخ‬٣ٝ
shadow in the lesser sac. Which of the following is the most likely ُْ‫ ٖٓ أ‬٢ٗ‫ؼخ‬٣ ٍ‫ِح‬٣ ‫ ال‬،٠‫ ٖٓ حُٔٔظ٘ل‬ٚ‫ؿ‬َٝ‫ه‬
diagnosis? .ُ‫ال‬٤ٓ‫خص َٓطلؼش ٖٓ حأل‬٣ٞ‫ٓٔظ‬ٝ ٖ‫ حُزط‬٢‫ك‬
ٖ‫ٓذ ُِزط‬ٞ‫ حُٔل‬٢‫َ حُٔوطؼ‬٣ٜٞ‫َ حُظ‬ٜ‫ظ‬٣
a. Pancreatic pseudocyst
ٞٛ ٢ُ‫ ٖٓ حُظخ‬١‫ أ‬.َ‫ـ‬ٛ‫ْ حأل‬٤ٌُ‫ ح‬٢‫ظال ك‬
b. Acute cholangitis ‫لخ؟‬٤‫ حألًؼَ طَؿ‬ٚ٤‫حُظ٘و‬
c. Perforated peptic ulcer ‫ش‬٤ٓ‫خ‬٣ٌَ٘‫ٔش حٌُخًرش حُز‬٤ٌُ‫ ح‬.a
d. Acute cholecystitis ‫ش حُلخى‬٣ٝ‫لَح‬ُٜ‫ش ح‬٤٘‫خد حأله‬ٜ‫ حُظ‬.b
‫ش ٓؼوزش‬٤ٔ٠ٛ ‫ هَكش‬.c
Un homme de 44 ans se présente à l‟urgence en se plaignant de 3 ‫خد حَُٔحٍس حُلخى‬ٜ‫ حُظ‬.d
heures de douleurs épigastriques sévères qui irradient vers son dos. Il
nie avoir vécu des épisodes similaires dans le passé, mais remarque
qu‟il a déjà été diagnostiqué avec des calculs biliaires, mais qu‟il a
choisi de ne pas subir de cholécystectomie. L‟examen et les résultats
de laboratoire sont compatibles avec une pancréatite aiguë, et il est
admis à l‟hôpital. Deux mois après son congé, il continue d‟avoir des
douleurs abdominales et des taux élevés d‟amylase. CT de l‟abdomen
montre une ombre dans le sac inférieur. Lequel des énoncés suivants
est le diagnostic le plus probable?
a. Pseudokyste pancréatique
b. Cholangite aiguë
c. Ulcère peptique perforé
d. Cholécystite aiguë

80
142. A previously healthy 7-year-old boy presents to the emergency ‫ظٔظغ ٓخروخ‬٣ ‫حص‬ٞ٘ٓ 7 َٔ‫زِؾ ٖٓ حُؼ‬٣ ٢‫ز‬ٛ
department with cough and runny nose for 2 weeks, and fever and ٞٛٝ ‫حٍة‬ٞ‫ هْٔ حُط‬٠ُ‫َ ا‬٠‫ل‬٣ ‫يس‬٤‫لش ؿ‬ٜ‫ر‬
headache of 1 day„s duration. On physical examination, ‫ ٖٓ ٓيس‬٢‫الٕ أٗل‬٤ٓٝ ٍ‫ ٖٓ ٓؼخ‬ٌٞ٘٣
٢‫ ك‬.‫حكي‬ٝ ّٞ٣ ‫يحع ُٔيس‬ٛٝ ٠ٔ‫ك‬ٝ ،ٖ٤‫ػ‬ٞ‫أٓز‬
his temperature is 39.4°C, heart rate is 122 beats/min, and blood
‫ ىٍؿش‬39.4 ٚ‫ ىٍؿش كَحٍط‬،٢ٗ‫ حُزي‬ٚ‫حُلل‬
pressure is 100/60 mm Hg. His physical examination shows / ‫ش‬٠‫ ٗز‬122 ‫َرخص حُوِذ‬ٟ ٍ‫ٓؼي‬ٝ ،‫ش‬٣ٞ‫ٓج‬
erythematous nasal mucosa and clear nasal discharge; all .‫ ِْٓ ُثزن‬60/100 ّ‫ حُي‬٢‫ـ‬ٟٝ ،‫وش‬٤‫ىه‬
other findings are within normal parameters. Soon after his initial ‫ ُألٗق‬٢١‫ حُـ٘خء حُٔوخ‬٢ٗ‫ حُزي‬ٜٚ‫َ كل‬ٜ‫ظ‬٣
evaluation he has a seizure. His mother reports that the seizure started ‫غ‬٤ٔ‫ ؿ‬.‫ش‬٤‫خك‬ٛ ‫اكَحُحص حألٗق‬ٝ ٢ٓ‫كٔخ‬
in his left arm and leg and then his whole body started shaking. The ‫رؼي‬ .‫ش‬٣‫َ حُؼخى‬٤٣‫ٖٔ حُٔؼخ‬ٟ َٟ‫حُ٘ظخثؾ حأله‬
seizure ends after 2 minutes, and the boy is lethargic. Over the next 10 ‫رش‬ٞ٘‫ذ ر‬٤ٛ‫ أ‬٢ُٝ‫ حأل‬ٚٔ٤٤‫ِس ٖٓ طو‬٤‫ؿ‬ٝ ‫كظَس‬
٢‫رش ريأص ك‬ُٞ٘‫ إٔ ح‬ٚ‫حُيط‬ٝ ‫ ًًَص‬.‫َع‬ٛ
minutes, he starts to wake up and move around, but his left arm and
ًِٚ ٙ‫ ػْ ريأ ؿٔي‬ٚ‫ٓخه‬ٝ َٟٔ٤ُ‫ ح‬ٚ‫ًٍحػ‬
leg are weak. Of the following, the best next step in this boy „s ٢‫ز‬ُٜ‫ح‬ٝ ،‫وش‬٤‫ ىه‬2 ‫رش رؼي‬ُٞ٘‫ ح‬٢ٜ‫ ط٘ظ‬.‫َطـق‬٣
evaluation and management is to: ‫ ريأ‬،‫ش‬٤ُ‫ حُظخ‬10 ٍ‫ هالٍ حُيهخثن ح‬.َٓ‫هخ‬
a. administer lorazepam intravenously َٟٔ٤ُ‫ ح‬ٚ‫ ٌُٖ ًٍحػ‬،‫حُظلَى‬ٝ ‫وخظ‬٤‫رخالٓظ‬
b. perform brain imaging َ٠‫ كبٕ أك‬،٢ِ٣ ‫ٖ ٓخ‬٤‫ ٖٓ ر‬.ٕ‫لظخ‬٤‫ؼ‬ٟ ٚ‫ٓخه‬ٝ
c. administer antibiotics ٚ‫اىحٍط‬ٝ ٢‫ز‬ُٜ‫ٌح ح‬ٛ ْ٤٤‫ طو‬٢‫ش ك‬٤ُ‫س طخ‬ٞ‫هط‬
d. administer acetaminophen orally :٢ٛ
١‫ي‬٣ٍٝ ّ‫زخ‬٣ُ‫ٍح‬ُٞ ‫ اػطخء‬.a
‫َ ُِيٓخؽ‬٣ٜٞ‫ اؿَحء ط‬.b
١ٞ٤‫خى ك‬٠ٓ ‫ اػطخء‬.c
Un garçon de 7 ans auparavant en bonne santé se présente à l‟urgence ‫ن‬٣َ١ ٖ‫ٖ ػ‬٤‫ك‬ٞ٘٤ٓ‫ظخ‬٤ٓ‫ اػطخء ح‬.d
avec une toux et un écoulement nasal pendant 2 semaines, ainsi que ْ‫حُل‬
de la fièvre et des maux de tête d‟une durée de 1 jour. À l‟examen
physique, sa température est de 39,4 °C, sa fréquence cardiaque est de
122 battements / min et sa pression artérielle est de 100/60 mm Hg.
Son examen physique montre une muqueuse nasale érythémateuse et
un écoulement nasal clair; Tous les autres résultats sont dans les
paramètres normaux. Peu de temps après son évaluation initiale, il a
une crise. Sa mère rapporte que la crise a commencé dans son bras et
sa jambe gauches, puis tout son corps a commencé à trembler. La
crise se termine après 2 minutes et le garçon est léthargique. Au cours
des 10 minutes suivantes, il commence à se réveiller et à bouger, mais
son bras et sa jambe gauches sont faibles. Parmi les éléments suivants,
la meilleure étape suivante dans l‟évaluation et la prise en charge de
ce garçon est de:
a. administrer du lorazépam par voie intraveineuse
b. effectuer une imagerie cérébrale
c. administrer des antibiotiques
d. administrer de l‟acétaminophène par voie orale

81
143. A 9-month-old infant is brought in by her parents because she has an َٜٗ‫ أ‬9 َٔ‫ؼش طزِؾ ٖٓ حُؼ‬٤ٍٟ ٍ‫خ‬٠‫ظْ اك‬٣
umbilical hernia. Physical examination shows an umbilical defect .١َٓ ‫خرش رلظن‬ٜٓ ‫خ‬ٜٗ‫خ أل‬ٜ٣‫حُي‬ٝ َ‫ٖٓ هز‬
about 1 cm in diameter, with a small bulge when the girl cries. The َٙ‫زِؾ هط‬٣ ‫خ‬٣َٓ ‫زخ‬٤‫ ػ‬٢ٗ‫ حُزي‬ٚ‫َ حُلل‬ٜ‫ظ‬٣
٢ٌ‫َ ػ٘يٓخ طز‬٤‫ـ‬ٛ ‫ ٓغ حٗظلخم‬،ْٓ 1 ٢ُ‫ح‬ٞ‫ك‬
hernial contents can be easily reduced. The hernia is not painful, and
.‫ُش‬ٜٞٔ‫خص حُلظن ر‬٣ٞ‫َ ٓلظ‬٤ِ‫ٌٖٔ طو‬٣ .‫حُطلِش‬
the girl is otherwise asymptomatic. Which of the following is the ١‫ أ‬.ٝ‫ٕ أػَح‬ٝ‫حُطلِش ري‬ٝ ،‫ْ ٓئُٔخ‬٤ُ ‫حُلظن‬
most appropriate next step in management? ‫َ؟‬٤‫ حُظير‬٢‫ش ك‬٤ُ‫س طخ‬ٞ‫ أٗٔذ هط‬ٞٛ ٢ِ٣ ‫ٓٔخ‬
a. Mesh repair ‫الف حُ٘زٌش‬ٛ‫ ا‬.a
b. Urgent surgical repair َ‫ حُؼخؿ‬٢‫الف حُـَحك‬ٛ‫ حإل‬.b
c. Observation ‫ حُٔالكظش‬.c
d. Herniotomy ‫غ حُلظن‬٠‫ ر‬.d

Un nourrisson de 9 mois est amené par ses parents parce qu‟il a une
hernie ombilicale. L‟examen physique montre un défaut ombilical
d‟environ 1 cm de diamètre, avec un petit renflement lorsque la fille
pleure. Le contenu de la hernie peut être facilement réduit. La hernie
n‟est pas douloureuse et la fille est par ailleurs asymptomatique.
Laquelle des étapes suivantes est la prochaine étape la plus appropriée
dans la gestion?
a. Réparation de mailles
b. Réparation chirurgicale urgente
c. Observation
d. Herniotomie

144. A 29-year-old construction worker fell 15 feets from a roof and broke ٠ِ‫ ػخٓخ ػ‬29 َٔ‫زِؾ ٖٓ حُؼ‬٣ ‫ ػخَٓ ر٘خء‬٢‫ٓو‬
his right humerus, as depicted in the accompanying radiograph. Given ‫ي‬٠‫ًَٔ ػظْ حُؼ‬ٝ ‫ هيٓخ ٖٓ ٓطق‬15 ‫رؼي‬
his injury is at the spiral groove, which of the following nerves is ‫ٍس‬ُٜٞ‫ ح‬٢‫ق ك‬ٟٞٓ ٞٛ ‫ ًٔخ‬،ٖٔ٣‫حأل‬
٢‫ ك‬ٚ‫خرظ‬ٛ‫ اًح ًخٗض ا‬.‫ش حَُٔكوش‬٤‫حُ٘ؼخػ‬
most at risk?
‫ش‬٤‫ط‬٥‫خد ح‬ٜ‫ حألػ‬١‫ كؤ‬،٢ِِٗٝ‫ى حُل‬ٝ‫حألهي‬
a. Median nerve ‫ش ُِوطَ؟‬َٟ‫أًؼَ ػ‬
b. Radial nerve ٢ٓٞ‫ذ حُٔظ‬ٜ‫ حُؼ‬.a
c. Ulnar nerve ١َ‫ذ حٌُؼز‬ٜ‫ حُؼ‬.b
d. Posterior interosseous nerve ١‫ذ حُِٗي‬ٜ‫ حُؼ‬.c
ّ‫ٖ حُؼظخ‬٤‫ ر‬٢‫ذ حُوِل‬ٜ‫ حُؼ‬.d
Un travailleur de la construction de 29 ans est tombé de 15 pieds d‟un
toit et s‟est cassé l‟humérus droit, comme le montre la radiographie
ci-jointe. Étant donné que sa blessure est au sillon en spirale, lequel
des nerfs suivants est le plus à risque?
a. Nerf médian
b. Nerf radial
c. Nerf cubital
d. Nerf interosseux postérieur

82
145. In second-degree AV block, Möbitz type II: ‫ ٖٓ حُيٍؿش‬٢٘٤‫ حُزط‬٢٘٣ً‫ حُلـذ حأل‬٢‫ك‬
a. The PR interval is fixed and prolonged. :II ٢ٔٗ ِ‫ظ‬٤‫ر‬ٞٓ ،‫ش‬٤ٗ‫حُؼخ‬
b. The PR interval gets longer and longer until finally one .PR ‫خُش كظَس‬١‫ا‬ٝ ‫الف‬ٛ‫ظْ ا‬٣ .a
beat drops. ٠ُ‫ٍ ا‬ٞ١‫أ‬ٝ ٍٞ١‫زق أ‬ٜ‫ ط‬PR ‫ كظَس‬.b
c. The PR interval remains unchanged but the P wave .‫َح‬٤‫حكيس أه‬ٝ ‫ش‬٠‫َ ٗز‬ٜ‫إٔ طل‬
suddenly fails to conduct to the ventricles. ‫ؿش‬ٞٓ ٌُٖٝ َ٤٤‫ٕ طـ‬ٝ‫ ى‬PR ٠‫ طزو‬.c
d. The P waves are dissociated from the QRS complexes. ٠ُ‫َ ا‬٤ٛٞ‫ اؿَحء حُظ‬٢‫ طلَ٘ ك‬P
e. Also known as the Wenckebach phenomenon. .ٖ٤٘٤‫حُزط‬
‫ ػٖ ٓؼويحص‬P ‫ؿخص‬ٞٓ َٜ‫ظْ ك‬٣ .d
Dans le bloc AV du deuxième degré, Möbitz type II: .QRS
a. L‟intervalle PR est fixe et prolongé. .‫ٌ٘زخم‬٣ٝ ‫َس‬ٛ‫خ رظخ‬٠٣‫ طؼَف أ‬.e
b. L‟intervalle PR devient de plus en plus long jusqu‟à ce
qu‟un battement tombe finalement.
c. L‟intervalle PR reste inchangé mais l‟onde P ne parvient
soudainement pas à se diriger vers les ventricules.
d. Les ondes P sont dissociées des complexes QRS.
e. Aussi connu sous le nom de phénomène de Wenckebach.

146. Miliary Tuberculosis refers to: :٠ُ‫َ ا‬٤٘٣ ٢٘‫حَُٔ حُيه‬


a. Systemic disease that may affect multiple organs ‫ذ‬٤ٜ٣ ٕ‫ٌٖٔ أ‬٣ ١ٌُ‫ ح‬١ُ‫خ‬ٜ‫ ىحء ؿ‬.a
b. Patient with a prior latent infection with tuberculosis .‫خء ٓظؼيىس‬٠‫أػ‬
c. Patient with pulmonary fibrosis secondary to tuberculosis ‫ ًخٓ٘ش ٓخروخ‬ٟٝ‫خد رؼي‬ٜٓ ٞ٣َٓ .b
d. Patient with peritoneal adhesions secondary to tuberculosis .َُٔ‫ٓغ ىحء ح‬
e. Patient with neurologic disability secondary to ٢ُ‫ طخ‬١ٞ‫ق ٍث‬٤ِ‫خد رظ‬ٜٓ ٞ٣َٓ .c
tuberculosis .َُٔ‫ُيحء ح‬
‫ش‬٤‫لخه‬ٛ ‫خهخص‬ٜ‫خد رخُظ‬ٜٓ ٞ٣َٓ .d
La tuberculose miliaire fait référence à: .َُٔ‫ش ُيحء ح‬٤ُ‫طخ‬
a. Maladie systémique pouvant affecter plusieurs organs ٢ُ‫ طخ‬٢‫ز‬ٜ‫خد رؼـِ ػ‬ٜٓ ٞ٣َٓ .e
b. Patient ayant une infection latente antérieure par la .َُٔ‫ُيحء ح‬
tuberculose
c. Patient atteint de fibrose pulmonaire secondaire à la
tuberculose
d. Patient présentant des adhérences péritonéales secondaires
à la tuberculose
e. Patient présentant un incapacité neurologique secondaire à
la tuberculose

83
147. The most common bladder tumor is: :ٞٛ ‫ػخ‬ٞ٤ٗ َ‫ٍّ حُٔؼخٗش حألًؼ‬ٝ
a. Transitional cell carcinoma .‫ش‬٤ُ‫خ حالٗظوخ‬٣‫ حُوال‬ٚٗ‫خ‬١َٓ .a
b. Squamous cell carcinoma .‫ش‬٤‫خ حُلَٗل‬٣‫ حُوال‬ٚٗ‫خ‬١َٓ .b
c. Adenocarcinoma .‫ش‬٣‫ ؿي‬ٚٗ‫خ‬١َٓ .c
d. Small cell carcinoma .‫َس‬٤‫ـ‬ُٜ‫خ ح‬٣‫ حُوال‬ٚٗ‫خ‬١َٓ .d
e. Sarcoma .‫ٓخ‬ًٍٞ‫ ٓخ‬.e
La tumeur de la vessie la plus courante est :
a. Carcinome à cellules transitionnelles
b. Carcinome épidermoïde
c. Adénocarcinome
d. Carcinome à petites cellules
e. Sarcome
148. Child scoring system to assess liver disease severity included all ‫خرش‬ٛ‫ْ ٗيس حإل‬٤٤‫لخٍ ُظو‬١‫حُ٘ظخّ حُٔظزغ ػ٘ي حأل‬
except: :‫ ٓخ ػيح‬٢ِ٣ ‫َ٘ٔ ًَ ٓخ‬٣ ١‫ ًزي‬َٝٔ‫ر‬
a. Bilirubin .ٖ٤‫ر‬َٝ٤ِ٤‫ ر‬.a
b. Albumin .ٖ٤ٓٞ‫ أُز‬.b
c. Hemoglobin .ٖ٤‫ر‬ِٞ‫ؿ‬ٞٔ٤ٛ .c
d. INR .INR .d
e. Hepatic encephalopathy .١‫ ًزي‬٢‫ حػظالٍ ىٓخؿ‬.e

Le système de notation de l‟enfant pour évaluer la gravité de la


maladie du foie comprenait tous sauf :

a. Bilirubine
b. Albumine
c. Hémoglobine
d. INR
e. Encéphalopathie hépatique

149. The most common primary tumor in the brain is: :ٞٛ ‫ حُيٓخؽ‬٢‫ػخ ك‬ٞ٤ٗ َ‫ حألًؼ‬٢ٓ‫ٍّ حألٓخ‬ُٞ‫ح‬
a. Glioma
b. Meningioma .٢‫ٍّ ىرو‬ٝ .a
c. Primary CNS lymphoma .٢‫ٍّ ٓلخث‬ٝ .b
d. Medulloblastoma ‫ حُـِٔش‬٢‫ ك‬٢ٓ‫ أٓخ‬١ٝ‫ٍّ ُٔلخ‬ٝ .c
e. Pituitary Tumor .‫ش‬٣ًَُِٔ‫ش ح‬٤‫ز‬ٜ‫حُؼ‬
.٢‫ ٗوخػ‬٢ٍٓٝ‫ٍّ أ‬ٝ .d
La tumeur primaire la plus courante dans le cerveau est: .٢ٓ‫ٍّ ٗوخ‬ٝ .e
a. Gliome
b. Méningiome
c. Lymphome primitif du SNC
d. Médulloblastome
e. Tumeur hypophysaire

84
150. Jaundice without bilirubin in the urine is seen in: :٢‫ي ك‬ٛ‫٘خ‬٣ ٍٞ‫ حُز‬٢‫ٖ ك‬٤‫ر‬َٝ٤ِ٤‫ٕ ر‬ٝ‫َهخٕ ري‬٣
a. Carcinomatosis .َ٘‫ ٓ٘ظ‬٢ٗ‫خ‬١َٓ ‫ ىحء‬.a
b. Acquired haemolytic anemia .‫ ٌٓظٔذ‬٢ُ‫ كوَ ىّ حٗلال‬.b
c. Obstruction of common bile duct .‫ش حُ٘خثؼش‬٣ٝ‫لَح‬ُٜ‫ حٗٔيحى حُو٘خس ح‬.c
d. Infective hepatitis .٢ٗ‫ حٗظخ‬١‫خد ًزي‬ٜ‫ حُظ‬.d
e. Chlorpromazine hepatitis .ٖ٣ُ‫ٓخ‬َٝ‫ٍر‬ٌُِٞ‫ ٖٓ ح‬١‫خد ًزي‬ٜ‫ حُظ‬.e

Ictère sans bilirubine dans l‟urine est observée dans:


a. Carcinomatose
b. Anémie hémolytique acquise
c. Obstruction du canal biliaire commun
d. Hépatite infectieuse
e. Hépatite de chlorpromazine
151. A 50-year-old woman has a two-week history of acute and ٌ٘ٓ ٢ٗ‫ ػخٓخ طؼخ‬50 َٔ‫حَٓأس طزِؾ ٖٓ حُؼ‬
progressive difficulty in walking and weakness in her limbs. On ٢ُ٘ٔ‫ٓظيٍؿش رخ‬ٝ ‫رش كخىس‬ٞ‫ؼ‬ٛ ٖٓ ٖ٤‫ػ‬ٞ‫أٓز‬
examination, there was proximal and distal limb weakness which is ٕ‫ ًخ‬،ٚ‫ ػ٘ي حُلل‬.‫خ‬ٜ‫َحك‬١‫ أ‬٢‫ؼق ك‬ٟٝ
more marked in the legs than the arms. All tendon reflexes were ‫يس‬٤‫حُزؼ‬ٝ ‫زش‬٣َ‫َحف حُو‬١‫ حأل‬٢‫ؼق ك‬ٟ ‫٘خى‬ٛ
absent and the plantar responses were flexor. There was no sensory .ٖ٤‫ٖ ٖٓ حٌٍُحػ‬٤‫ حُٔخه‬٢‫كخ ك‬ٟٞٝ َ‫ أًؼ‬ٞٛٝ
loss. What is the most likely diagnosis? ‫حُٔ٘ؼٌٔخص‬ٝ ‫ش ؿخثزش‬٣َ‫ط‬ُٞ‫غ حُٔ٘ؼٌٔخص ح‬٤ٔ‫ؿ‬
a. Cervical cord compression ٕ‫٘خى كويح‬ٛ ٌٖ٣ ُْ .‫ش‬٤٘‫ش ٓؼ‬٤ٜٔ‫حأله‬
b. Guillain-Barre syndrome :‫لخ؟‬٤‫ حألًؼَ طَؿ‬ٚ٤‫ حُظ٘و‬ٞٛ ‫ ٓخ‬.ّ‫اكٔخ‬
c. Myasthenia gravis .٢‫ حُؼ٘و‬٢ًُٞ٘‫ حُلزَ ح‬٠ِ‫ ػ‬٢‫ـ‬ٟ .a
d. Polymyalgia rheumatica .١ٍ‫رخ‬-ٕ‫ال‬٤‫ ٓظالُٓش ؿ‬.b
e. Polymyositis .ْ٤‫ه‬ٝ ٢ِ٠‫ٖ ػ‬ٛٝ .c
.١ٞ٤‫ ٓظؼيى ٍػ‬٢ِ٠‫ أُْ ػ‬.d
Une femme de 50 ans a des antécédents de deux semaines de .‫الص‬٠‫خد حُؼ‬ٜ‫ حُظ‬.e
difficulté aiguë et progressive à marcher et de faiblesse dans ses
membres. À l‟examen, il y avait une faiblesse proximale et distale des
membres qui est plus marquée dans les jambes que dans les bras.
Tous les réflexes tendineux étaient absents et les réponses plantaires
étaient fléchisseurs. Il n‟y a pas eu de perte sensorielle. Quel est le
diagnostic le plus probable?
a. Compression du cordon cervicale
b. Syndrome de Guillain-Barré
c. Myasthénie grave
d. Polymyalgie rhumatismale
e. Polymyosite

85
152. A 30-year-old woman presents to the colorectal outpatient clinic with ٠ُ‫َ ا‬٠‫ ػخٓخ طل‬30 َٔ‫حَٓأس طزِؾ ٖٓ حُؼ‬
severe burning perianal pain on defaecation. The pain persists for up ٢ٛٝ ‫ش‬٤‫ٕ حُوخٍؿ‬ُٞٞ‫حُو‬ٝ ْ٤‫خىس حُٔٔظو‬٤‫ػ‬
to an hour following defaecation, and she often notices spots of fresh ‫ي‬٣‫ رخَُ٘ؽ ٓغ كَم ٗي‬٢٤‫ ٖٓ أُْ ٓل‬٢ٗ‫طؼخ‬
red blood on the toilet paper. There is no history of a fever. What is ‫َ ُٔخػش‬ٜ‫ٔظَٔ حألُْ ُٔيس ط‬٣ .١ٞ‫ػ٘ي حُظـ‬
the most likely diagnosis? ٖٓ ‫ ؿخُزخ ٓخ طالكع روؼخ‬٢ٛٝ ،١ٞ‫رؼي حُظـ‬
a. Anal fissure ‫ ال‬.‫ض‬٤ُ‫ح‬ٞ‫ٍم حُظ‬ٝ ٠ِ‫حُيّ حألكَٔ حُطخُؽ ػ‬
b. Fistula in ano ٚ٤‫ حُظ٘و‬ٞٛ ‫ ٓخ‬.٠ٔ‫ن ٖٓ حُل‬٣ٍ‫ؿي طخ‬ٞ٣
c. Hemorrhoids :‫لخ؟‬٤‫حألًؼَ طَؿ‬
d. Perianal abscess .٢‫ ٗن َٗؿ‬.a
e. Perineal hematoma .٢‫ٍ َٗؿ‬ٞٓ‫ ٗخ‬.b
.َ٤ٓ‫ح‬ٞ‫ ر‬.c
Une femme de 30 ans se présente à la clinique externe colorectale .‫ٍ حَُ٘ؽ‬ٞ‫ هَحؽ ك‬.d
avec de graves douleurs périanales brûlantes lors de la défécation. La .٢ٗ‫ ػـخ‬١ٞٓ‫ٍّ ى‬ٝ .e
douleur persiste jusqu‟à une heure après la défécation et elle remarque
souvent des taches de sang rouge frais sur le papier toilette. Il n‟y a
pas d‟antécédents de fièvre. Quel est le diagnostic le plus probable?
a. Fissure anale
b. Fistule anale
c. Les hémorroïdes
d. Abcès périanal
e. Hématome périnéal

153. A young man is admitted to the emergency room for a stab in the ٢‫حٍة ُطؼ٘ش ك‬ٞ‫ ؿَكش حُط‬٠ُ‫أىهَ ٗخد ا‬
abdomen one hour ago. There is no major external bleeding. His BP is ٢‫ق هخٍؿ‬٣ِٗ ‫ؿي‬ٞ٣ ‫ ال‬.‫ هزَ ٓخػش‬ٚ٘‫رط‬
80/30, Pulse is 130bpm and he looks pale. The next thing to do is: /‫ش‬٠‫ ٗز‬130 ٞ‫ حُ٘ز‬،30/80 ٚ‫ـط‬ٟ .َ٤‫ًز‬
a. Take a detailed history ‫ـذ‬٣ ١ٌُ‫ ح‬٢ُ‫ء حُظخ‬٢ُ٘‫ ح‬.‫ ٗخكزخ‬ٝ‫زي‬٣ٝ ‫وش‬٤‫ىه‬
b. Ask for an abdominal ultrasound :‫؟‬ٞٛ ٚ‫خّ ر‬٤‫حُو‬
c. Ask for a CT Scan .َٜ‫ن ٓل‬٣ٍ‫ أهٌ طخ‬.a
d. Ask for an abdominal MRI ‫م‬ٞ‫ؿخص ك‬ُٞٔ‫ رخ‬ٚ‫ِذ اؿَحء كل‬١ .b
e. Prepare for immediate laparotomy .ٖ‫ش ُِزط‬٤‫ط‬ُٜٞ‫ح‬
.٢‫َ ٓوطؼ‬٣ٜٞ‫ِذ اؿَحء ط‬١ .c
Un jeune homme est admis aux urgences pour un coup de couteau ٖ٤َُٗ‫َ رخ‬٣ٜٞ‫ِذ اؿَحء ط‬١ .d
dans l‟abdomen il y a une heure. Il n‟y a pas de saignement externe .ٖ‫ ُِزط‬٢ٔ٤١‫حُٔـ٘خ‬
majeur. Sa tension artérielle est de 80/30, son pouls est de 130 bpm et .١ٍٞ‫َ ُلظق رطٖ ك‬٤٠‫ حُظل‬.e
il a l‟air pâle. La prochaine chose à faire est:
a. Prenez un historique détaillé
b. Demandez une échographie abdominale
c. Demandez un Tomodenditométrie
d. Demandez une IRM abdominale
e. Préparez-vous à une laparotomie immediate

86
154. A 77-year-old male presents with significant postherpetic neuralgia in ُْ‫ ٖٓ أ‬٢ٗ‫ؼخ‬٣ ‫ ػخٓخ‬77 َٔ‫زِؾ ٖٓ حُؼ‬٣ َ‫ٍؿ‬
a chest wall distribution. ٠ِ‫ُع ػ‬ٞ‫ظ‬٣ ٍُ‫َرْ رخ‬ُِٜ ٢ُ‫ طخ‬٢‫ز‬ٜ‫ػ‬
Which one of the following is most likely to be effective in ٕ‫ حألًؼَ حكظٔخال أ‬ٞٛ ٢ُ‫ ٖٓ حُظخ‬١‫ أ‬.ٍ‫ي‬ُٜ‫ح‬
diminishing his discomfort? :‫؟‬ُٚٔ‫ حُلي ٖٓ أ‬٢‫ٕ كؼخال ك‬ٌٞ٣
a. Oral valacyclovir (Valtrex) ْ‫ن حُل‬٣َ١ ٖ‫َ ػ‬٤‫ك‬ٌِٞ٤ٓ‫ كخال‬.a
b. Topical lidocaine (Xylocaine) patches .)ًَْ‫(كخُظ‬
c. Thoracic epidural corticosteroid injections ‫ش‬٤‫ؼ‬ٟٞٓ ٖ٤‫ًخث‬ٝ‫ي‬٤ُ ‫خهش‬ُٜ .b
d. Herpes zoster vaccine .)ٖ٤‫ًخث‬ِٞ٣ًِ(
e. Acupuncture ‫ش‬٣ٍ‫ي‬ٛ ‫ي‬٤‫ث‬َٝ٤‫ٓظ‬ٌٞ٤‫ٍط‬ًٞ ٖ‫ كو‬.c
.‫ش‬٤‫م حُـخك‬ٞ‫ك‬
Un homme de 77 ans présente une névralgie post-herpétique .٢‫َرْ حُ٘طخه‬ُٜ‫ ُوخف ح‬.d
importante dans une distribution de la paroi thoracique. Lequel des .َ‫هِ رخإلر‬ُٞ‫ ح‬.e
éléments suivants est le plus susceptible d‟être efficace pour diminuer
son inconfort?
a. Valacyclovir oral (Valtrex)
b. Timbres topiques de lidocaïne (xylocaïne)
c. Injections de corticostéroïdes épiduraux thoraciques
d. Vaccin contre le zona
e. Acupuncture

155. A 55-year-old woman with progressive but episodic muscle weakness ٖٓ ٢ٗ‫ ػخٓخ طؼخ‬55 َٔ‫حَٓأس طزِؾ ٖٓ حُؼ‬
is diagnosed as having myasthenia gravis. Her chest radiograph is ْ‫هي ط‬ٝ ٢َٟ‫ ٌُٖ ػ‬١‫خػي‬ٜ‫ ط‬٢ِ٠‫ؼق ػ‬ٟ
normal and reveals no evidence of mediastinal mass or tumor. What is ٍ‫ي‬ُٜ‫ أٗؼش ح‬.ْ٤‫ه‬ٝ ٢ِ٠‫ٖ ػ‬ٛٞ‫خ ر‬ٜٜ٤‫ط٘و‬
the most definitive treatment that can be offered to this patient? ٍّٝ ٝ‫َ ٌُظِش أ‬٤ُ‫ ى‬١‫ال طٌ٘ق ػٖ أ‬ٝ ‫ش‬٤‫ؼ‬٤‫ز‬١
a. Prednisone ٌٖٔ٣ ‫ ٖٓ حُٔئًي‬١ٌُ‫ حُؼالؽ ح‬ٞٛ ‫ ٓخ‬.٢‫ل‬ٜ٘ٓ
b. Neostigmine :‫ش؟‬٠٣َُٔ‫ ح‬ٌُٜٙ ٚٔ٣‫طوي‬
c. Thymectomy .ِٕٝ٤ٗ‫ رَى‬.a
d. Plasmapheresis .ٖ٤ٔ‫ٓظـ‬ٞ٤ٗ .b
e. Atropine .‫طش‬ٞ‫خٍ حُظ‬ٜ‫ حٓظج‬.c
.‫خىس حُزالُٓخ‬ٜ‫ ك‬.d
Une femme de 55 ans présentant une faiblesse musculaire progressive .ٖ٤‫ر‬َٝ‫ أط‬.e
mais épisodique est diagnostiquée comme ayant une myasthénie
grave. Sa radiographie thoracique est normale et ne révèle aucune
preuve de masse médiastinale ou de tumeur. Quel est le traitement le
plus définitif qui puisse être proposé à ce patient?
a. Prednisone
b. Néostigmine
c. Thymectomie
d. Plasmaphérèse
e. Atropine

87
156. A14-year-old female presents to the emergency room 20 hours ٠ُ‫َ ا‬٠‫ ػخٓخ طل‬14 َٔ‫ طزِؾ ٖٓ حُؼ‬٠‫أٗؼ‬
following ingestion of 8.5 g of acetaminophen. Therapy with which of ‫خ‬ٜ‫ حرظالػ‬٠ِ‫ ٓخػش ػ‬20 ‫حٍة رؼي‬ٞ‫ؿَكش حُط‬
the following should be initiated? ‫زيأ حُؼالؽ‬٣ ٕ‫ـذ أ‬٣ .ٖ٤‫ك‬ٞ٘٤ٓ‫ظخ‬٤ٓ‫ ؽ أ‬8.5ٍ
a. Deferoxamine :‫؟‬٢ُ‫ ٖٓ حُظخ‬١‫رؤ‬
b. Physostigmine .ٖ٤ٓ‫ًٔخ‬َٝ٤‫ ىك‬.a
c. N-acetylcysteine .ٖ٤ٔ‫ٓظـ‬ِٝ٤‫ ك‬.b
d. Glutathione .ٖ٤‫ٔظج‬٤ٓ َ٤‫أٓظ‬-N .c
e. No therapy is indicated .ٕٞ٤‫طخط‬ِٞ‫ ؿ‬.d
.‫ ػالؽ‬٠ُ‫٘خٍ ا‬٣ ‫ ال‬.e
Une jeune femme de 14 ans se présente à l‟urgence 20 heures après
l‟ingestion de 8.5 g d‟acétaminophène. Thérapie avec laquelle des
suivantes devrait être initiée?
a. Déféroxamine
b. Physostigmine
c. N-acétylcystéine
d. Glutathion
e. Aucun traitement n‟est indiqué

157. A 55-year-old nonsmoking male presents with hemoglobin of ،ٖ‫َ ٓيه‬٤‫ ػخٓخ ؿ‬55 َٔ‫زِؾ ٖٓ حُؼ‬٣ ًًَ
18.0g/dL, and hematocrit of 50%. Splenomegaly is noted on ‫ض‬٣ًَٞ‫ٔخط‬٤ُٜ‫ح‬ٝ َٓ‫ ى‬/‫ ؽ‬18 ّ‫خد حُي‬٠‫ه‬
examination. Which of the following is the most likely diagnosis? .ٍ‫لخ‬١ ‫وخٓش‬ٟ ‫كع‬ُٞ ٚ‫ ػ٘ي حُلل‬.%50
a. Polycythemia vera :‫لخ؟‬٤‫ حألًؼَ طَؿ‬ٚ٤‫ حُظ٘و‬ٞٛ ٢ُ‫ ٖٓ حُظخ‬١‫أ‬
b. Idiopathic myelofibrosis .‫ش‬٤‫و‬٤‫ ًؼَس حُلَٔ حُلو‬.a
c. Secondary erythrocytosis .‫ٍ حُٔزذ‬ٜٞ‫ ٓـ‬٢٤‫ق ٗو‬٤ِ‫ ط‬.b
d. Myelodysplastic syndrome .١ٞٗ‫خص كَٔحء ػخ‬٣ًَ ‫ ًؼَس‬.c
e. Chronic myelogenous leukemia .٢‫ ٓظالُٓش هَِ حُظ٘ٔؾ حُ٘وخػ‬.d
.ِٖٓٓ ١ٞ‫ ٗو‬ٝ‫خ‬٠٤‫ حر‬.e
Un homme non-fumeur de 55 ans présente une hémoglobine de 18.0 g
/ dL et un hématocrite de 50%. La splénomégalie est notée à
l‟examen. Lequel des éléments suivants est le diagnostic le plus
probable?
a. Polycythémie vera
b. Myélofibrose idiopathique
c. Érythrocytose secondaire
d. Syndrome myélodysplasique
e. Leucémie myélogène chronique

88
158. A 12-year-old girl is diagnosed with asthma. She has nighttime ‫ ػخٓخ‬12 َٔ‫ كظخس طزِؾ ٖٓ حُؼ‬ٚ٤‫ظْ ط٘و‬٣
symptoms twice a week and daily daytime symptoms. Which of the ٢‫ٖ ك‬٤‫ش َٓط‬٤ِ٤ُ ٝ‫خ أػَح‬ٜ٣‫ ُي‬.ٞ‫رخَُر‬
following should NOT be part of her long-term treatment? ٖٓ ١‫ أ‬.‫ش‬٤ٓٞ٣ ‫ش‬٣ٍ‫خ‬ٜٗ ٝ‫أػَح‬ٝ ‫ع‬ٞ‫حألٓز‬
a. Inhaled steroids ‫خ‬ٜ‫ٕ ؿِء ٖٓ ػالؿ‬ٌٞ٣ ٕ‫ أ‬٢‫٘زـ‬٣ ‫ ال‬٢ُ‫حُظخ‬
b. Leukotriene-receptor antagonist :‫َ؟‬٣ٞ‫ حُط‬ٟ‫ حُٔي‬٠ِ‫ػ‬
c. Short-acting beta agonist .٢‫ي حٓظ٘٘خه‬٤‫ث‬َٝ٤‫ ٓظ‬.a
d. Oral prednisone .ٖ٤٣َ‫ط‬ٌٞ٤ُِ‫خى ٓٔظوزالص ح‬٠ٓ .b
e. Long-acting beta agonist .ٟ‫َ حُٔي‬٤ٜ‫ظخ ه‬٤‫ ٓوِي ر‬.c
.ْ‫ن حُل‬٣َ١ ٖ‫ٕ ػ‬ِٝ٤ٗ‫ رَى‬.d
Une fillette de 12 ans reçoit un diagnostic d‟asthme. Elle a des .ٟ‫َ حُٔي‬٣ٞ١ ‫ظخ‬٤‫ ٓوِي ر‬.e
symptômes nocturnes deux fois par semaine et des symptômes
diurnes quotidiens. Lequel des éléments suivants ne devrait PAS faire
partie de son traitement à long terme?
a. Stéroïdes inhalés
b. Antagoniste des récepteurs leucotriènes
c. Bêta-agoniste à courte durée d‟action
d. Prednisone orale
e. Bêta-agoniste à action prolongée

159. Phenotype features for Down syndrome in a 8 year old girl include all ‫ٕ ػ٘ي‬ٝ‫ ُٔظالُٓش ىح‬١َٛ‫ حُظخ‬٢ُٔ٘‫ِحص ح‬٤ٔٓ
of the following except: ‫حص طَ٘ٔ ًَ ٓخ‬ٞ٘ٓ 8 َٔ‫كظخس طزِؾ ٖٓ حُؼ‬
a. Flattened nasal bridge :‫ ٓخ ػيح‬٢ِ٣
b. Clinodactyly .‫ ٓٔطق‬٢‫ ؿَٔ أٗل‬.a
c. Tongue protrusion .‫خرغ‬ٛ‫ حٗلَحف حأل‬.b
d. Cleft lip .ٕ‫ُ حُِٔخ‬َٝ‫ ر‬.c
e. Low-implanted ears .‫هش‬ٞ‫ ٗلش ٓ٘و‬.d
.‫ال‬٤ِ‫ٓش ه‬َٝ‫ٖ ٓـ‬٤ًٗ‫ أ‬.e
Les caractéristiques phénotypiques du syndrome de Down chez une
fille de 8 ans comprennent toutes les caractéristiques suivantes, sauf:
a. Pont nasal aplati
b. Clinodactylie
c. Saillie de la langue
d. Fente labiale
e. Oreilles peu implantées

160. Headache associated with a brain tumor is most common in which of ٌٕٞ٣ ‫ش‬٤ُ‫ش حُظخ‬٣َ٣َُٔ‫َ ح‬ٛ‫ح‬ٞ‫ ٖٓ حُظ‬١‫ أ‬٢‫ك‬
the following clinical settings? ٞٛ ٢‫ٍّ حُيٓخؿ‬ُٞ‫يحع حُٔظَحكن ٓغ ح‬ُٜ‫ح‬
a. Upon awakening in the morning :‫ػخ؟‬ٞ٤ٗ َ‫حألًؼ‬
b. After eating breakfast .‫زق‬ُٜ‫ ح‬٢‫وخظ ك‬٤‫ ػ٘ي حالٓظ‬.a
c. After eating lunch .٢‫زخك‬ُٜ‫ٍ ح‬ٞ‫ٍ حُلط‬ٝ‫ رؼي ط٘خ‬.b
d. After eating dinner .‫ٍ حُـيحء‬ٝ‫ رؼي ط٘خ‬.c
e. During the night .‫ٍ حُؼ٘خء‬ٝ‫ رؼي ط٘خ‬.d
Les maux de tête associés à une tumeur cérébrale sont les plus .َ٤ُِ‫ هالٍ ح‬.e
fréquents dans lequel des contextes cliniques suivants?
a. Au réveil le matin
b. Après avoir pris le petit déjeuner
c. Après avoir déjeuné
d. Après avoir dîné
e. Au cours de la nuit

89
161. A patient with atrial fibrillation presents with acute pain in the left ٞٛٝ َ٠‫ ك‬،٢٘٣ً‫ ٖٓ ٍؿلخٕ أ‬٢ٗ‫ؼخ‬٣ ٞ٣َٓ
lower limb which progressed to loss of sensation and feeling of cold َٔ٣‫ حأل‬٢ِ‫ حُطَف حُٔل‬٢‫ ٖٓ أُْ كخى ك‬ٌٞ٘٣
and numbness over few hours. What is the first treatment should start ‫ىس‬َٝ‫ٍ رخُز‬ٞ‫حُ٘ؼ‬ٝ ّ‫ كويحٕ حإلكٔخ‬٠ُ‫ٍ ا‬ٞ‫طط‬
a. Prepare the patient for embolectomy ‫ حُؼالؽ‬ٞٛ ‫ ٓخ‬.‫غ ٓخػخص‬٠‫حُويٍ ٌٓ٘ ر‬ٝ
b. Immediate heparinization :‫؟‬ٚ‫ـذ حُزيء ر‬٣ ١ٌُ‫ح‬
c. Send the patient for arteriography ٍ‫خ‬ٜ‫ الٓظج‬ٞ٣َُٔ‫َ ح‬٤٠‫ طل‬.a
d. Identify and treat the underlying cause .ّ‫ٔخ‬ٜٗ‫حال‬
Patient avec une fibrillation auriculaire se présente avec une douleur .‫ٍح‬ٞ‫ٖ ك‬٣ٍ‫زخ‬٤ٛ ‫ اػطخء‬.b
aigue au niveau du membre inferieur gauche datant depuis quelques ٢٤‫ الؿَحء طوط‬ٞ٣َُٔ‫ حٍٓخٍ ح‬.c
heures progressant vers une perte de sensibilité et froideur. Quel est .٢ٗ‫خ‬٣َٗ
le traitement a débuter ? .‫ػالؽ حُٔزذ‬ٝ ‫ق‬٣َ‫ طؼ‬.d
a. Préparer le patient pour une embolectomie
b. Une héparinisation immédiate
c. Envoyer le patient pour une artériographie
d. Identifier et traiter la cause
162. A 59 y.o. man with history of hepatitis B, cirrhosis and recurrent ٖٓ ٢ٗ‫ؼخ‬٣ ‫ ػخٓخ‬59 َٔ‫زِؾ ٖٓ حُؼ‬٣ َ‫ٍؿ‬
pulmonary infections has a large right-sided pleural effusion. ‫خد‬ٜ‫حُظ‬ٝ ١‫ ط٘ٔغ ًزي‬،B ٢َٓٝ٤‫خد ًزي ك‬ٜ‫حُظ‬
Thoracentesis reveals a ratio of pleural protein to serum protein >0.5. ‫زخد ؿ٘خء‬ٜٗ‫ ٖٓ ح‬ٌٞ٘٣ ،ٌٍَ‫ ٓظ‬١ٞ‫ٍث‬
The likely explanation for these fndings is: ٕ‫يٍ أ‬ُٜ‫ٖ ٖٓ رٍِ ح‬٤‫ طز‬.َ٤‫ٖٔ ًز‬٣‫ ح‬٢‫ؿ٘ز‬
a. Hypoalbuminemia َُٜٔ‫ٖ ح‬٤‫ط‬َٝ‫ ر‬٠ُ‫ ا‬٢‫ٖ حُـ٘ز‬٤‫ط‬َٝ‫ٗٔزش حُز‬
b. Atelectasis ٌُٜٙ ‫َ حألٍؿق‬٤ٔ‫ حُظل‬.0.5 ٖٓ َ‫ أًز‬ٞٛ
c. Pneumonia :ٞٛ ‫ـش‬٤‫حُ٘ظ‬
d. Cirrhosis .ّ‫ٖ حُي‬٤ٓٞ‫ أُز‬ٚ‫ ٗو‬.a
e. Pulmonary infarction .‫ حَُثش‬ٙ‫ حٗؤخ‬.b
Un homme de 59 ans ayant une histoire d‟hépatite B, la cirrhose et .١ٞ‫خد ٍث‬ٜ‫ حُظ‬.c
des infections pulmonaires récurrentes a un grand épanchement .١‫ ط٘ٔغ ًزي‬.d
pleural droit. La thoracentèse révèle un rapport de la protéine pleurale .١ٞ‫ حكظ٘خء ٍث‬.e
à la protéine sérique > 0,5. L'explication probable de cette
constatation est:
a. Hypoalbuminémie
b. Atélectasie
c. Pneumonie
d. Cirrhose
e. Infarctus pulmonaire
163. The coagulation profile in a 13-year old girl with Menorrhagia having ٖٓ ٢ٗ‫ ػخٓخ طؼخ‬13 َٔ‫كظخس طزِؾ ٖٓ حُؼ‬
von Willebrand‟s disease is: .‫ِزَحٗي‬٣ٝ ٕٞ‫خرش ريحء ك‬ٜٓٝ ‫ٔغ‬١ ‫ؿِحٍس‬
A. Isolated prolonged PTT with a normal PT :١َ‫خ حُظوؼ‬ٜ‫َ ِٓل‬ٜ‫ظ‬٣
B. Isolated prolonged PT with a normal PTT PTٝ ٍِٝ‫ حُٔؼ‬PTT ٢‫خُش ك‬١‫ ا‬.a
C. Prolongation of both PT and PTT .٢‫ؼ‬٤‫ز‬١
D. Prolongation of thrombin time ُٖٓٝ ٍِٝ‫ حُٔؼ‬PT ٢‫خُش ك‬١‫ ا‬.b
Le profil de coagulation chez une jeune fille de 13 ans atteinte de .٢‫ؼ‬٤‫ز‬١ PTT
ménorragie et porteuse de la maladie de von Willebrand est: .PTTٝ PT ‫ ًال‬٢‫خُش ك‬١‫ ا‬.c
a. PTT allongé isolé avec un PT normal .ٖ٤‫ٓز‬َٝ‫ ُٖٓ حُؼ‬٢‫خُش ك‬١‫ ا‬.d
b. PT allongé isolé avec un PTT normal
c. Allongement des deux PT et PTT
d. Allongement du temps de thrombine

90
164. From which diameter size an abdominal aortic aneurysm should be َ‫ؼظز‬٣ ١ٌُ‫ش ح‬٣َٜ‫ كـْ هطَ أّ حُيّ حألر‬ٞٛ ‫ٓخ‬
considered for treatment : :‫رلخؿش ُِٔؼخُـش؟‬
a- 6 cm .ْٓ 6 .a
b- 5 cm .ْٓ 5 .b
c- 8 cm .ْٓ 8 .c
d- 4 cm .ْٓ 4 .d
Le Diamètre au delas duquel il faut opérer un anevrysme de l‟aorte
abdominal est :
a. 6cm
b. 5cm
c. 8cm
d. 4cm
165. A 41-year-old woman comes to the office for follow-up of abdominal ٠ُ‫َ ا‬٠‫ ػخٓخ طل‬41 َٔ‫حَٓأس طزِؾ ٖٓ حُؼ‬
liver function test results. The patient has a 3-year history of .‫ظخثق حٌُزي‬ٝ ٚ‫خىس ُٔظخرؼش ٗظخثؾ كل‬٤‫حُؼ‬
hypercholesterolemia treated with a statin medication that was ّ‫ٍ حُي‬َٝ‫ُٔظ‬ًٞ ‫ ٖٓ حٍطلخع‬٢ٗ‫ش طؼخ‬٠٣َُٔ‫ح‬
discontinued 6 months ago when a moderate elevation in hepatic ١ٌُ‫ح‬ٝ ٖ٤‫حء ٓظخط‬ٝ‫طؼخُؾ ري‬ٝ ‫حص‬ٞ٘ٓ 3 ٌ٘ٓ
transaminases was noted. She has no jaundice, abdominal metformin ‫َ ػ٘ي ٓالكظش حٍطلخع‬ٜٗ‫ أ‬6 ٌ٘ٓ ٚ‫هلظ‬ٝ‫أ‬
and lisinopril, both of which she has taken for several years. The ‫خ‬ٜ٣‫ْ ُي‬٤ُ .‫٘خُ حٌُزي‬٤ٓ‫ طَحٗٔؤ‬٢‫ ك‬٢ٓٞ‫ٓظ‬
patient drinks 1-2 glasses of wine on weekends and does not use ٖ٤ٍٓٞ‫ش ٗخطـش ػٖ حُٔظل‬٤٘‫ آالّ رط‬ٝ‫ أ‬،ٕ‫َهخ‬٣
tobacco or illicit drugs. There is no family history of liver disease. ‫ٔخ ُؼيس‬ُٜٝ‫ٔخ ًخٗض طظ٘خ‬ٛ‫ ًال‬،َ٣َ‫ر‬ٞ٘٣ُِِ‫ح‬ٝ
Vital signs are normal. BMI is 36kg/m². Examination shows mild ٖٓ ّ‫ ًؤ‬2 – 1 ‫ش‬٠٣َُٔ‫ٍ ح‬ٝ‫ طظ٘خ‬.‫حص‬ٞ٘ٓ
hepatomegaly. Laboratory results are as follows : ٠١‫ال طظؼخ‬ٝ ‫ع‬ٞ‫ش حألٓز‬٣‫خ‬ٜٗ ‫ ػطِش‬٢‫ٌ ك‬٤‫حُ٘ز‬
Albumin 4.0g/dL, Total bilirubin 1.0mg/dL, alkaline phosphatase ْ٤ُ .‫ػش‬َُٝ٘ٔ‫َ ح‬٤‫حُٔويٍحص ؿ‬ٝ‫ٖ أ‬٤‫حُظيه‬
121U/L, Asparate aminotransferase (SGOT) 82 U/L, Alanine .١‫ ًزي‬َُٝٔ ٢ِ‫ ػخث‬٢َٟٓ ‫ن‬٣ٍ‫٘خى طخ‬ٛ
aminotransferase (SGPT) 93 U/L. which of the following is the most ْٔ‫ ٓئَٗ ًظِش حُـ‬.‫ش‬٤‫ؼ‬٤‫ز‬١ ‫ش‬٣ٞ٤‫حإلٗخٍحص حُل‬
likely diagnosis in this patient ? ‫ى‬ٞ‫ؿ‬ٝ ٚ‫ٖ ٖٓ حُلل‬٤‫ظز‬٣.۲ّ/‫ ًؾ‬36 ٞٛ
a. Autoimmune hepatitis ٢ٛ ‫ش‬٣َ‫ حُ٘ظخثؾ حُٔوز‬.‫لش‬٤‫لخٍ هل‬١ ‫وخٓش‬ٟ
b. Hemochromatosis :٢ُ‫ًخُظخ‬
c. Hepatitis A infection 1 ٢ٌُِ‫ٖ ح‬٤‫ر‬َٝ٤ِ٤‫ حُز‬،َٓ‫ى‬/‫ؽ‬4 ٖ٤ٓٞ‫أُز‬
d. Nonalcoholic fatty liver disease ،َ‫ظ‬٤ُ/‫كيس‬ٝ 121 ‫ش‬٣ِٞ‫ٓلخطخُ ه‬ٞ‫ ك‬،َٓ‫ى‬/‫ِٓؾ‬
e. Primary biliary cholangitis 82 )SGOT( ُ‫َح‬٤‫طَحٗٔل‬ٞ٘٤ٓ‫أٓزخٍطخص أ‬
Une femme de 41 ans se présente au cabinet pour un suivi des ُ‫َح‬٤‫طَحٗٔل‬ٞ٘٤ٓ‫ٖ أ‬٤ٗ‫ أال‬،َ‫ظ‬٤ُ/‫كيس‬ٝ
résultats des tests de la fonction hépatique abdominale. La patiente a ٞٛ ٢ُ‫ ٖٓ حُظخ‬١‫ أ‬.َ‫ظ‬٤ُ/‫كيس‬ٝ 93 )SGPT(
une histoire de 3 ans d'hypercholestérolémie traitée avec un ٌٙٛ ‫لخ ػ٘ي‬٤‫ حألًؼَ طَؿ‬ٚ٤‫حُظ٘و‬
médicament à base de statine qui a été arrêté il y a 6 mois lorsqu'une :‫ش؟‬٠٣َُٔ‫ح‬
élévation modérée des transaminases hépatiques a été notée. Elle n'a .٢‫ ًحط‬٢‫ ٓ٘خػ‬١‫خد ًزي‬ٜ‫ حُظ‬.a
pas de jaunisse, de metformine abdominale et de lisinopril, qu'elle .‫ش‬٣ٞٓ‫زـش حُي‬ٛ‫ ىحء طَٓذ حأل‬.b
prend tous les deux depuis plusieurs années. Elle boit 1 à 2 verres de
.A ٢َٓٝ٤‫ ك‬١‫خد ًزي‬ٜ‫ حُظ‬.c
vin le week-end et ne consomme ni tabac ni drogues illicites. Il n'y a
pas d'antécédents familiaux de maladie du foie. Les signes vitaux sont .٢ُٞ‫َ حٌُل‬٤‫ ؿ‬٢٘ٛ‫ ىحء حٌُزي حُي‬.d
normaux. L'IMC est de 36 kg / m². L'examen montre une .٢‫ش حُزيث‬٣ٝ‫لَح‬ُٜ‫ش ح‬٤٘‫خد حأله‬ٜ‫ حُظ‬.e
hépatomégalie légère. Les résultats de laboratoire sont les suivants:
Albumine 4,0 g / dL, bilirubine totale 1,0 mg / dL, phosphatase
alcaline 121U / L, asparate aminotransférase (SGOT) 82 U / L,
alanine aminotransférase (SGPT) 93 U / L. lequel des éléments
suivants est le diagnostic le plus probable chez elle
a. Hépatite auto-immune
b. Hémochromatose
c. Infection par l'hépatite A
d. Maladie du foie gras non alcoolique
e. Cholangite biliaire primitive
91
166. An 82-year-old woman comes to the emergency department with ٠ُ‫َص ا‬٠‫ ػخٓخ ك‬82 َٔ‫حَٓأس طزِؾ ٖٓ حُؼ‬
sudden-onset epigastric pain. The pain started an hour ago and is 8/10 ‫ ٖٓ ريء ٓلخؿت‬ٌٞ٘‫ ط‬٢ٛٝ ‫حٍة‬ٞ‫هْٔ حُط‬
in severity. The patient has been nauseated and vomited twice in the ٞٛٝ ‫ ُوي ريأ حألُْ ٌٓ٘ ٓخػش‬.٢‫ك‬َٞٓٗ ُْ‫أل‬
past hour. Her past medical history includes type 2 diabetes mellitus,
hyperlipidemia, hypertension, peptic ulcer disease 10 years ago, and ٕ‫خ‬٤‫ش ُِـؼ‬٠٣َُٔ‫ض ح‬َٟ‫ ُوي طؼ‬.10/8 ‫ر٘يس‬
choletithiasis. She has a 40-pack-year smoking history and drinks ٖٔ٠‫ظ‬٣ .‫ش‬٤ٟ‫ حُٔخػش حُٔخ‬٢‫ٖ ك‬٤‫خء َٓط‬٤‫حإله‬ٝ
alcohol occasionally. Her temperature is 36.8C (98.2F), blood ،2 ٢ٔٗ ١ٌَٓ ‫ حُٔخرن ىحء‬٢َُٟٔ‫خ ح‬ٜ‫و‬٣ٍ‫طخ‬
pressure is 140/80mmHg, pulse is 90/min, and respirations are ‫ هَكش‬،ّ‫ حُي‬٢‫ـ‬ٟ ‫ حٍطلخع‬،ّ‫ّ حُي‬ٞ‫ ٗل‬١َ‫ك‬
14/min. the lughs are clear to auscultation. Abdominal examination ٢ٜ‫طل‬ٝ ،‫حص‬ٞ٘ٓ 10 ٌ٘ٓ ‫ش‬٤ٔ٠ٛ
shows a soft abdomen with no tenderness. Murphy‟s sign is negative. 40 ٍ‫ٖ رٔؼي‬٤‫ حُظيه‬٠١‫ طظؼخ‬٢ٛ .١ٝ‫لَح‬ٛ
Which of the following studies should be performed first in this .‫خٗخ‬٤‫ٍ أك‬ٞ‫طَ٘د حٌُل‬ٝ ‫ حُٔ٘ش‬٢‫ػِزش ك‬
patient ?
a. Abdominal ultrasound ِْٓ 80/140 ّ‫ حُي‬٢‫ـ‬ٟ ،ّ°36.8 ‫حُلَحٍس‬
b. Electrocardiogram ‫ حَُثش‬.‫ى‬/14 ْ‫حُظ٘ل‬ٝ ،‫ى‬/90 ٞ‫ حُ٘ز‬،‫ُثزن‬
c. Serum amylase and lipase ٚ‫َ حُلل‬ٜ‫ظ‬٣ .‫لش ػ٘ي حُظٔٔغ‬ٟ‫ح‬ٝ
d. Upper gastrointestinal endoscopy ٢‫ٍك‬ٞٓ ‫ ػالٓش‬.ّ‫ال‬٣‫ٕ ا‬ٝ‫ٖ ى‬٤ُ ٖ‫ رط‬١َ٣َُٔ‫ح‬
e. Upright abdominal x-ray ‫ـذ‬٣ ‫ش‬٤ُ‫ ٖٓ حُيٍحٓخص حُظخ‬١‫ أ‬.‫ش‬٤‫ِٓز‬
Une femme de 82 ans se présente au service des urgences avec une :‫ش؟‬٠٣َُٔ‫ ح‬ٌٙٛ ‫ال ػ٘ي‬ٝ‫خ أ‬ٛ‫اؿَحإ‬
douleur épigastrique soudaine. La douleur a commencé il y a une .ٖ‫ص ُِزط‬ُٜٞ‫م ح‬ٞ‫ؿخص ك‬ٞٓ .a
heure et est de 8/10 de gravité. Le patient a eu des nausées et des
vomissements deux fois au cours de la dernière heure. Ses antécédents .٢‫َرخث‬ًٜ ‫ هِذ‬٢٤‫ طوط‬.b
médicaux incluent le diabète sucré de type 2, l'hyperlipidémie, .َُٜٔ‫زخُ ح‬٤ُٝ ُ‫ال‬٤ٓ‫ أ‬.c
l'hypertension, l'ulcère gastroduodénal il y a 10 ans et la cholétithiase. .١ِٞ‫ ػ‬١ٞ‫ ٓؼ‬١‫ ٓؼي‬٢ِ‫َ ىحه‬٤‫ ط٘ظ‬.d
Elle a une histoire de tabagisme de 40 ans et boit de l'alcool à .ٖ‫ٔش ُِزط‬٤‫ش ٓٔظو‬٤٘٤ٓ ‫ أٗؼش‬.e
l'occasion. Sa température est de 36,8 ° C (98,2 ° F), sa tension
artérielle de 140/80 mmHg, son pouls de 90 / min et sa respiration de
14 / min. les rires sont clairs à l'auscultation. L'examen abdominal
montre un abdomen mou sans sensibilité. Le signe de Murphy est
négatif. Laquelle des études suivantes doit être réalisée en premier
chez ce patient?
a. Échographie abdominale
b. Électrocardiogramme
c. Amylase et lipase sériques
d. Endoscopie gastro-intestinale haute
e. Radiographie abdominale debout
167. A middle-aged man was recently diagnosed with hypertension. The ‫ ٓئهَح‬ٜٚ٤‫ق حُؼَٔ طْ ط٘و‬ٜ‫ ٓ٘ظ‬٢‫ٍؿَ ك‬
prescribed antihypertensive medication enhances natriuresis serum ٢‫ـ‬٠ُِ ٞ‫حء حُوخك‬ٝ‫ حُي‬.ّ‫ حُي‬٢‫ـ‬ٟ ‫رخٍطلخع‬
angiotensin II concentration, and decreases aldosterone production. II ٖ٤ٔ٘‫ط‬ٞ٤‫ِ أٗـ‬٤ًَ‫ؼُِ ط‬٣ ‫ف‬ُٞٛٞٔ‫ح‬
This medication most likely belongs to which of the following drug
classes ? ‫وَِ ٖٓ حٗظخؽ‬٣ٝ ،ّٞ٣‫ى‬ُِٜٞ ٍ‫َ حُٔي‬ُٜٔ‫رخ‬
a. Aldosterone receptor antagonist ٕ‫لخ أ‬٤‫حء حألًؼَ طَؿ‬ٝ‫ٌح حُي‬ٛ .َٕٝ٤‫ٓظ‬ٝ‫حألُي‬
b. Alpha adrenergic blocker :‫ش؟‬٤ُ‫ش حُظخ‬٣ٝ‫ ٖٓ كجخص حألى‬١‫ أل‬٢ٔ‫٘ظ‬٣
c. Angiotensin II receptor blocker .َٕٝ٤‫ٓظ‬ٝ‫خى ٓٔظوزَ حألُي‬٠ٓ .a
d. Direct renin inhibitor .‫ كخؿذ أُلخ‬.b
e. Loop diuretic ٖ٤ٔ٘‫ط‬ٞ٤‫ كخؿذ ٓٔظوزَ أٗـ‬.c
Un homme d'âge moyen a récemment eu un diagnostic .II
d'hypertension. Le médicament antihypertenseur prescrit augmente la
concentration sérique d'angiotensine II de natriurèse et diminue la .َٗ‫ٖ ٓزخ‬٤ٍٗ ٢‫ ٓؼز‬.d
production d'aldostérone. Ce médicament appartient probablement à .‫س‬َٝ‫ ٓيٍ ػ‬.e
laquelle des classes de médicaments suivantes?
a. Antagoniste des récepteurs d'aldostérone
b. Bloqueur alpha-adrénergique
c. Bloqueur des récepteurs de l'angiotensine II
d. D.Inhibiteur direct de la rénine
e. Diurétique de l'anse

92
168. A 39-year-old woman comes to the physician with a “pins and ٠ُ‫َ ا‬٠‫ ػخٓخ طل‬39 َٔ‫حَٓأس طزِؾ ٖٓ حُؼ‬
needles” sensation around her mouth for the last 2-3 weeks. She َ‫ ٖٓ اكٔخّ " ار‬ٌٞ٘‫ ط‬٢ٛٝ ‫ذ‬٤‫حُطز‬
occasionally has similar sensations in her feet, along with muscle 3-2 ‫غ‬٤‫خ هالٍ حألٓخر‬ٜٔ‫ٍ ك‬ٞ‫ْ" ك‬٤‫ىرخر‬ٝ
cramps, especially at the end of the day. Her post medical history is
unremarkable, and she does not use tobacco, alcohol or illicit drugs. ٢‫خٗخ اكٔخّ ٓٔخػَ ك‬٤‫خ أك‬ٜ٣‫ ُي‬.‫ش‬٤ٟ‫حُٔخ‬
Family history is not significant. Vital signs are normal, and ،‫ش‬٤ِ٠‫ػ‬ ‫ ؿ٘ذ ٓغ ط٘٘ـخص‬٠ُ‫ ؿ٘زخ ا‬،‫خ‬ٜ٤ٓ‫هي‬
examination is unremarkable. Laboratory findings results are as ٢َُٟٔ‫خ ح‬ٜ‫و‬٣ٍ‫ طخ‬.ّٞ٤ُ‫ش ح‬٣‫خ‬ٜٗ ٢‫ش ك‬ٛ‫هخ‬
follows : ٠١‫ ال طظؼخ‬٢ٛٝ ،‫ظ‬ٞ‫َ ِٓل‬٤‫حُٔخرن ؿ‬
Complete blood count : hemoglobin 13.2g/dL, leukocytes 6.300/µL. .‫ػش‬َٝ٘ٓ َ٤‫ ٓويٍحص ؿ‬ٝ‫ٍ أ‬ٞ‫ حٌُل‬،ٖ٤‫حُظيه‬
Serum chemistry : sodium 140mEq/L, potassium 4.0mEq/L, chloride ‫ش‬٣ٞ٤‫ حُؼالٓخص حُل‬.ٍُ‫َ رخ‬٤‫ ؿ‬٢ِ‫ن حُؼخث‬٣ٍ‫حُظخ‬
100mEq/L, bicarnonate 24mEq/L, blood urea nitrogen 10mEq/L, .‫جخ‬٤ٗ ‫ِلع‬٣ ُْ ١َ٣َُٔ‫ ح‬ٚ‫حُلل‬ٝ ،‫ش‬٤‫ؼ‬٤‫ز‬١
creatinine4 0.8mg/dL, glucose 100mg/L, calcium 6.5mg/L,
phosphorus 5.8mg/L, total protein 7.0g/dL, albumin 4.0g/dL. Which ‫ طؼيحى‬:٢ُ‫شًخُظخ‬٣َ‫ حُٔوز‬ٙٞ‫ٗظخثؾ حُلل‬
of the following is the most likely cause of this patient‟s condition ? / ‫ؽ‬ 13.2 ٖ٤‫ر‬ِٞ‫ؿ‬ٞٔ٤ٛ :َٓ‫خص حُيّ حٌُخ‬٣ًَ
a. Decreased secretion of parathyroid hormone ‫خء‬٠٤‫خص حُيّ حُز‬٣ًَ ،َٓ‫ى‬
b. Impaired 1-hydroxylation of vitamin D ّٞ٣‫ى‬ٞٛ :َُٜٔ‫خء ح‬٤ٔ٤ً .َ‫ظ‬٤ٌَُٝ٤ٓ/6300
c. Inadequate availability of vitamin D ٢ِِٓ 4 ّٞ٤ٓ‫طخ‬ٞ‫ ر‬،َ‫ظ‬٤ُ/‫ ٌٓخكت‬٢ِِ٤ٓ 140
d. Indadequate calcium intake / ‫ ٌٓخكت‬٢ِِ٤ٓ 100 ‫ي‬٣ًٍِٞ ،َ‫ظ‬٤ُ /‫ٌٓخكت‬
e. Precipitation of calcium in solt tissues ،َ‫ظ‬٤ُ / ‫ ٌٓخكت‬٢ِِ٤ٓ 24 ‫ٗخص‬ٞ‫ٌَر‬٤‫ ر‬،َ‫ظ‬٤ُ
Une femme de 39 ans se présente chez le médecin avec une sensation
de «picotements et d'aiguilles» autour de la bouche depuis 2 à 3 ،َ‫ظ‬٤ُ / ‫ ٌٓخكت‬٢ِِ٤ٓ 10 ّ‫خ حُي‬٣ٍٞ٣ ٖ٤‫ؿ‬َٝ‫ٗظ‬
semaines. Elle a parfois des sensations similaires dans ses pieds, ainsi ‫ ِٓؾ‬100 ًُِٞٞ‫ ؿ‬،َٓ‫ ى‬/ ‫ ِٓؾ‬0.8 ٖ٤٘‫خط‬٣ًَ
que des crampes musculaires, surtout à la fin de la journée. Ses ٖ٤ٓٞ‫ أُز‬،َٓ‫ ى‬/ ‫ ؽ‬7 ٢ٌُِ‫ٖ ح‬٤‫ط‬َٝ‫ حُز‬،َ‫ظ‬٤ُ /
antécédents médicaux sont sans particularité et elle ne consomme ni َ‫ حُٔزذ حألًؼ‬ٞٛ ٢ُ‫ ٖٓ حُظخ‬١‫ أ‬.َٓ‫ ى‬/ ‫ ؽ‬4
tabac, ni alcool, ni drogues illicites. Les antécédents familiaux ne sont :‫ش؟‬٠٣َُٔ‫ ح‬ٌٙٛ ‫لخ ُلخُش‬٤‫طَؿ‬
pas significatifs. Les signes vitaux sont normaux et l'examen est ‫ٕ ؿخٍحص‬ٍٞٓٞٛ ُ‫ اكَح‬ٚ‫ ٗو‬.a
banal. Les résultats des analyses de laboratoire sont les suivants:
Hémogramme: hémoglobine 13,2 g / dL, leucocytes 6,300 / µL. .‫حُيٍم‬
Chimie du sérum: sodium 140 mEq / L, potassium 4,0 mEq / L, . D ٖ٤ٓ‫ظخ‬٤‫ك‬ ‫ئًٍِش‬ٛ ‫ؼق‬ٟ .b
chlorure 100 mEq / L, bicarnonate 24 mEq / L, azote uréique sanguin .D ٖ٤ٓ‫ظخ‬٤‫َ ٓ٘خٓذ ُل‬٤‫كَ ؿ‬ٞ‫ ط‬.c
10 mEq / L, créatinine4 0,8 mg / dL, glucose 100 mg / L, calcium 6,5 .‫َ ٓ٘خٓذ‬٤‫ّ ؿ‬ٞ٤ُٔ‫ٍ ًخ‬ٞ‫ ٓيه‬.d
mg / L, phosphore 5,8 mg / L, protéines totales 7,0 g / dL, albumine ‫ حألٗٔـش‬٢‫ّ ك‬ٞ٤ُٔ‫ طَٓذ حٌُخ‬.e
4,0 g / dL. Lequel des éléments suivants est la cause la plus probable .‫٘ش‬٤ُِ‫ح‬
chez elle?
a. Diminution de la sécrétion de l'hormone parathyroïdienne
b. 1-hydroxylation altérée de la vitamine D
c. Disponibilité insuffisante de vitamine D
d. Apport de calcium insuffisant
e. Précipitation de calcium dans les tissus mous
169. Which of the following is the type of cancer that most often causes ‫ ؿخُزخ‬١ٌُ‫خٕ ح‬١َُٔ‫ع ح‬ٞٗ ٞٛ ٢ُ‫ ٖٓ حُظخ‬١‫أ‬
liver metastases? :‫ش؟‬٣‫ِش ًزي‬٤‫ٔزذ ٗو‬٣ ‫ٓخ‬
a. Uterine adenocarcinoma .‫ش‬٤ٔ‫ش ٍك‬٣‫ ؿي‬ٚٗ‫خ‬١َٓ .a
b. Squamous cell carcinoma of the esophagus ٢‫ش ك‬٤‫خ حُلَٗل‬٣‫ حُوال‬ٚٗ‫خ‬١َٓ .b
c. Colorectal adenocarcinoma .‫ء‬١َُٔ‫ح‬
d. Osteosarcoma .‫ش‬٤‫ش َٗؿ‬٤ُٗٞٞ‫ش ه‬٣‫ ؿي‬ٚٗ‫خ‬١َٓ .c
e. Testicular seminome .‫ش‬٤ٔ‫ٓخ ػظ‬ًٍٞ‫ ٓخ‬.d
Quel est, parmi les localisations suivantes, le type de cancer qui donne .١ٜٞ‫ ه‬١ٞ٘ٓ ٍّٝ .e
le plus des métastases hépatiques ?
a. Adénocarcinome utérin
b. Carcinome épidermoïde de l'oesophage
c. Adénocarcinome colorectal
d. Ostéosarcome
e. Seminome testiculaire

93
170. What is the incorrect item regarding neurofibromatosis ( von ٍُّٞ‫ق ر٘ؤٕ ح‬٤‫ل‬ُٜ‫َ ح‬٤‫ حُز٘ي ؿ‬ٞٛ ‫ٓخ‬
Recklinghausen disease) :‫ٖٓ) ؟‬ٝ‫خ‬ٛ ‫٘ؾ‬٤ًٍِ ٕٞ‫ (ىحء ك‬٢‫ل‬٤ُِ‫ ح‬٢‫ز‬ٜ‫حُؼ‬
a. Characterized by the presence of multiple benign ‫ش‬٤‫ل‬٤ُ ‫ش‬٤‫ز‬ٜ‫ٍحّ ػ‬ٝ‫ى أ‬ٞ‫ؿ‬ٞ‫ق ر‬ٜ‫ظ‬٣ .a
neurofibromas .‫يس ٓظؼيىس‬٤ٔ‫ك‬
b. Affects the bone, the nervous system, soft tissue, and the ،٢‫ز‬ٜ‫خُ حُؼ‬ٜ‫ حُـ‬،ْ‫ذ حُؼظ‬٤ٜ٣ .b
skin .‫حُـِي‬ٝ ،‫٘ش‬٤ُِ‫حألٗٔـش ح‬
c. Clinical symptoms decrease over time ٠ِ‫ش ػ‬٣َ٣َُٔ‫ ح‬ٝ‫ حألػَح‬ٚ‫ طظ٘خه‬.c
d. Neurologic problems and malignancy may develop .ُِٖٓ‫َٓ ح‬
e. It is an autosomal dominant neurogenetic disorder ٝ‫ش أ‬٤‫ز‬ٜ‫ٌٖٔ إٔ ط٘٘ؤ ٓ٘خًَ ػ‬٣ .d
Quel est le point incorrect concernant la neurofibromatose (maladie .‫هزخػش‬
de von Recklinghausen) ٢‫ز‬ٜ‫ ػ‬٢‫ٍحػ‬ٝ ‫طَحد‬ٟ‫ ح‬ٞٛ .e
a. Elle est caractérisée par la présence de multiples .ٖٔ٤ٜٓ ١‫ ؿٔي‬٢‫زـ‬ٛ
neurofibromes bénins
b. Affecte les os, le système nerveux, les tissus mous et la peau
c. Les symptômes cliniques diminuent avec le temps
d. Des problèmes neurologiques ou des tumeurs malignes
peuvent se développer
e. C'est un trouble neurogénétique autosomique dominant
171. Headache associated with a brain tumor is most common in which of ٌٕٞ٣ ‫ش‬٤ُ‫ش حُظخ‬٣َ٣َُٔ‫َ ح‬ٛ‫ح‬ٞ‫ ٖٓ حُظ‬١‫ أ‬٢‫ك‬
the following clinical settings? ٞٛ ٢‫ٍّ حُيٓخؿ‬ُٞ‫يحع حُٔظَحكن ٓغ ح‬ُٜ‫ح‬
a. Upon awakening in the morning :‫ػخ؟‬ٞ٤ٗ َ‫حألًؼ‬
b. After eating breakfast .‫زق‬ُٜ‫ ح‬٢‫وخظ ك‬٤‫ ػ٘ي حالٓظ‬.a
c. After eating lunch .٢‫زخك‬ُٜ‫ٍ ح‬ٞ‫ٍ حُلط‬ٝ‫ رؼي ط٘خ‬.b
d. After eating dinner .‫ٍ حُـيحء‬ٝ‫ رؼي ط٘خ‬.c
e. During the night .‫ٍ حُؼ٘خء‬ٝ‫ رؼي ط٘خ‬.d
Les maux de tête associés à une tumeur cérébrale sont les plus .‫ خالل الليل‬.e
fréquents dans lequel des contextes cliniques suivants?
a. Au réveil le matin
b. Après avoir pris le petit déjeuner
c. Après avoir déjeuné
d. Après avoir dîné
e. Au cours de la nuit
172. Among the following, which is the best blood test for active hepatitis َ٠‫ حألك‬١ٞٓ‫ حُي‬ٚ‫ حُلل‬ٞٛ ٢ُ‫ ٖٓ حُظخ‬١‫أ‬
B infection? :‫؟‬B ٢َٓٝ٤‫خد حٌُزي حُل‬ٜ‫خرش رخُظ‬ٛ‫ُإل‬
a. Anti-HBs Anti-HBs .a
b. Anti-HBc IgG Anti-HBc IgG .b
c. HBcAg HBcAg .c
d. HBsAg HBsAg .d
e. HBeAb HBeAb .e
Parmi les tests sanguins suivants, quel est le meilleur test sanguin
pour une infection active à l'hépatite B?
a. Anti-HBs
b. IgG anti-HBc
c. HBcAg
d. HBsAg
e. HBeAb

94
173. The pathophysiology of appendicitis include all the items except: ‫خد حُِحثيس‬ٜ‫ش الُظ‬٤َُٟٔ‫خ ح‬٤‫ؿ‬ُٞٞ٣ِ٤‫حُل‬
a. Lymphoid hyperplasia leads to luminal obstruction :‫ ٓخ ػيح‬٢ِ٣ ‫ش طَ٘ٔ ًَ ٓخ‬٣‫ى‬ٝ‫حُي‬
b. May be caused by human papilloma virus ‫ الٗٔيحى‬١‫ئى‬٣ ‫ش‬٣ٝ‫وْ حُِٔلخ‬٠‫ ط‬.a
c. Appendix distends, bacteria multiply .٢‫ُٔؼ‬
d. Visceral pain begins an average of 17 hours after obstruction ٍُّٞ‫ّ ح‬َٝ٤‫ ك‬ٚ‫ٕ ٓزز‬ٌٞ٣ ‫ هي‬.b
e. Increased pressure compromises blood supply .١َ٘‫ حُز‬٢ٔ٤ِ‫حُل‬
La physiopathologie de l'appendicite comprend tous les éléments .٢ٓٞ‫ طٌخػَ ؿَػ‬،‫ حٗظلخم ُحثيس‬.c
sauf: 17 ٍ‫زيأ رٔؼي‬٣ ‫ حألك٘خء‬٢‫ أُْ ك‬.d
a. L'hyperplasie lymphoïde entraîne une obstruction luminale .‫ٓخػش رؼي حالٗٔيحى‬
b. Peut être causé par le virus du papillome humain .ّ‫َ ربٓيحى حُي‬٠٣
ّ ٢‫ـ‬ٟ ‫خىس‬٣ُ .e
c. L'appendice se distend, les bactéries se multiplient
d. La douleur viscérale commence en moyenne 17 heures après
l'obstruction
e. L'augmentation de la pression compromet l‟irriguation
vasculaire
174. Cutaneous complications in patients with Crohn disease is represented ٠َُٟٔ‫ش ػ٘ي ح‬٣‫خػلخص حُـِي‬٠ُٔ‫طظٔؼَ ح‬
most frequently by : ٕ‫خ‬٤‫ ٓؼظْ حألك‬٢‫ٖ ك‬ًَٛٝ ‫ٖ ريحء‬٤‫خر‬ُٜٔ‫ح‬
a. Pyoderma gangrenosum :‫د‬
b. Erythema nodosum .٢٘٣َ‫ق حُـِي حُـ٘ـ‬٤‫ طو‬.a
c. Maculopapular rash .‫ ػويس‬٢ٓ‫ كٔخ‬.b
d. Vitiligo .٢١‫ كطخ‬٢‫لق روؼ‬١ .c
e. Chloasma .‫خم‬ٜ‫ ر‬.d
Les complications cutanées chez les patients atteints de la maladie de .‫ ًِق‬.e
Crohn sont le plus souvent représentées par:
a. Pyoderma gangrenosum
b. Érythème noueux
c. Éruption maculopapulaire
d. Vitiligo
e. Chloasma
175. Lung function abnormalities in pulmonary sarcoidosis is represented ‫ي‬٣ًٍٞ‫ حُٔخ‬٢‫ظخثق حَُثش ك‬ٝ ‫خص‬ٛٞ٘‫طظٔؼَ ط‬
mainly by : :‫ د‬٢ٓ‫ رٌَ٘ أٓخ‬١ٞ‫حَُث‬
a. Reduced forced vital capacity .‫ش‬٣َٔ‫ش حُو‬٣ٞ٤‫ حُويٍس حُل‬ٝ‫ حٗولخ‬.a
b. Asthma .ٞ‫ ٍر‬.b
c. Bronchospasm .‫زخص‬ٜ‫ ط٘٘ؾ ه‬.c
d. Increased respiratory rate .ْ‫خىس َٓػش حُظ٘ل‬٣ُ .d
e. Bronchiectasia ‫زخص‬ٜ‫ٓغ حُو‬ٞ‫ ط‬.e
Les anomalies de la fonction pulmonaire dans la sarcoïdose
pulmonaire sont principalement représentées par:
a. Capacité vitale forcée réduite
b. Asthme
c. Bronchospasme
d. Augmentation de la fréquence respiratoire
e. Bronchiectasie

95
176. When treating a 34 year old man with Crohn disease, all the items can ‫ ػخٓخ‬34 َٔ‫زِؾ ٖٓ حُؼ‬٣ َ‫ػ٘ي ٓؼخُـش ٍؿ‬
be likely to be beneficial in inducing remission except : ‫ ٖٓ حَُٔؿق‬٢ِ٣ ‫ ًَ ٓخ‬،ًَٖٛٝ ‫خد ريحء‬ٜٓ
a. Corticosteroids (oral) :‫يأس ٓخ ػيح‬ُٜ‫ِ ح‬٤‫ طلل‬٢‫ي ك‬٤‫ٕ ٓل‬ٌٞ٣ ٕ‫أ‬
b. NSAID .١ٞٔ‫ي ك‬٤‫ث‬َٝ٤‫ٓظ‬ٌٞ٤‫ٍط‬ًٞ .a
c. azathioprine َ٤‫خد ؿ‬ٜ‫خى حُظ‬٠ٓ .b
d. Methotrexate .١‫ي‬٤‫ث‬َٝ٤‫ٓظ‬
e. Infliximab .ٖ٣َ‫ر‬ٞ٤‫ أُحػ‬.c
Lors du traitement d'un homme de 34 ans atteint de la maladie de .‫طًَٔخص‬ٞ‫ظ‬٤ٓ .d
Crohn, tous les éléments peuvent être susceptibles d'être bénéfiques .‫ٔخد‬٤ٌٔ٤ِ‫ اٗل‬.e
pour induire une rémission sauf:
a. Corticostéroïdes (oraux)
b. AINS
c. azathioprine
d. Méthotrexate
e. Infliximab
177. A 41-year-old woman presents with amenorrhea for 9 months. She is ٖٓ ٌٞ٘‫ ػخٓخ ط‬41 َٔ‫حَٓأس طزِؾ ٖٓ حُؼ‬
found to have a prolactin- secreting pituitary adenoma. Laboratory ‫خرش‬ٜٓ ‫خ‬ٜٗ‫ٖ أ‬٤‫ طز‬.َٜٗ‫ أ‬9 ٌ٘ٓ ‫ٔغ‬١ ‫حٗوطخع‬
data reveals a serum calcium level of 12.0 mg/dL and hypo- glycemia ‫ طٌ٘ق‬.ٖ٤‫الًظ‬َٝ‫ ٓلَُ ُِز‬٢ٓ‫ٍّ ٗوخ‬ٞ‫ر‬
(serum glucose of 49 mg/dL). Which of the following is the most ّٞ٤ُٔ‫ ًخ‬ٟٞ‫ش ػٖ ٓٔظ‬٣َ‫ حُٔوز‬ٙٞ‫حُلل‬
likely diagnosis? ّ‫ ٌَٓ ى‬ٚ‫ٗو‬ٝ َٓ‫ ى‬/ ‫ ِٓؾ‬12 َٜٓ
a. MEN 1 syndrome ٖٓ ١‫ أ‬.)َٓ‫ ى‬/ ‫ ِٓؾ‬49 َُٜٔ‫ُ ح‬ًِٞٞ‫(ؿ‬
b. MEN 2A syndrome :‫لخ؟‬٤‫ حألًؼَ طَؿ‬ٚ٤‫ حُظ٘و‬ٞٛ ٢ُ‫حُظخ‬
c. MEN 2B syndrome .MEN 1 ‫ ٓظالُٓش‬.a
d. Sipple syndrome .MEN 2A ‫ ٓظالُٓش‬.b
e. Hypothyroidism .MEN 2B ‫ ٓظالُٓش‬.c
Une femme de 41 ans présente une aménorrhée depuis 9 mois. Elle a .َ‫ز‬٤ٓ ‫ ٓظالُٓش‬.d
un adénome hypophysaire sécrétant de la prolactine. Les données de .‫ حُيٍم‬١‫ ٗ٘خ‬ٚ‫ ٗو‬.e
laboratoire révèlent un taux de calcium sérique de 12,0 mg / dL et une
hypoglycémie (glucose sérique de 49 mg / dL). Lequel des énoncés
suivants est le diagnostic le plus probable?
a. Syndrome de MEN 1
b. Syndrome de MEN 2A
c. Syndrome de MEN 2B
d. Syndrome de Sipple
e. Hypothyroïdie

96
178. A 22-year-old woman presents with the chief complaint of hirsutism. ١َ‫ ٖٓ ك‬ٌٞ٘‫ ػخٓخ ط‬22 َٔ‫كظخس طزِؾ ٖٓ حُؼ‬
She has had irregular periods since menarche at the age of 13. She has ‫ش‬٣َٜٗ ‫ٍحص‬ٝ‫ ٖٓ ى‬٢ٗ‫ ًخٗض طؼخ‬.‫ش‬٤ٗ‫ٗؼَح‬
an ideal body weight and her facies is normal. Physical examination .13 ٖٓ ٢‫ش ك‬ٟ‫َ ٓ٘ظظٔش ٌٓ٘ ريء حإلكخ‬٤‫ؿ‬
reveals excess back and chest hair. Pelvic examination is normal. The ٚ‫ حُلل‬١‫زي‬٣ .٢‫ؼ‬٤‫ز‬١ ‫خ‬ٜٜ‫ؿ‬ٝٝ ٢ُ‫خ ٓؼخ‬ُٜٗٝ
luteinizing hormone (LH) value is elevated. Serum 17- OH .َٜ‫حُظ‬ٝ ٍ‫ي‬ُٜ‫ ح‬٢‫ي ك‬٣‫ ٗؼَ ٓظِح‬١َ٣َُٔ‫ح‬
progesterone concentrations are highly elevated. Which of the َُٕٜٞٓ‫ ح‬ٟٞ‫ ٓٔظ‬.٢‫ؼ‬٤‫ز‬١ ٝٞ‫ حُل‬ٚ‫كل‬
following is the most likely diagnosis? ٢ُِٜٔ‫ِ ح‬٤ًَ‫ حُظ‬.‫) َٓطلغ‬LH( ٖ‫ط‬ُِٞٔ‫ح‬
a. Cushing syndrome ٖٓ ١‫ أ‬.‫َٓطلغ ؿيح‬17-OH َٕٝ٤‫ؿٔظ‬َٝ‫ُز‬
b. Congenital adrenal hyperplasia :‫لخ؟‬٤‫ حألًؼَ طَؿ‬ٚ٤‫ حُظ٘و‬ٞٛ ٢ُ‫حُظخ‬
c. Adrenal tumor .‫٘ؾ‬٤ًٗٞ ‫ ٓظالُٓش‬.a
d. Idiopathic hirsutism .٢‫ هِو‬١َ‫ ط٘ٔؾ ًظ‬١َ‫ ك‬.b
e. Polycystic ovary disease .١َ‫ٍّ ًظ‬ٝ .c
Une femme de 22 ans présente un hirsutisme. Elle a eu des règles .‫ٍ حُٔزذ‬ٜٞ‫ش ٓـ‬٤ٗ‫ ٗؼَح‬١َ‫ ك‬.d
irrégulières depuis ses premières règles à l'âge de 13 ans. Elle a un .‫ٔخص‬٤ٌُ‫ ٓظؼيى ح‬ٞ٤‫ ىحء حُٔز‬.e
poids corporel idéal et son faciès est normal. L'examen physique
révèle un excès de poils dans le dos et la poitrine. L'examen pelvien
est normal. La valeur de l'hormone lutéinisante (LH) est élevée. Les
concentrations sériques de progestérone 17-OH sont très élevées.
Lequel des énoncés suivants est le diagnostic le plus probable?
a. syndrome de Cushing
b. Hyperplasie surrénale congénitale
c. Tumeur surrénale
d. Hirsutisme idiopathique
e. Maladie des ovaires polykystiques
179. Regarding acute renal failure, the hydro-electrolyte anomaly to be ًٌُٝ٘‫ ح‬،‫ حُلخى‬١ٌُِٞ‫ٍ ح‬ٜٞ‫ظؼِن رخُو‬٣ ‫ٔخ‬٤‫ك‬
urgently sought is: ٠ِ‫ ػ‬ٚ٘‫ـذ حُزلغ ػ‬٣ ١ٌُ‫ ح‬٢‫حُٔخث‬-٢‫ظ‬٤ٌَُُٜ‫ح‬
a. Hyponatremia :ٞٛ ‫ حَُٔػش‬ٚ‫ؿ‬ٝ
b. Hyperkalemia .ّ‫ّ حُي‬ٞ٣‫ى‬ٞٛ ٚ‫ ٗو‬.a
c. L`hypocalcémie .ّ‫ّ حُي‬ٞ٤ٓ‫طخ‬ٞ‫ ر‬١َ‫ ك‬.b
d. L‟hyperphosphatemia .ّ‫ّ حُي‬ٞ٤ُٔ‫ ًخ‬ٚ‫ ٗو‬.c
e. Global dehydration .ّ‫ٓلخص حُي‬ٞ‫ ك‬١َ‫ ك‬.d
Au cours de l'insuffisance rénale aiguë, l'anomalie hydro- .ّ‫ طـلخف ػخ‬.e
électrolytique à rechercher d'urgence est:
a. L'hyponatrémie
b. L'hyperkaliémie
c. L`hypocalcémie
d. L'hyperphosphatémie
e. La déshydratation globale

97
180. A 52-year-old woman comes to the clinic for follow up of type 2 ٠ُ‫َ ا‬٠‫ ػخٓخ طل‬52 َٔ‫حَٓأس طزِؾ ٖٓ حُؼ‬
diabetes mellitus that was diagnosed 6 months ago after she was ْ‫ ط‬١ٌُ‫ح‬ٝ 2 ٢ٔٗ ١ٌَٓ ‫خىس ُٔظخرؼش ىحء‬٤‫حُؼ‬
hospitalized for cellulitis of the right lower leg. The patient‟s diabetes
is managed with insulin in addition to diet and exercise. Her only ٠‫خ ُِٔٔظ٘ل‬ُٜٞ‫َ رؼي ىه‬ٜٗ‫ أ‬6 َ‫ هز‬ٜٚ٤‫ط٘و‬
other medical condition is hypertension for which she takes ‫ أٓلَ حُٔخم‬٢‫ ك‬١ِٞ‫ؾ حُو‬٤ُٔ٘‫خد ح‬ٜ‫رٔزذ حُظ‬
antihypertensive medication. The patient‟s mother had systemic lupus ٌُٜٙ ١ٌَُٔ‫ظْ ٓؼخُـش ىحء ح‬٣ .٠٘ٔ٤ُ‫ح‬
erythematosus and died at age 60. Serum creatinine is 1.7mg/dL.
Urine albumin/creatinine ratio is elevated at 190mg/g and was also ‫ش‬٤‫ش حُـٌحث‬٤ٔ‫خكش ُِل‬ٟ‫ٖ ا‬٤ُٞٔٗ‫ش رخإل‬٠٣َُٔ‫ح‬
elevated 3 months ago. Which of the following additional findings ‫ش‬٤َُٟٔ‫خ ح‬ٜ‫ كخُظ‬.‫ش‬٤ٟ‫خ‬٣َُ‫ٖ ح‬٣ٍ‫حُظٔخ‬ٝ
would most strongly a diagnosis of diabetic nephropathy in this ‫ؼخُؾ‬٣ ٢‫ـ‬ٟ ‫ حٍطلخع‬٢ٛ َٟ‫يس حأله‬٤‫ك‬ُٞ‫ح‬
patient?
a. Discrepancy in right and left kidney size ‫ش‬٠٣َُٔ‫حُيس ح‬ٝ ‫ ًخٗض‬.٢‫ـ‬ٟ ‫خص‬٠‫روخك‬
b. Left ventricular hypertrophy ‫ض‬٤‫ك‬ٞ‫هي ط‬ٝ ‫ش‬٣ُ‫خ‬ٜ‫ش ؿ‬٤ٓ‫ ٖٓ ًثزش كٔخ‬٢ٗ‫طؼخ‬
c. Microscopic hematuria / ‫ ِٓؾ‬1.7 َُٜٔ‫ٖ ح‬٤٘‫خط‬٣ًَ .60 ٖٓ ٢‫ك‬
d. Rapidly progressive renal dysfunction
e. Retinal neovascularization ‫ٖ َٓطلؼش‬٤٘‫خط‬٣ًَ / ٍٞ‫ٖ حُز‬٤ٓٞ‫ ٗٔزش أُز‬.َٓ‫ى‬
Une femme de 52 ans se présente à la clinique pour un suivi du َ‫ُوي ٓزن إٔ حٍطلؼض هز‬ٝ َٓ‫ ى‬/ ‫ ِٓؾ‬190 ٍ
diabète sucré de type 2 diagnostiqué il y a 6 mois après avoir été ٢ٛ ‫ش‬٤ُ‫ش حُظخ‬٤‫خك‬ٟ‫ ٖٓ حُ٘ظخثؾ حإل‬١‫ أ‬.َٜٗ‫ أ‬3
hospitalisée pour cellulite de la jambe droite. Le diabète du patient est
géré avec de l‟insuline en plus du régime et de l‟exercice. Sa seule ‫ ػ٘ي‬١ٌَٓ ١ًِٞ ٍ‫ الػظال‬ٟٞ‫ حأله‬ٚ٤‫حُظ٘و‬
autre condition médicale est l'hypertension pour laquelle elle prend :‫ش؟‬٠٣َُٔ‫ ح‬ٌٙٛ
des antihypertenseurs. La mère de la patiente avait un lupus ٠٘ٔ٤ُ‫ش ح‬٤ٌُِ‫ٖ كـْ ح‬٤‫ كَم ر‬.a
érythémateux disséminé et est décédée à 60 ans. La créatinine sérique
est de 1,7 mg / dL. Le rapport albumine / créatinine urinaire est élevé .َٟٔ٤ُ‫ح‬ٝ
à 190 mg/g et a également été élevé il y a 3 mois. Lequel des résultats .َٔ٣‫ٖ حأل‬٤‫وْ حُزط‬٠‫ ط‬.b
supplémentaires suivants constituerait le plus fortement un diagnostic .‫ش‬٣َٜ‫ش ٓـ‬٣ٞٓ‫ِش ى‬٤‫ ر‬.c
de néphropathie diabétique chez ce patient ?
a. Écart de taille des reins droit et gauche .ْ‫غ حُظلخه‬٣َٓ ١ًِٞ َ٘‫ ك‬.d
b. Hypertrophie ventriculaire gauche .٢ٌ‫غ ٗز‬٣‫ع كي‬ ّ ٞ‫ ط‬.e
c. Hématurie microscopique
d. Dysfonction rénale rapidement progressive
e. Néovascularisation rétinienne

98

Vous aimerez peut-être aussi